SlideShare uma empresa Scribd logo
1 de 37
Baixar para ler offline
GI I ðÁP TOÁN C P 3 – THI ð I H C
CÁC BÀI TOÁN TRONG
TAM GIÁC, T GIÁC
CÁC BÀI TOÁN V
ðƯ NG TH NG
CÁC BÀI TOÁN V
ðƯ NG TRÒN
CÁC BÀI TOÁN V ELIP
CÁC HƯ NG TƯ DUY VÀ PHƯƠNG PHÁP GI I
TRONG HÌNH H C OXY
Biên so n: Thanh Tùng
*) Tóm t t lý thuy t ñ y ñ theo m t trình t logic và có h th ng.
*) ðưa ra các hư ng tư duy và phương pháp gi i khái quát cho t ng l p bài toán.
*) Có bài toán m u minh h a ñi kèm.
*) Ph n bài t p áp d ng có g i ý.
*) L i gi i chi ti t cho t ng bài toán c th
(tham kh o thêm trên http://www.facebook.com/giaidaptoancap3 ).
BÀI TOÁN TÌM ðI M
H À N I 0 3 / 2 0 1 3
2
CÁC HƯ NG TƯ DUY VÀ PHƯƠNG PHÁP GI I HÌNH H C OXY
A. KI N TH C CƠ B N
3
4
B. CÁC BÀI TOÁN
BÀI TOÁN 1: BÀI TOÁN TÌM ðI M
ð hi u rõ hơn cho 4 hư ng tư duy tương ng v i 4 TH c a Bài toán 1: “Bài Toán Tìm ði m” th y s
dùng 6 bài thi ð i H c năm 2012 v a qua ñ minh h a.
1) (A, A1 – 2012:CB). Cho hình vuông ABCD. G i M là trung ñi m c a c nh BC, N là ñi m trên c nh CD sao cho
CN = 2ND. Gi s
11 1
;
2 2
M
 
 
 
và ñư ng th ng AN có phương trình 2 3 0x y− − = . Tìm t a ñ ñi m A.
2) (A, A1 – 2012 :NC). Cho ñư ng tròn 2 2
( ) : 8C x y+ = . Vi t phương trình chính t c c a elip (E), bi t r ng (E) có
ñ dài tr c l n b ng 8 và (E) c t ( )C t i b n ñi m phân bi t t o thành b n ñ nh c a m t hình vuông.
3) (B – 2012:CB). Cho ñư ng tròn 2 2
1( ) : 4C x y+ = , 2 2
2( ) : 12 18 0C x y x+ − + = và ñư ng th ng : 4 0d x y− − = .
Vi t phương trình ñư ng tròn có tâm thu c 2( )C , ti p xúc v i d và c t 1( )C t i hai ñi m phân bi t A và B sao cho AB
vuông góc v i d.
4) (B – 2012 :NC). Cho hình thoi ABCD có AC = 2BD và ñư ng tròn ti p xúc v i các c nh c a hình thoi có phương
trình 2 2
4x y+ = . Vi t phương trình chính t c c a elip (E) ñi qua các ñ nh A, B, C, D c a hình thoi. Bi t A thu c Ox.
5) (D – 2012:CB). Cho hình ch nh t ABCD. Các ñư ng th ng AC và AD l n lư t có phương trình là 3 0x y+ = và
4 0x y− + = ; ñư ng th ng BD ñi qua ñi m
1
( ;1)
3
M − . Tìm t a ñ các ñ nh c a hình ch nh t ABCD.
6) (D – 2012 :NC). Cho ñư ng th ng : 2 3 0d x y− + = . Vi t phương trình ñư ng tròn có tâm thu c d , c t tr c Ox
t i A và B, c t tr c Oy t i C và D sao cho AB = CD = 2.
5
1) (A, A1 – 2012:CB). Cho hình vuông ABCD. G i M là trung ñi m c a c nh BC, N là ñi m trên c nh CD sao cho
CN = 2ND. Gi s
11 1
;
2 2
M
 
 
 
và ñư ng th ng AN có phương trình 2 3 0x y− − = . Tìm t a ñ ñi m A.
Cách 1
Phân tích: :
+) Ta có { }A AN AM= ∩ nên Theo hư ng tư duy 1 (TH1) ta ph i ñi l p thêm phương trình AM
+) Bi t M nhưng chưa bi t A (chính là ñáp s ta c n tìm) nên ta ph i ñi tìm thêm vtpt ho c vtcp
+) Bài toán không có y u t song song, vuông góc ñ tìm vtpt ho c vtcp nên ta ph i khai thác yt ñ nh lư ng
+) Y u t ñ nh lư ng: cos MAN∠ = ( )cos ,AM ANn n
uuuur uuur
AMn⇒
uuuur
⇒ phương trình AM → t a ñ ñi m A
Gi i:
ð t AB a=
2
; ;
3 3 2
a a a
ND NC MB MC⇒ = = = = ( vì ABCD là hình vuông và 2CN ND= )
Và áp d ng Pitago ta ñư c:
5 5
;
2 6
a a
AM MN= = và
10
3
a
AN =
Trong AMN∆ ta có: cos MAN∠
2 2 2
2
2 . 2
AM AN MN
AM AN
+ −
= =
G i ( ; )AMn a b=
uuuur
là vtpt c a AM và ta có (2; 1)ANn = −
uuur
cos⇒ MAN∠ = ( )cos ,AM ANn n
uuuur uuur
2 2 2 2 2
2 2 2 2
322
2(2 ) 5( ) 3 8 3 0 (3 )( 3 ) 0
32 . 2 1
a ba b
a b a b a ab b a b a b
a ba b
= −− 
⇔ = ⇔ − = + ⇔ − − = ⇔ + − = ⇔  =+ + 
+) V i 3a b= − ch n 1; 3a b= = − (1; 3)AMn⇒ = −
uuuur
⇒ phương trình
11 1
: 3 0
2 2
AM x y
   
− − − =   
   
hay : 3 4 0AM x y− − = . Vì { }A AN AM= ∩ nên ta gi i h :
2 3 0 1
(1; 1)
3 4 0 1
x y x
A
x y y
− − = = 
⇔ ⇒ − 
− − = = − 
+) V i 3a b= ch n 3; 1a b= = (3;1)AMn⇒ =
uuuur
⇒ phương trình
11 1
:3 0
2 2
AM x y
   
− + − =   
   
hay :3 17 0AM x y+ − = . Vì { }A AN AM= ∩ nên ta gi i h :
2 3 0 4
(4;5)
3 17 0 5
x y x
A
x y y
− − = = 
⇔ ⇒ 
+ − = = 
V y (1; 1)A − ho c (4;5)A
6
Cách 2:
Phân tích: A AN∈ nên Theo hư ng tư duy 2 (TH2) ta g i ( )A t AN∈ ta c n thi t l p 1 phương trình ( ) 0f t =
(còn d ki n
11 1
;
2 2
M
 
 
 
là trung ñi m c a BC ta chưa s d ng – s giúp ta làm ñi u này) ?t A→ = →
Gi i:
+) G i H là hình chi u c a M lên AN
2 2
11 1
2. 3
3 52 2
( , )
22 1
MH d M AN
− −
⇒ = = =
+
ð t AB a=
2
; ;
3 3 2
a a a
ND NC MB MC⇒ = = = = ( vì ABCD là hình vuông và 2CN ND= )
Và áp d ng Pitago ta ñư c:
5 5
;
2 6
a a
AM MN= = và
10
3
a
AN =
Trong AMN∆ ta có: cos MAN∠
2 2 2
2
2 . 2
AM AN MN
AM AN
+ −
= =
⇒ MAN∠ = 0
45 MAH⇒ ∆ c n t i H
3 5 3 10
2 2.
2 2
AM MH⇒ = = = (*)
+) G i ( ;2 3)A t t AN− ∈ và 2 45
2
AM = (theo (*))
⇔
2 2
2 1 (1; 1)11 7 45
2 5 4 0
4 (4;5)2 2 2
t A
t t t t
t A
= −    
− + − = ⇔ − + = ⇔ ⇒     =     
V y (1; 1)A − ho c (4;5)A
Cách 3:
Phân tích: A AN∈ và
11 1
;
2 2
M
 
 
 
c ñ nh . N u AM h const= = ( ta s tìm cách ñi tính AM ).
Nên Theo hư ng tư duy 3 (TH3) : { } ( )A AN C= ∩ v i ( )C là ñư ng tròn tâm M bán kính R h=
7
Gi i: +) G i H là hình chi u c a M lên AN
2 2
11 1
2. 3
3 52 2
( , )
22 1
MH d M AN
− −
⇒ = = =
+
ð t AB a=
2
; ;
3 3 2
a a a
ND NC MB MC⇒ = = = = ( vì ABCD là hình vuông và 2CN ND= )
Và áp d ng Pitago ta ñư c:
5 5
;
2 6
a a
AM MN= = và
10
3
a
AN =
Trong AMN∆ ta có: cos MAN∠
2 2 2
2
2 . 2
AM AN MN
AM AN
+ −
= =
⇒ MAN∠ = 0
45 MAH⇒ ∆ c n t i H
3 5 3 10
2 2.
2 2
AM MH⇒ = = =
V y
3 10
2
AM = ⇒ A n m trên ñư ng tròn có phương trình:
2 2
11 1 45
2 2 2
x y
   
− + − =   
   
Mà : 2 3 0A AN x y∈ − − = Nên ta xét h :
2 2
11 1 45
1
2 2 2
1
2 3 0
xx y
y
x y
    =− + − =    ⇔    
= − − − =
ho c
4
5
x
y
=

=
V y (1; 1)A − ho c (4;5)A
Cách 4: (Các em có th tham kh o thêm cách gi i c a B Giáo D c nhưng vì cách gi i này theo th y không ñư c
“t nhiên” nên th y không trình bày ñây)
2) (A, A1 – 2012 :NC). Cho ñư ng tròn 2 2
( ) : 8C x y+ = . Vi t phương trình chính t c c a elip (E), bi t r ng (E) có
ñ dài tr c l n b ng 8 và (E) c t ( )C t i b n ñi m phân bi t t o thành b n ñ nh c a m t hình vuông.
Phân tích:
+) Phương trình ( )E :
2 2
2 2
1
x y
a b
+ = như v y ta c n tìm ;a b +) (E) có ñ dài tr c l n b ng 8 2 8 4a a⇒ = ⇒ =
+) Theo Hư ng tư duy 4 (TH4) ta g i ( ; )A x y ( 0x > ) là m t giao ñi m c a (E) và ( )C : 2 2
( ) 8A C x y∈ ⇒ + =
và d ki n (E) c t ( )C t i b n ñi m phân bi t t o thành b n ñ nh c a m t hình vuông giúp ta thi t l p thêm phương
trình: y x= (4 ñ nh n m trên hai ñư ng phân giác thu c góc ph n tư th nh t và th hai – nhưng vì ta ch n ñi m
( ; )A x y ( 0x > ) thu c góc ph n tư th nh t)⇒ t a ñ ñi m A +) Mà ( )A E b∈ ⇒ → phương trình (E).
Gi i: G i phương trình chính t c c a elip ( )E có d ng:
2 2
2 2
1
x y
a b
+ =
+) (E) có ñ dài tr c l n b ng 8 2 8 4a a⇒ = ⇒ =
+) G i ( ; )A x y ( 0x > ) là m t giao ñi m c a (E) và ( )C .Ta có: 2 2
( ) 8A C x y∈ ⇒ + = (1)
M t khác: (E) c t ( )C t i b n ñi m phân bi t t o thành b n ñ nh c a m t hình vuông ⇒ y x= (2)
T (1) và (2) 2
2 8 2x x⇒ = ⇒ = (vì 0x > ) 2 (2;2)y A⇒ = ⇒
+) Mà ( )A E∈
2 2
2
2 2
2 2 16
1
4 3
b
b
⇒ + = ⇒ = . V y phương trình chính t c c a elip (E) là:
2 2
1
1616
3
x y
+ =
8
3) (B – 2012:CB). Cho ñư ng tròn 2 2
1( ) : 4C x y+ = , 2 2
2( ) : 12 18 0C x y x+ − + = và ñư ng th ng : 4 0d x y− − = .
Vi t phương trình ñư ng tròn có tâm thu c 2( )C , ti p xúc v i d và c t 1( )C t i hai ñi m phân bi t A và B sao cho AB
vuông góc v i d.
Phân tích:
Mu n vi t phương trình ñư ng tròn ta c n:
+) Xác ñ nh tâm I (dùng Thu t Toán Tìm ði m) . Khi ñó theo Hư ng tư duy 2 (TH2) ta g i 1( )I t II∈
(Trư c ñó ta ñi l p phương trình 1II ñi qua 1I vuông góc v i AB (tính ch t ñư ng n i tâm) hay song song v i d )
Và d ki n 2( )I C∈ giúp ta thi t l p ñư c phương trình : ( ) 0 ?f t t= → = → t a ñ ñi m I
( Ta có th làm theo Hư ng tư duy 3 (TH3) v i { } 1 2( )I II C= ∩ → t a ñ I - cách trình bày khác c a TH2)
+) Xác ñ nh bán kính: R nh ( , )R d I d=
Gi i:
G i I là tâm ñư ng tròn ( )C c n vi t phương trình. Ta có 2 2
1( ) : 4C x y+ = ⇒ tâm c a 1( )C là 1(0;0)I
Vì
1
1
II AB
II
AB d
⊥
⇒
⊥
// d ⇒ phương trình 1II : 0x y− = .
G i 1( ; )I t t II∈ mà 2( )I C∈ 2 2
12 18 0t t t⇒ + − + = 2
6 9 0 3t t t⇔ − + = ⇔ = (3;3)I⇒
Mà ( )C ti p xúc v i d ⇒
2 2
3 3 4
( , ) 2 2
1 1
R d I d
− +
= = =
+
. V y phương trình( )C là: 2 2
( 3) ( 3) 8x y− + − =
4) (B – 2012 :NC). Cho hình thoi ABCD có AC = 2BD và ñư ng tròn ti p xúc v i các c nh c a hình thoi có phương
trình 2 2
4x y+ = . Vi t phương trình chính t c c a elip (E) ñi qua các ñ nh A, B, C, D c a hình thoi. Bi t A thu c Ox.
Phân tích: +) Phương trình ( )E :
2 2
2 2
1
x y
a b
+ = ( 0)a b> > như v y ta c n tìm ;a b
+) Theo Hư ng tư duy 2 (TH2) vì (E) ñi qua các ñ nh A, B, C, D và A Ox∈ nên g i ( ;0)A a Ox∈ và (0; )B b Oy∈
+) Khai thác d ki n: AC = 2BD 1( , ) 0f a b→ = (1)
+) Khai thác d ki n: ñư ng tròn 2 2
4x y+ = ti p xúc v i các c nh c a hình thoi 2 ( , ) 0f a b→ = (2)
T (1) và (2) 2
?a→ = và 2
?b = → phương trình (E).
9
Gi i: G i phương trình chính t c c a elip ( )E :
2 2
2 2
1
x y
a b
+ = ( v i 0a b> > )
Vì (E) ñi qua các ñ nh A, B, C, D và A Ox∈ nên không m t tính t ng quát gi s : ( ;0)A a và (0; )B b .
Mà hình thoi ABCD có AC = 2BD 2 4 2OA OB OA OB⇔ = ⇔ = 2a b⇔ = (vì 0a b> > ) hay (2 ;0)A b , (0; )B b
G i H là hình chi u c a O lên AB
2OH R⇒ = = ( vì ñư ng tròn 2 2
4x y+ = ti p xúc v i các c nh c a hình thoi)
Xét tam giác OAB ta có: 2 2 2
1 1 1
OH OA OB
= + hay 2
2 2
1 1 1
5
4 4
b
b b
= + ⇔ = 2 2
4 20a b⇒ = =
V y phương trình chính t c c a elip ( )E là:
2 2
1
20 5
x y
+ =
5) (D – 2012:CB). Cho hình ch nh t ABCD. Các ñư ng th ng AC và AD l n lư t có phương trình là 3 0x y+ =
và 4 0x y− + = ; ñư ng th ng BD ñi qua ñi m
1
( ;1)
3
M − . Tìm t a ñ các ñ nh c a hình ch nh t ABCD.
Cách 1:
Phân tích: +) Theo Hư ng tư duy 1 (TH1) : { }A AC AD= ∩ → t a ñ ñi m A
+) Theo Hư ng tư duy 2 (TH2) : D AD∈ , B AB∈ nên ta g i 1 2( ), ( )D t B t (trư c ñó ta ñi l p pt AB )
+) G i { }I AC BD= ∩ ( I là trung ñi m c a AC và BD ) 1 2( , )I t t⇒ mà 1 1 2( , ) 0I AC f t t∈ ⇒ = (1)
Vì ,MB MD
uuur uuuur
cùng phương 2 1 2( , ) 0f t t⇒ = (2)
+) T (1) và (2)
1
2
?
?
t
t
=
⇒ ⇒
=
t a ñ c a , ,B D I và C
Gi i: Vì { }A AC AD= ∩ nên xét h :
3 0
4 0
x y
x y
+ =

− + =
3
1
x
y
= −
⇔ ⇒
=
( 3;1)A −
AB ñi qua A và vuông góc v i AD nên AB có phương trình:
3 1
2 0
1 1
x y
x y
+ −
= ⇔ + + =
−
G i 1 1( ; 2)B t t AB− − ∈ và 2 2( ; 4)D t t AD+ ∈ ( 1 2; 3t t ≠ − ) 2 1 2 1 2
;
2 2
t t t t
I
+ − + 
⇒  
 
: là trung ñi m c a BD
Mà 2 1 2 1
2 1 1 2
2
3. 0 2 3 0 2 3
2 2
t t t t
I AC t t t t
+ − +
∈ ⇒ + = ⇔ − + = ⇔ = + (*)
Có: 1 1 2 2
1 10
; 3 2 ; 2 6
3 3
MB t t t t
   
= + − − = + − −   
   
uuur
(theo (*)) và 2 2
1
; 3
3
MD t t
 
= + + 
 
uuuur
M t khác , ,B D M th ng hàng⇒ ,MB MD
uuur uuuur
cùng phương 2 2
2
2 2
6 10 2 6
2 1
3 1 3
t t
t
t t
+ − −
⇒ = = − ⇔ = −
+ +
1 1t⇒ =
⇒ (1; 3), ( 1;3)B D− − và (0;0)I ⇒ (3; 1)C − ( vì I là trung ñi m c a AC )
10
5) (D – 2012:CB). Cho hình ch nh t ABCD. Các ñư ng th ng AC và AD l n lư t có phương trình là 3 0x y+ =
và 4 0x y− + = ; ñư ng th ng BD ñi qua ñi m
1
( ;1)
3
M − . Tìm t a ñ các ñ nh c a hình ch nh t ABCD.
Cách 2:
Phân tích: +) Theo Hư ng tư duy 1 (TH1) : { }A AC AD= ∩ → t a ñ ñi m A
+) Do trong bài toán có nhi u tính ch t ñ i x ng nên ta nghĩ t i vi c tìm các ñi m ph liên quan. C th :
+) Ta tìm ñi m N ñ i x ng v i M qua ñư ng trung tr c d c a AD b ng cách vi t pt 'd ñi qua M song
song v i AD và { } 'N d AC= ∩ ⇒ pt trung tr c d c a AD ⇒ t a ñ trung ñi m ,I J c a AC và AD
⇒ t a ñ , ,C D B
Gi i: Vì { }A AC AD= ∩ nên xét h :
3 0
4 0
x y
x y
+ =

− + =
3
1
x
y
= −
⇔ ⇒
=
( 3;1)A −
Phương trình c a 'd ñi qua M song song AD có d ng:
1
( 1) 0 3 3 4 0
3
x y x y+ − − = ⇔ − + =
G i { } 'N d AC= ∩ nên ta xét h :
13 0 1
1;13 3 4 0 33
xx y
N
yx y
= −+ =   
⇔ ⇒ −   =− + =   
G i d là ñư ng trung tr c c a AD c t , ,MN AC AD l n lư t t i , ,H I J
⇒ , ,H I J l n lư t là trung ñi m , ,MN AC AD
5 5
;
4 4
H
 
⇒ − 
 
⇒ pt c a d :
5 5
0 0
4 4
x y x y
   
+ + − = ⇔ + =   
   
Ta có: }{I d AC= ∩ nên ta xét h : ( )
0 0
0;0
3 0 0
x y x
I
x y y
+ = = 
⇔ ⇒ 
+ = = 
⇒ (3; 1)C − ( I là trung ñi m c a AC )
và }{J d AD= ∩ nên ta xét h : ( )
0 2
2;2
4 0 2
x y x
J
x y y
+ = = − 
⇔ ⇒ − 
− + = = 
⇒ ( 1;3)D − ( J là trung ñi m c a AD )
⇒ (1; 3)B − ( I là trung ñi m c a BD )
6) (D – 2012 :NC). Cho ñư ng th ng : 2 3 0d x y− + = . Vi t phương trình ñư ng tròn có tâm thu c d , c t tr c Ox
t i A và B, c t tr c Oy t i C và D sao cho AB = CD = 2.
11
Phân tích: Mu n vi t phương trình ñư ng tròn ta c n:
+) Xác ñ nh tâm I (dùng Thu t Toán Tìm ði m) . Khi ñó theo Hư ng tư duy 2 (TH2) ta g i ( )I t d∈
Và d ki n AB CD= giúp ta thi t l p ñư c phương trình : ( ) 0 ?f t t= → = → t a ñ ñi m I
+) Xác ñ nh bán kính: R nh 2 2 2 2
R IA IH HA= = + v i ( , )IH d I Ox= và 1
2
AB
HA = =
Gi i:
+) G i I là tâm ñư ng tròn c n l p và g i ( ;2 3)I t t d+ ∈
+) Ta có AB CD=
2 3 3 ( 3; 3)
( , ) ( , ) 2 3
2 3 1 ( 1;1)
t t t I
d I Ox d I Oy t t
t t t I
+ = = − − −  
⇔ = ⇔ + = ⇔ ⇔ ⇒  + = − = − −  
+) V i ( 3; 3)I − − ( , ) 3 3IH d I Ox⇒ = = − = và ta có:
2
1
2 2
AB
AH = = = 2 2 2 2
10R IA IH HA⇒ = = + =
V y phương trình ñư ng tròn: 2 2
( 3) ( 3) 10x y+ + + = .
+) V i ( 1;1)I − ( , ) 1 1IH d I Ox⇒ = = = và ta có:
2
1
2 2
AB
AH = = = 2 2 2 2
2R IA IH HA⇒ = = + =
V y phương trình ñư ng tròn: 2 2
( 1) ( 1) 2x y+ + − = .
CHÚ Ý: Trư c khi vào ph n BÀI TOÁN 2 chúng ta có m t s quy ư c sau:
+) ( )M t ∈∆ : ta ràng bu c ñi m M theo m t n là t.
+) 1 2( , )M t t : ñi m M có t a ñ ph thu c vào hai n 1t và 2t .
+) 1 2( ; )M t t : ñi m M có t a ñ :
1
2
M
M
x t
y t
=

=
BÀI TOÁN 2: CÁC BÀI TOÁN LIÊN QUAN ð N BÀI TOÁN 1
D ng 1: Các bài toán trong tam giác, t giác
Lo i 1: Các bài toán v ð nh Tính
Lo i 1.1: Các bài toán v ñư ng trung tuy n, ñư ng cao, trung tr c
Bài 1: Bi t ñ nh A c a tam giác ABC và 2 trung tuy n BM, CN. Vi t phương trình các c nh c a ABC∆ .
Cách gi i:
Bài t p áp d ng (Các em hãy d a vào ý tư ng Bài 1 ñ gi i các ví d sau)
Ví d 1: Cho tam giác ABC có A(4; – 1) và phương trình hai ñư ng trung tuy n BM: 8x – y – 3 = 0,
CN: 14x – 13y – 9 = 0. Tìm t a ñ các ñ nh B, C. (ðs: B(1; 5), C(–4; – 5))
Ví d 2: Cho tam giác ABC có A(1; – 2) và phương trình hai ñư ng trung tuy n BM và CN l n lư t là
x – 6y + 3 = 0 và 5x – 6y – 1 = 0. Tính di n tích tam giác ABC. (ðs: 16ABCS∆ = (ñvdt))
12
Ví d 3: Cho tam giác ABC có tr ng tâm G(-2;0) bi t phương trình các c nh AB, AC theo th t là
4x + y + 14 = 0 và 2x + 5y – 2 = 0. Vi t phương trình BC. (ðs: x – 2y – 1 = 0 ; v i B( 3; 2)− − , C(1;0) )
Ví d 4: Cho hai ñư ng th ng 1d : x – y + 1 = 0, 2d : 2x + y – 1 = 0 và ñi m P(2; 1). Vi t phương trình
ñư ng th ng 3d qua P và c t 1d , 2d l n lư t t i A và B sao cho P là trung ñi m c a AB. (ðs: 4x – y – 7 = 0)
Ví d 5: Cho tam giác ABC có trung ñi m c a AB là I(1; 3), trung ñi m AC là J(-3; 0). ði m A thu c Oy và
ñư ng BC qua g c t a ñ O. Tìm t a ñ các ñ nh c a tam giác ABC. (ðs: A
9
0;
2
 
 
 
, B
3
2;
2
 
 
 
,
9
6;
2
 
− − 
 
)
( Các em tham kh o ph n gi i m u qua các Ví d 2, Ví d 3, Ví d 5 )
Ví d 2: Cho tam giác ABC có A(1; – 2) và phương trình hai ñư ng trung tuy n BM và CN l n lư t là
x – 6y + 3 = 0 và 5x – 6y – 1 = 0. Tính di n tích tam giác ABC.
Gi i:
+) G i B
3
;
6
t
t
+ 
∈ 
 
BM . Do N là trung ñi m c a AB
1
2 2
9
2 12
A B
N
A B
N
x x t
x
y y t
x
+ +
= =
⇒ 
+ − = =

⇒ N
1 9
;
2 12
t t+ − 
 
 
Mà N∈CN
1 9
5. 6. 1 0 3
2 12
t t
t
+ −
⇒ − − = ⇔ = − ⇒ B ( 3;0)−
+) T a ñ tr ng tâm G c a ∆ ABC là nghi m c a h :
1
6 3 0
2
5 6 1 0
3
x
x y
x y y
=
− + = 
⇔ 
− − = = 
⇒ G
2
1;
3
 
 
 
3 3 1 3 5
3 2 2 0 4
C G A B
C G A B
x x x x
y y y y
= − − = − + =
⇒ 
= − − = + − =
⇒ C (5;4) .V y phương trình BC:
3
2 3 0
8 4
x y
x y
+
= ⇔ − + =
G i H là chân ñư ng cao k t A xu ng BC ⇒ AH =
2 2
1 2.( 2) 3 8
( ; )
51 2
d A BC
− − +
= =
+
Ta có: BC = 2 2
8 4 4 5+ =
1 1 8
. . .4 5 16
2 2 5
ABCS AH BC∆⇒ = = = (ñvdt)
Ví d 3: Cho tam giác ABC có tr ng tâm G(-2;0) bi t phương trình các c nh AB, AC theo th t là
4x + y + 14 = 0 và 2x + 5y – 2 = 0. Vi t phương trình BC
Gi i:
13
+) T a ñ ñi m A là nghi m c a h :
4 14 0
2 5 2 0
x y
x y
+ + =

+ − =
4
2
x
y
= −
⇔ 
=
⇒ A( 4;2)−
+) G i B 1 1( ; 4 14)t t− − ∈AB và C 2 2(5 1; 2 )t t+ − ∈AC
+) Vì G là tr ng ∆ ABC nên ta có:
3
3
A B C G
A B C G
x x x x
y y y y
+ + =

+ + =
1 2 1 2 1
1 2 1 2 2
4 5 1 6 5 3 3
2 4 14 2 0 2 6 0
t t t t t
t t t t t
− + + + = − + = − = −  
⇒ ⇔ ⇔  
− − − = + = − =  
⇒
B(-3;-2)
C(1;0)



V y phương trình BC:
3 2
4 2
x y+ +
= ⇔ 2 1 0x y− − =
Ví d 5: Cho tam giác ABC có trung ñi m c a AB là I(1; 3), trung ñi m AC là J(-3; 0). ði m A thu c Oy và
ñư ng BC qua g c t a ñ O. Tìm t a ñ các ñ nh c a tam giác ABC.
Gi i:
G i A(0; ) Oya ∈
B(2;6 a)
C( 6; a)
−
⇒ 
− −
( Vì I(1; 3), J(-3; 0) l n lư t là trung ñi m c a AB và AC)
Ta có:
OB (2;6 a)
OC ( 6; a)
 = −

= − −
uuur
uuur Mà BC ñi qua g c t a ñ O hay O,B,C th ng hàng
2 6 9
6 2
a
a
a
−
⇒ = ⇔ =
− −
⇒ A
9
0;
2
 
 
 
, B
3
2;
2
 
 
 
, C
9
6;
2
 
− − 
 
Bài 2: Bi t ñ nh A c a tam giác ABC và 2 ñư ng cao BH và CK. Vi t phương trình các c nh.
Cách gi i: +) Vi t phương trình AB, AC v i
AB CK
AC BH
n u
n u
 =

=
uuur uuur
uuur uuur +) Tìm B, C v i
{ }
{ }
B AB BH
C AC CK
=

=
I
I
Bài t p áp d ng (Các em hãy d a vào ý tư ng Bài 2 ñ gi i các ví d sau)
Ví d 1: Cho tam giác ABC bi t ñ nh A(1; –3); phương trình hai ñư ng cao xu t phát t B và C l n lư t là
x+ 2y – 8 =0 và 3x + 5y – 1 = 0. Vi t phương trình c nh BC. (ðs: 3x – 2y – 8 = 0)
Ví d 2 (A – 2004): Cho hai ñi m A (0; 2) và B( 3− ; 1− ). Tìm t a ñ tr c tâm và t a ñ tâm ñư ng tròn ngo i ti p
c a tam giác OAB. (ðs: H( 3; 1)− , ( 3;1)I − )
14
Bài 3: Cho ñ nh A và hai ñư ng trung tr c 1 2,d d c a c nh AB và AC (ho c BC).Vi t phương trình các c nh.
TH1 TH2
Cách gi i: TH1: B, C l n lư t ñ i x ng v i A qua 1d và 2d
B
C

⇒ 

TH2: +) B ñ i x ng v i A qua 1d ⇒B
+) C ñ i x ng v i B qua 2d ⇒ C
Bài t p áp d ng (Các em hãy d a vào ý tư ng Bài 3 ñ gi i các ví d sau)
Ví d 1: Cho tam giác ABC có ñ nh A(1; 2) và hai ñư ng trung tr c c a c nh AB và AC l n lư t là x – 2y – 2 = 0
và x – y + 5 = 0. Vi t phương trình trung tuy n AM c a tam giác ABC. (ðs: y = 2)
Ví d 2 : Cho tam giác ABC có ñi m M(0; 3) thu c ño n AC; hai ñư ng trung tr c c a c nh AB và AC l n lư t có
phương trình là x – 2y – 2 = 0 và x – y + 5 = 0. Vi t phương trình c nh BC c a tam giác ABC bi t AC = 4AM.
(ðs: 4x + 3y – 6 = 0)
Bài 4: Bi t ñ nh A c a tam giác ABC và ñư ng cao BH, trung tuy n CM. L p phương trình các c nh.
Cách gi i:
Bài t p áp d ng (Các em hãy d a vào ý tư ng Bài 4 ñ gi i các ví d sau)
Ví d 1: Cho tam giác ABC có A(–4; – 5) và phương trình ñư ng cao BH: x + 2y – 2 = 0, ñư ng trung tuy n
CM: 8x – y – 3 = 0. Tìm t a ñ ñ nh B, C. (ðs: B(4; –1), C(1; 5))
Ví d 2: Cho tam giác ABC có phương trình c a trung tuy n AM và ñư ng cao BH l n lư t là: 2x + y – 3 = 0;
2x – y – 4 = 0. ði m
1 5
;
2 2
N
 
− 
 
thu c ño n BC và ñ nh C thu c ñư ng th ng d: x + y – 3 = 0. Vi t phương trình
ñư ng tròn ngo i ti p tam giác ABC bi t BC không song song v i hai tr c t a ñ . (ðs:
2 2
3 1 5
2 2 2
x y
   
− + + =   
   
)
Ví d 3: Cho tam giác ABC có ñ nh A(2;1) , ñư ng cao qua ñ nh B có phương trình là : x – 3y – 7 = 0 và ñư ng trung
tuy n qua ñ nh C có phương trình : x + y + 1 = 0.Xác ñ nh t a ñ c a B và C. (ðs: ( 2; 3), (4; 5)B C− − − )
Ví d 4: Cho tam giác ABC cân t i ñ nh A có tr ng tâm
4 1
;
3 3
G
 
 
 
, phương trình ñư ng th ng BC là x – 2y – 4 = 0 và
ñư ng th ng BG là 7x – 4y – 8 = 0.Tìm t a ñ các ñ nh A,B,C. (ðs:
16 19 52 8
; , (0; 2), ;
9 9 9 9
A B C
   
− −   
   
)
Ví d 5:Cho tam giác ABC có ñ nh A thu c ñư ng th ng d : x – 4y – 2 = 0 , c nh BC song song v i ñư ng th ng d.
Phương trình ñư ng cao BH : x + y + 3 = 0 và trung ñi m c a c nh AC là M(1;1). Tìm t a ñ các ñ nh A,B,C.
(ðs:
2 2 18 3 8 8
; , ; , ;
3 3 5 5 3 3
A B C
     
− − −     
     
)
15
Bài 5: Bi t ñ nh A và trung tuy n CC’, ñư ng trung tr c c a c nh BC. Tìm t a ñ B, C.
Cách gi i:
Bài t p áp d ng (Các em hãy d a vào ý tư ng Bài 5 ñ gi i các ví d sau)
Ví d 1: Cho tam giác ABC bi t A(5; 13). Phương trình ñư ng trung tr c c nh BC, ñư ng trung tuy n CC’
(C’ thu c AB) l n lư t là x + y – 6 = 0 và 2x – y + 1 = 0. Vi t phương trình c nh BC. (ðs: x – y + 2 = 0)
Ví d 2 : Cho tam giác ABC có ñư ng trung tr c c a c nh BC c t ñương th ng ñi qua AB t i ñi m (1;2)M và
song song v i ñư ng th ng 2 2013 0x y− + = bi t 2AB MA=
uuur uuur
và ñư ng trung tuy n xu t phát t ñ nh C có
phương trình 11 7 11 0x y+ + = . Tìm t a ñ 3 ñ nh c a tam giác ABC. (ðs:
2 11
(0;1), ( 2; 1), ;
5 5
A B C
 
− − − 
 
)
Bài 6: Bi t trung ñi m M c a AB và trung tuy n AN, ñư ng cao BH. Vi t phương trình các c nh c a ∆ ABC.
Cách gi i:
Bài t p áp d ng (Các em hãy d a vào ý tư ng Bài 6 ñ gi i các ví d sau)
Ví d 1: Tam giác ABC có ñư ng trung tuy n AN : x – y + 1 = 0, ñư ng cao BH : x + 2y – 1= 0, ño n AB có trung
ñi m M(1; 1). Vi t phương trình các c nh c a tam giác ABC. (ðs: AB: x = 1; AC: 2x – y = 0; BC: 3x – y – 3 = 0)
Ví d 2: Cho tam giác ABC có ñi m M(0; 3) là trung ñi m c a AB. Phương trình trung tuy n AN: 2x – y – 2 = 0,
ñư ng cao BH: x – 3y + 14 = 0.Vi t phương trình ñư ng tròn ngo i ti p tam giác ABC (ðs: 2 2
( 3) ( 3) 50x y+ + + = )
16
Bài 7: Bi t ñ nh A (ho c ñư ng cao xu t phát t A ñi qua ñi m N và tr ng tâm G thu c m t ñư ng th ng…)
c a tam giác ABC và trung tuy n BM, ñư ng cao BH. Vi t phương trình các c nh.
TH1 TH2
Cách gi i:
Bài t p áp d ng (Các em hãy d a vào ý tư ng Bài 7 ñ gi i các ví d sau)
Ví d 1:Cho tam giác ABC bi t ñ nh A(1; – 1), ñư ng cao và trung tuy n cùng xu t phát t B l n lư t có phương
trình: x + 2y – 3 = 0 và x + 3y – 5 = 0 . Vi t phương trình BC. (ðs: x – 4y + 9 = 0)
Ví d 2: Cho tam giác ABC bi t ñư ng cao BH và trung tuy n BM l n lư t có phương trình: 4x + 3y + 2 = 0; x – 1 =
0. Tính di n tích tam giác ABC bi t r ng tr ng tâm G c a tam giác thu c ñư ng th ng d: 2x + 3y – 1 = 0 và ñư ng cao
xu t phát t ñ nh A có hoành ñ âm ñi qua ñi m N(3; –3). (ðs: 5ABCS∆ = (ñvdt))
Ví d 3 (D – 2009): Cho tam giác ABC có M(2; 0) là trung ñi m c a c nh AB. ðư ng trung tuy n và ñư ng cao qua
ñ nh A l n lư t có phương trình là 7x – 2y – 3 = 0 và 6x – y – 4 = 0. Vi t phương trình ñư ng th ng AC.
(ðs: 3x – 4y + 5 = 0)
Ví d 4 (B – 2003): Cho tam giác ABC có AB = AC , BAC∠ = 900
. Bi t M(1; -1) là trung ñi m c nh BC và G
2
;0
3
 
 
 
là tr ng tâm c a tam giác ABC. Tìm t a ñ các ñ nh A, B, C. (ðs: A(0; 2), B(4; 0), C(–2; – 2))
Ví d 5 (A – 2009): Cho hình ch nh t ABCD có ñi m I(6; 2) là giao ñi m c a hai ñư ng chéo AC và BD. ði m
M(1; 5) thu c ñư ng th ng AB và trung ñi m E c a c nh CD thu c ñư ng th ng ∆ : x + y – 5 = 0. Vi t phương trình
ñư ng th ng AB. (ðs: x – 4y + 19 = 0 và y – 5 = 0)
Bài 8: S d ng ñi u ki n vuông góc (trư ng h p riêng c a Bài 19) ñ gi i bài toán.
Cách gi i:
*) G i t a ñ các ñi m (n u chưa bi t) liên quan t i y u t vuông góc theo m t n nh vào:
+) ñi m thu c ñư ng th ng.
+) ñi m có m i liên h v i ñi m khác: trung ñi m, tr ng tâm, th a mãn h th c véctơ…
17
*) “C t nghĩa” ñi u ki n vuông góc:
0
. 0
. 0
. 0
90 . 0
a b
a b
AB MN AB MN
n n
a b
u u
AMB MA MB
 ⊥ ⇔ =

 = 
⊥ ⇒ 
= 

∠ = ⇔ =
uuur uuuur
uur uur
uur uur
uuur uuur
( ) 0 ?f t t⇒ = ⇒ = ⇒ t a ñ các ñi m
Bài t p áp d ng (Các em hãy d a vào ý tư ng Bài 8 ñ gi i các ví d sau)
Ví d 1 (D – 2004): Cho tam giác ABC có các ñ nh A(-1; 0); B (4; 0); C(0;m) v i m ≠ 0. Tìm t a ñ tr ng tâm G c a
tam giác ABC theo m. Xác ñ nh m ñ tam giác GAB vuông t i G. (ðs: (1; ), 3 6
3
m
G m = ± )
Ví d 2 (D – 2008): Cho (P): 2
16y x= và ñi m A(1; 4). Hai ñi m phân bi t B, C (B và C khác A) di ñ ng trên (P) sao
cho BAC∠ = 0
90 . Ch ng minh r ng ñư ng th ng BC luôn ñi qua m t ñi m c ñ nh. (ðs: ñi m c ñ nh I(17; –4))
Ví d 3 (A – 2009): Cho hình ch nh t ABCD có ñi m I(6; 2) là giao ñi m c a hai ñư ng chéo AC và BD. ði m
M(1; 5) thu c ñư ng th ng AB và trung ñi m E c a c nh CD thu c ñư ng th ng ∆ : x + y – 5 = 0. Vi t phương trình
ñư ng th ng AB. (ðs: x – 4y + 19 = 0 và y – 5 = 0)
Ví d 4: Cho ñi m M(3; 3), vi t phương trình ñư ng th ng ñi qua I(2; 1) c t Ox, Oy l n lư t t i A và B sao cho tam
giác AMB vuông t i M. (ðs: x + 2y – 4 = 0 và x + y – 3 = 0).
CHÚ Ý:
Qua các bài toán trên liên quan t i y u t trung tuy n và ñư ng cao, ñư ng trung tr c các em có th rút ra ñư c m t
vài ñi u như sau (tuy ñơn gi n nhưng hư ng tư duy này s giúp chúng ta gi i quy t t t nh ng bài toán d ng trên):
+) N u M là trung ñi m c a AB
2
2
A B M
A B M
x x x
y y y
+ =
⇒ 
+ =
: nghĩa là khi có d ki n này s giúp chúng ta thi t l p ñư c 2
phương trình
+) AH là ñư ng cao c a BC: giúp chúng ta bi t ñư c phương c a ñư ng này n u bi t ñư ng kia.
+) d là trung tr c c a BC: nghĩa là B ñ i x ng v i C qua d.
Lo i 1.2: Các bài toán v ñư ng phân giác trong
Bài 9: Bi t ñ nh A và hai ñư ng phân giác trong BB’ và CC’. L p phương trình BC.
Cách gi i:
+) Tìm 1A ñ i x ng v i A qua BB’ 1A BC⇒ ∈ (1) +) Tìm 2A ñ i x ng v i A qua CC’ 2A BC⇒ ∈ (2)
+) T (1) và (2) ⇒ phương trình BC (chính là phương trình 1 2A A )
Bài t p áp d ng (Các em hãy d a vào ý tư ng Bài 9 ñ gi i các ví d sau)
Ví d 1: Trong m t ph ng t a ñ Oxy, cho tam giác ABC có ñ nh A(1; 1), phương trình ñư ng phân giác trong góc B,
góc C l n lư t là BD: 2x + y + 4 = 0; CE: x + 3y + 1 = 0. L p phương trình c nh BC. (ðs:x + 23y + 46 = 0)
Ví d 2: Trong m t ph ng t a ñ Oxy, cho tam giác ABC có phương trình ñư ng phân giác trong góc B, góc C l n
lư t là BD: 6x + 8y – 17 = 0; CE: x – 2y + 3 = 0, ñi m M
17
1;
7
 
− 
 
và N
1
1;
3
 
 
 
l n lư t thu c c nh AB, AC. Tìm t a
ñ các ñ nh c a tam giác ABC. (ðs:A(0; – 1), B
7
;3
6
 
− 
 
, C(3; 3))
18
Bài 10: Bi t ñ nh A và trung tuy n BM, phân giác trong BD. Vi t phương trình các c nh.
Cách gi i:
Bài t p áp d ng (Các em hãy d a vào ý tư ng Bài 10 ñ gi i các ví d sau)
Ví d 1: Cho tam giác ABC có ñ nh A(3; 4), ñư ng phân giác trong và trung tuy n xu t phát t ñ nh B l n lư t có
phương trình x – y + 1 = 0 và 2x + 3y – 4 = 0. Tìm t a ñ ñ nh C. (ðs:C(–1; 6))
Ví d 2: Cho tam giác ABC có ñ nh A(1; 2), ñư ng phân giác trong và trung tuy n xu t phát t ñ nh B l n lư t có
phương trình 2x + y – 1 = 0 và 2x + 3y – 3 = 0. Tìm t a ñi m D là chân ñư ng phân giác trong c a góc B xu ng AC.
( ðs: D( 5;11)− )
Bài 11: Bi t ñ nh A và trung tuy n BM, phân giác trong CD. Vi t phương trình các canh.
Cách gi i:
Bài t p áp d ng (Các em hãy d a vào ý tư ng Bài 11 ñ gi i các ví d sau)
Ví d 1: Trong m t ph ng Oxy cho tam giác ABC có ñ nh A(1; 2), ñư ng trung tuy n BM: 2x + y + 1 = 0 và phân
giác trong CD: x + y – 1 = 0. Vi t phương trình ñư ng th ng BC (ðs: 4x + 3y + 4 = 0).
Ví d 2: Cho tam giác ABC có chân ñư ng trung tuy n k t B xu ng AC là M(1; – 1), ñư ng phân giác trong c a
góc C là x + y – 2 = 0. Vi t phương trình c nh AC bi t ñi m N(–7; 7) thu c c nh BC. (ðs: 5x + 3y – 2 = 0)
Ví d 3 (D – 2011 ): Cho tam giác ABC có ñ nh B(– 4; 1), tr ng tâm G(1; 1) và ñư ng th ng ch a phân giác trong
c a góc A có phương trình x – y – 1 = 0. Tìm t a ñ các ñ nh A và C. (ðs: (4;3), (3; 1)A C − )
19
Bài 12: Bi t ñ nh A và ñư ng cao BH, phân giác trong BD. Vi t phương trình các c nh tam giác.
Cách gi i:
Bài t p áp d ng (Các em hãy d a vào ý tư ng Bài 12 ñ gi i các ví d sau)
Ví d : Tam giác ABC có A(-3;1), ñư ng cao BH, phân giác trong BD l n lư t có phương trình: x + 7y + 32 = 0 và
x + 3y + 12 = 0. Tìm t a ñ ñi m C. (ðs: C(–4; – 6) )
Bài 13: Bi t ñ nh A và ñư ng cao BH, phân giác trong CD. Vi t phương trình các c nh tam giác.
Cách gi i:
Bài t p áp d ng (Các em hãy d a vào ý tư ng Bài 13 ñ gi i các ví d sau)
Ví d 1: Cho tam giác ABC có C(–2; 3). ðư ng cao c a tam giác k t ñ nh A và ñư ng phân giác trong c a góc B
l n lư t là: 3x – 2y – 25 = 0; x – y = 0. Vi t phương trình c nh AC c a tam giác. (ðs: 8x + 7y – 5 = 0)
Ví d 2 (B – 2008): Trong m t ph ng v i h t a ñ Oxy, hãy xác ñ nh t a ñ ñ nh C c a tam giác ABC bi t r ng hình
chi u vuông góc c a C trên ñư ng th ng AB là ñi m H(– 1; – 1), ñư ng phân giác trong c a góc A có phương trình
x – y + 2 = 0 và ñư ng cao k t B có phương trình 4x + 3y – 1 = 0. (ðs:
10 3
;
3 4
C
 
− 
 
)
CHÚ Ý:
Như v y qua các bài toán liên quan ñ n ñư ng phân giác trong c a tam giác các em s nh n th y ta luôn tìm thêm
ñi m ñ i x ng v i ñi m ñã bi t t a ñ trên c nh k c a góc ch a phân giác qua phân giác ñó , và ñi m ñó s thu c
c nh k còn l i (ñây là ñ c ñi m luôn ñư c khai thác khi có bài toán ch a phân giác)
20
Lo i 2: Các bài toán v ð nh Lư ng
Bài 14: Bi t ñ nh A ho c tr ng tâm G c a tam giác ABC thu c m t ñư ng th ng d cho trư c.
Bi t t a ñ ñ nh B, C và di n tích tam giác ABC( ho c di n tích c a 1 trong 3 tam giác ABG, BCG, CAG) . Tìm
t a ñ ñ nh A.(N u bi t thêm trung tuy n AM thì thay d ki n bi t t a ñ B, C b i bi t ñư ng th ng BC và câu h i là
tìm t a ñ ñ nh B, C)
TH1 TH2
Cách gi i:
(trư ng h p riêng c a Bài 16)
Bài t p áp d ng (Các em hãy d a vào ý tư ng Bài 14 ñ gi i các ví d sau)
Ví d 1 (B – 2004): Cho hai ñi m A(1; 1), B(4; -3). Tìm ñi m C thu c ñư ng th ng x – 2y – 1 = 0 sao cho kho ng
cách t C ñ n ñư ng th ng AB b ng 6. ( ðs: (7;3)C ho c
43 27
;
11 11
C
 
− − 
 
)
Ví d 2: Cho tam giác ABC v i A(2; –1), B(1; –2), tr ng tâm G c a tam giác n m trên ñư ng th ng x + y – 2 = 0.
Tìm t a ñ ñ nh C bi t tam giác ABC có di n tích là 13,5. (ðs: C(15; –9) ho c C(–12;18))
Ví d 3: Cho tam giác ABC v i A(1; 1), B(–2; 5), tr ng tâm G thu c ñư ng th ng 1∆ : 2x + 3y – 1 = 0, ñ nh C thu c
ñư ng th ng 2∆ : x + y – 1 = 0. Tính di n tích tam giác ABC. (ðs: 6ABCS∆ = (ñvdt))
CHÚ Ý: Tam giác ABC có G là tr ng tâm thì:
1
3
ABG BCG CAG ABCS S S S∆ ∆ ∆ ∆= = =
21
Bài 15: Bi t ñ nh A và phương trình ñư ng th ng BC và hình chi u H c a A xu ng BC chia theo BH kHC=
uuur uuur
và bi t
di n tích tam giác ABC (ho c bi t ñ dài ño n BC). Tìm t a ñ B, C.
Cách gi i:
Bài t p áp d ng (Các em hãy d a vào ý tư ng Bài 15 ñ gi i các ví d sau)
Ví d 1:Cho tam giác ABC có ñ nh A(2; – 2) và phương trình ñư ng th ng BC là 3x – 4y + 1 = 0 và hình chi u H c a
A xu ng BC th a mãn 6HC BH= −
uuur uuur
. Tìm t a ñ các ñi m B và C, bi t di n tích tam giác ABC b ng 7,5 .
(ðs: B(1; 1), C(5; 4))
Ví d 2 (B – 2009 – NC): Cho tam giác ABC cân t i A có ñ nh A(–1;4) và các ñ nh B,C thu c ñư ng th ng
∆: x – y – 4 = 0. Xác ñ nh to ñ các ñi m B và C, bi t di n tích tam giác ABC b ng 18.
( ðs:
11 3 3 5
; , ;
2 2 2 2
B C
   
−   
   
ho c
3 5 11 3
; , ;
2 2 2 2
B C
   
−   
   
)
Ví d 3 (B – 2002): Cho hình ch nh t ABCD có tâm I
1
;0
2
 
 
 
, phương trình ñư ng th ng AB là x – 2y + 2 = 0 và
AB = 2AD. Tìm t a ñ các ñi m A, B, C, D bi t r ng A có hoành ñ âm. (ðs: ( 2;0), (2;2), (3;0), ( 1; 2)A B C D− − − )
Bài 16:
TH1: Bi t ñ nh A và phương trình ñư ng th ng BC, ñư ng th ng d ñi qua ñi m H thu c BC th a mãn BH kHC=
uuur uuur
và bi t di n tích tam giác ABC (ho c bi t ñ dài ño n BC). Tìm t a ñ B, C.
TH2: Bi t phương trình AC và bi t phương trình ñi qua A căt BC t i H (bi t A), bi t B ( ho c C) và th a mãn
BH.BC = k. Tìm ñ nh còn l i.
BH.BC = k
TH1 TH2
22
Cách gi i:
Bài t p áp d ng (Các em hãy d a vào ý tư ng Bài 16 ñ gi i các ví d sau)
Ví d 1:Cho tam giác ABC có ñ nh A(1; 1), c nh BC có phương trình 3x – 4y + 6 = 0. ðư ng th ng d c t ño n BC t i
ñi m H sao cho HC = 3BH. Xác ñ nh t a ñ ñi m B, C bi t ñư ng th ng d có phương trình x – 4y + 8 = 0 và tam giác
ABC có di n tích b ng 1,5. (ðs: B(2; 3), C(-2; 0) )
Ví d 2 (B – 2011): Cho hai ñư ng th ng ∆ : x – y – 4 = 0 và d: 2x – y – 2 = 0. Tìm t a ñ ñi m N thu c ñư ng th ng
d sao cho ñư ng th ng ON c t ñư ng th ng ∆ t i ñi m M th a mãn OM.ON = 8. (ðs: (0; 2)N − ho c
6 2
;
5 5
N
 
 
 
Bài 17: Bi t t a ñ ñi m A, ñư ng phân giác trong c a góc B và cho bi t ñ l n góc B và di n tích tam giác ABC.
Tìm t a ñ B,C
Cách gi i:
Bài t p áp d ng (Các em hãy d a vào ý tư ng Bài 17 ñ gi i ví d sau)
23
Ví d (B – 2010): Cho tam giác ABC vuông t i A có ñ nh C(– 4; 1), phân giác trong góc A có phương trình
x + y – 5 = 0. Vi t phương trình ñư ng th ng BC, bi t di n tích tam giác ABC b ng 24 và ñ nh A có hoành ñ dương.
Bài 18: Tam giác ABC cân t i A, bi t AB và BC n m l n lư t trên 2 ñư ng th ng 1 2,d d . Bi t M 0 0( ; )x y ∈ AC. Tìm
t a ñ các ñ nh.
Cách gi i:
C2:
+) Tìm 1 2{ }B d d= I
+) Vi t phương trình 3d qua M song song v i 2d
+) Tìm 1 3{ }N d d= I ⇒ phương trình trung tr c 4d c a MN 4 1{ }A d d⇒ = I
+) Vi t phương trình AM 2{C} AM d⇒ = I
NH N XÉT: C2 hay hơn C1
Bài t p áp d ng (Các em hãy d a vào ý tư ng Bài 18 ñ gi i các ví d sau)
Ví d 1: Cho tam giác ABC cân t i A có phương trình hai c nh BC, AB l n lư t là: x – 3y – 1 = 0 và x – y – 5 = 0.
ðư ng th ng AC ñi qua M(–4; 1). Tìm t a ñ ñ nh C. (ðs:
8 1
;
5 5
C
 
 
 
).
Ví d 2: Trong m t ph ng Oxy, hãy xác ñ nh t a ñ các ñ nh c a tam giác ABC vuông cân t i A. Bi t r ng c nh
huy n n m trên ñư ng th ng d: x + 7y – 31 = 0, ñi m N(7; 7) thu c ñư ng th ng AC, ñi m M(2; –3) thu c AB và
n m ngoài ño n AB. (ðs: ( 1;1), ( 4;5), (3;4)A B C− − )
Ví d 3: Cho tam giác ABC cân t i A có phương trình AB, BC l n lư t là y + 1 = 0 và x + y – 2 = 0. Tính di n tích
24
tam giác ABC bi t AC ñi qua ñi m M(–1; 2) (ðs: 8ABCS∆ = ).
Ví d 4 (A – 2010 – NC): Cho tam giác ABC cân t i A có ñ nh A(6; 6); ñư ng th ng ñi qua trung ñi m c a các c nh
AB và AC có phương trình x + y – 4 = 0. Tìm t a ñ các ñ nh B và C, bi t ñi m E(1; - 3) n m trên ñư ng cao ñi qua
ñ nh C c a tam giác ñã cho. (ðs: (0; 4), ( 4;0)B C− − ho c ( 6;2), (2; 6)B C− − )
Ví d 5 (B – 2007): Cho ñi m A(2; 2) và các ñư ng th ng 1d : x + y – 2 = 0, 2d : x + y – 8 = 0. Tìm t a ñ các ñi m
B và C l n lư t thu c 1d và 2d sao cho tam giác ABC vuông cân t i A. (ðs: ( 1;3), (3;5)B C− ho c (3; 1), (5;3)B C− )
Ví d 6 (B – 2011 – NC): Cho tam giác ABC có ñ nh B
1
;1
2
 
 
 
. ðư ng tròn n i ti p tam giác ABC ti p xúc v i các
c nh BC, CA, AB tương ng t i các ñi m D, E, F. Cho D(3; 1) và ñư ng th ng EF có phương trình y – 3 = 0. Tìm t a
ñ ñ nh A, bi t A có tung ñ dương. ( ðs:
13
3;
3
A
 
 
 
)
Bài 19: Các ñi m liên h v i nhau b i m t n và m t ñi u ki n v ñ nh lư ng
Cách gi i:
+) Khai thác d ki n bài toán ñ chuy n các ñi m v 1 n t (nh thu t toán tìm ñi m)
+) Thi t l p phương trình: ( ) 0 ?f t t= ⇒ = ⇒ các ñi m c n tìm.
CHÚ Ý: Bài 8 là trư ng h p ñ c bi t c a Bài 19 khi ñi u ki n ñ nh lư ng là ñi u ki n góc 0
90 (vuông góc).
Bài t p áp d ng (Các em hãy d a vào ý tư ng Bài 19 ñ gi i các ví d sau)
Ví d 1: Cho tam giác ABC vuông t i A. Hai ñi m A, B thu c tr c hoành. Phương trình c nh BC là 4x + 3y – 16 = 0.
Xác ñ nh t a ñ tr ng tâm G c a tam giác ABC, bi t bán kính ñư ng tròn n i ti p tam giác ABC b ng 1.
(ðs: G
4
2;
3
 
 
 
ho c G
4
6;
3
 
− 
 
)
Ví d 2 (A – 2002): Cho tam giác ABC vuông t i A, phương trình ñư ng th ng BC là 3x y 3 0− − = , các ñ nh A và B
thu c tr c hoành và bán kính ñư ng tròn n i ti p b ng 2. Tìm t a ñ tr ng tâm G c a tam giác ABC.
(ðs:
7 4 3 6 2 3
;
3 3
G
 + +
  
 
ho c
4 3 1 6 2 3
;
3 3
G
 − − − −
  
 
)
Ví d 3 (D – 2008): Cho (P): 2
16y x= và ñi m A(1; 4). Hai ñi m phân bi t B, C (B và C khác A) di ñ ng trên (P) sao
cho góc 0
90BAC∠ = . Ch ng minh r ng ñư ng th ng BC luôn ñi qua m t ñi m c ñ nh. (ðs: ñi m c ñ nh I(17; –4))
Ví d 4 (A – 2006): Cho các ñư ng th ng: 1d : x + y + 3 = 0, 2d : x – y – 4 = 0, 3d : x – 2y = 0. Tìm t a ñ ñi m M
n m trên ñư ng th ng 3d sao cho kho ng cách t M ñ n ñư ng th ng 1d b ng hai l n kho ng cách t M ñ n ñư ng
th ng 2d . (ðs: ( 22; 11)M − − ho c (2;1)M )
Ví d 5 (B – 2005): Cho hai ñi m A(2;0) và B(6;4). Vi t phương trình ñư ng tròn (C) ti p xúc v i tr c hoành t i
ñi m A và kho ng cách t tâm c a (C) ñ n ñi m B b ng 5.
(ðs: 2 2
( ):( 2) ( 1) 1C x y− + − = ho c 2 2
( ):( 2) ( 7) 49C x y− + − = )
Ví d 6 (A – 2005): Cho hai ñư ng th ng d1 : x – y = 0 và d2 : 2x + y – 1 = 0 tìm t a ñ các ñ nh c a hình vuông
ABCD bi t r ng ñ nh A thu c d1 , ñ nh C thu c d2 và các ñ nh B, D thu c tr c hoành.
(ðs: (1;1), (0;0), (1; 1), (2;0)A B C D− ho c (1;1), (2;0), (1; 1), (0;0)A B C D− )
Ví d 7 (D – 2006): Cho ñư ng tròn 2 2
( ) : 2 2 1 0C x y x y+ − − + = và ñư ng th ng d: x – y + 3 = 0. Tìm t a ñ
ñi m M n m trên d sao cho ñư ng tròn tâm M, có bán kính g p ñôi bán kính ñư ng tròn (C), ti p xúc ngoài v i ñư ng
tròn (C). ( ðs: (1;4)M ho c ( 2;1)M − )
Ví d 8 (D – 2004): Cho tam giác ABC có các ñ nh A(-1; 0); B (4; 0); C(0;m) v i m ≠ 0. Tìm t a ñ tr ng tâm G c a
25
tam giác ABC theo m. Xác ñ nh m ñ tam giác GAB vuông t i G. (ðs: (1; ), 3 6
3
m
G m = ± )
Ví d 9 (B – 2002): Cho hình ch nh t ABCD có tâm I
1
;0
2
 
 
 
, phương trình ñư ng th ng AB là x – 2y + 2 = 0 và
AB = 2AD. Tìm t a ñ các ñi m A, B, C, D bi t r ng A có hoành ñ âm. (ðs: ( 2;0), (2;2), (3;0), ( 1; 2)A B C D− − − )
D ng 2: Các bài toán v ñư ng th ng
Lo i 1: ði qua m t ñi m và th a mãn m t y u t ñ nh lư ng
Cách gi i chung:
C1:
+) G i phương trình ñi qua ñi m M 0 0( ; )x y có h s góc k có d ng:
0 0( )y k x x y= − + hay 0 0 0kx y y kx− + − = ( ∆ )
+) Sau ñó “c t nghĩa” d ki n v ñ nh lư ng ñ thi t l p phương trình: ( ) 0 ?f k k= ⇒ = ⇒ phương trình ∆ .
C2:
+) G i phương trình ñi qua ñi m M 0 0( ; )x y có vtpt ( ; )n a b=
r
( 2 2
0a b+ ≠ ) có d ng:
0 0( ) ( ) 0a x x b y y− + − = hay 0 0 0ax by ax by+ − − = ( ∆ )
+) Sau ñó “c t nghĩa” d ki n v ñ nh lư ng ñ thi t l p phương trình: ( , ) 0f a b a kb= → = (*)
+) T (*) ch n
?
?
a
b
=
⇒
=
phương trình ∆
CHÚ Ý: Chúng ta ñã s d ng cách này trong Bài 18
Bài t p áp d ng
Ví d 1: Trong m t ph ng t a ñ Oxy, cho hai ñi m M(1; 4) và N(6; 2). L p phương trình ñư ng th ng ∆ qua M sao
cho kho ng cách t N t i ∆ b ng 5. (ðs: 21 20 59 0x y− + = và x = 1).
Ví d 2: Trong m t ph ng t a ñ Oxy, cho hai ñi m A(1; 2) và B(5; –1). Vi t phương trình ñư ng th ng qua M(3; 5)
và cách ñ u A và B. (ðs: 3x + 4y – 29 = 0 và x = 3)
Ví d 3: Trong m t ph ng Oxy, cho ñi m M(1; 2). Vi t phương trình ñư ng th ng qua M c t Ox, Oy l n lư t t i hai
ñiêm A, B sao cho OAB là tam giác vuông cân. (ðs: x + y – 3 = 0 và x – y + 1 = 0)
Ví d 4: Trong m t ph ng Oxy, cho ñi m M(4; 3). Vi t phương trình ñư ng th ng qua M sao cho nó t o v i hai tr c
t a ñ m t tam giác có di n tích b ng 3. (ðs: 3 2 6 0x y− − = và 3x – 8y + 12 = 0).
Ví d 5: Trong m t ph ng Oxy, cho tam giác ABC có A(4; 1), B(1; 7) và C(-1; 0). Vi t phương trình ñư ng th ng ñi
qua C và chia tam giác thành hai ph n b ng nhau, ph n ch a ñi m A có di n tích g p ñôi ph n ch a ñi m B.
(ðs: 6x – 5y + 6 = 0).
Ví d 6: Trong m t ph ng Oxy, cho ba ñi m A( - 1; 2), B(5; 4) và M(2; 5). Vi t phương trình ñư ng th ng ñi qua M
và cách ñ u hai ñi m A và B (ðs: 5x – 3y + 13 = 0 và x = 2)
Ví d 7: Trong m t ph ng Oxy, cho ñi m M(9; 4). Vi t phương trình ñư ng th ng qua M, c t hai tia Ox và tia Oy t i
A và B sao cho:
1) tam giác OAB có di n tích nh nh t. (ðs: 4x + 9y – 72 = 0)
2) OB + OC nh nh t. (ðs: 4x + 9y – 72 = 0)
Ví d 8 : Trong m t ph ng Oxy, cho hình thoi ABCD có c nh AB n m trên ñư ng th ng x – 2y + 5 = 0 và ba ñi m
M(–1; 4), N(1; 1), P(–3; 3) l n lư t thu c các c nh BC, CD và AD. Vi t phương trình c nh AD.
(ðs: 2 3 0x y+ − = ho c 11 2 39 0x y− + = )
26
CHÚ Ý:
+) N u bài toán ñ c p t i các ñi m A(a; 0) và B(0; b) là các giao ñi m v i hai tr c t a ñ các em có th vi t
phương trình ñư ng th ng theo ño n ch n ñi qua AB: 1
x y
a b
+ =
+) N u A(a; 0) , B(0; b) thì OA = a và OB = b
Lo i 2: C t ñư ng tròn, Elip (xem D ng 3, D ng 4)
D ng 3: Các bài toán v ñư ng tròn
Lo i 1: Vi t phương trình ñư ng tròn và xác ñ nh các y u t c a ñư ng tròn
Bài 1: Thi t l p phương trình ñư ng tròn
Cách gi i chung:
C2: +) G i phương trình ñư ng tròn có d ng 2 2
0x y ax by c+ + + + =
+) Tìm a, b, c nh “c t nghĩa” d ki n bài toán
Bài t p áp d ng
Ví d 1: Vi t phương trình ñư ng tròn:
1) ñư ng kính AB v i A(3; 1) và (B(2; – 2).
2) Có tâm I(1; – 2) và ti p xúc v i ñư ng th ng d: x + y – 2 = 0.
3) Có bán kính b ng 5, tâm thu c tr c hoành và ñi qua A(2; 4).
4) Có tâm I(2; – 1) và ti p xúc ngoài v i ñư ng tròn: 2 2
( 5) ( 3) 9x y− + − =
5) có tâm n m trên ñư ng th ng ∆ và ti p xúc v i hai tr c t a ñ Ox, Oy.
6) qua A(–2; –1), B(–1; 4) và C(4; 3) (ñư ng tròn ngo i ti p tam giác ABC).
7) qua A(0; 2), B(–1; 1) và có tâm n m trên ñư ng th ng 2x + 3y = 0.
8) qua A(5; 3) và ti p xúc v i ñư ng th ng d: x + 3y + 2 = 0 t i ñi m T(1; –1).
9) N i ti p tam giác OAB bi t A(3; 0) và B(0; 4).
Ví d 2(A – 2007): Trong m t ph ng v i h t a ñ Oxy, cho tam giác ABC có A(0; 2), B(– 2; – 2) và C(4; – 2). G i H
là chân ñư ng cao k t B; M và N l n lư t là trung ñi m c a các c nh AB và BC. Vi t phương trình ñư ng tròn ñi
qua các ñi m H, M, N. ( ðs: 2 2 2
2 0x y z x y+ + − + − = )
Ví d 3(B – 2010 – NC): Trong m t ph ng t a ñ Oxy, cho ñi m A(2; 3 ) và (E):
2 2
1
3 2
x y
+ = . G i 1F và 2F là các
tiêu ñi m c a (E) ( 1F có hoành ñ âm); M là giao ñi m có tung ñ dương c a ñương th ng 1AF v i (E); N là ñi m ñ i
x ng c a 2F qua M. Vi t phương trình ñư ng tròn ngo i ti p tam giác 2ANF . (ðs:
2
2 2 3 4
( 1)
3 3
x y
 
− + − =  
 
)
27
Bài 2: Xác ñ nh tâm và bán kính. L p phương trình ti p tuy n c a ñư ng tròn
Cách gi i chung:
*) Xác ñ nh tâm và bán kính
trong ñó
2 2
0
2 2
a b
c
   
+ − >   
   
: ði u ki n t n t i ñư ng tròn.
C2: S d ng h ng ñ ng th c (tách ghép) ñưa ñư ng tròn v d ng:
trong ñó 0h > : ði u ki n t n t i ñư ng tròn.
*) L p phương trình ti p tuy n c a ñư ng tròn
C1: N u bi t ti p ñi m M ⇒ phương trình ti p tuy n d c a (C) t i M nh n IM
uuur
làm véc tơ pháp tuy n.
C2: N u không bi t ti p ñi m thì dùng ñi u ki n : ∆ là ti p tuy n c a (C) ( , )d I R⇔ ∆ =
Bài t p áp d ng
Ví d 1: Cho ñư ng tròn (C): 2 2
2 4 4 0x y x y+ − + − =
1) Tìm tâm và bán kính c a (C).
2) Cho A(3; – 1). Ch ng minh A là ñi m n m trong ñư ng tròn. Vi t phương trình ñư ng th ng qua A và c t (C)
theo m t dây cung có ñ dài nh nh t.
3) Cho d: 3x – 4y = 0. Ch ng minh d c t (C) t i hai ñi m phân bi t M, N và sau ñó tính MN.
Ví d 2(Các bài toán cơ b n: Vi t phương trình ti p tuy n t i m t ñi m cho trư c, có phương cho trư c và qua 1
ñi m cho trư c) Vi t phương trình ti p tuy n c a ñư ng tròn:
1) 2 2
( 3) ( 1) 25x y− + + = t i ñi m có hoành ñ b ng – 1
2) 2 2
4 2 5 0x y x y+ + − − = t i ñi m ñư ng tròn c t tr c Ox.
3) 2 2
2x y+ = có h s góc b ng 1.
4) 2 2
2 24 0x y y+ − − = bi t ti p tuy n vuông góc v i ñư ng th ng 3x – 4y + 2012 = 0.
5) có tâm I(2; 1), bán kính R = 3 và ñi qua ñi m A(–1; 2).
Lo i 2: S tương giao
Lo i 2.1: S tương giao gi a ñư ng th ng và ñư ng tròn
28
Bài 1: Vi t phương trình ñư ng th ng ∆ ñi qua M 0 0( ; )x y c t ñư ng tròn (C) t i A, B sao cho AB = l.
Cách gi i
+) G i ( ; )n a b∆ = ⇒
uur
phương trình ∆ : 0 0 0 0( ) ( ) 0 : ( ) 0a x x b y y ax by ax by− + − = ⇔ ∆ + − + =
+) T (*) ta ch n :
?
?
a
b
=
⇒
=
phương trình ∆ ( N u mu n tìm c th A, B ta gi i h :
( )C
∆


)
Bài t p áp d ng (Các em hãy d a vào ý tư ng Bài 1 ñ gi i các ví d sau)
Ví d 1: Cho ñư ng tròn 2 2
( ) : 6 4 12 0C x y x y+ − + − = . Vi t phương trình ñư ng th ng ∆ ñi qua M(1; 3) c t (C)
theo dây cung AB có ñ dài b ng 2 . (ðs: x – y + 2 = 0 và x + 41y – 124 = 0)
Ví d 2 (A – 2009 – NC): Cho ñư ng tròn 2 2
( ) : 4 4 6 0C x y x y+ + + + = và ñư ng th ng ∆ : 2 3 0x my m+ − + = ,
v i m là tham s th c. G i I là tâm c a ñư ng tròn (C). Tìm m ñ ∆ c t (C) t i hai ñi m phân bi t A và B sao cho
di n tích tam giác IAB l n nh t. (ðs: 0m = ho c
8
15
m = )
Ví d 3 (B – 2009 – NC): Cho tam giác ABC cân t i A có ñ nh A(–1;4) và các ñ nh B,C thu c ñư ng th ng
∆: x – y – 4 = 0. Xác ñ nh to ñ các ñi m B và C, bi t di n tích tam giác ABC b ng 18.
( ðs:
11 3 3 5
; , ;
2 2 2 2
B C
   
−   
   
ho c
3 5 11 3
; , ;
2 2 2 2
B C
   
−   
   
)
Ví d 4(D – 2009 – NC): Cho ñư ng tròn 2 2
( ) :( 1) 1C x y− + = . G i I là tâm c a (C). Xác ñ nh t a ñ ñi m M thu c
(C) sao cho IOM∠ = 0
90 . ( ðs:
3 3
;
2 2
M
 
  
 
ho c
3 3
;
2 2
M
 
−  
 
)
Ví d 5: Cho ñư ng tròn 2 2
( ) : 4 2 15 0C x y x y+ − + − = . G i I là tâm ñư ng tròn (C). Vi t phương trình ñư ng
thăng ∆ qua M(1; –3) c t (C) t i A, B sao cho tam giác IAB có di n tích b ng 8 và c nh AB là c nh l n nh t.
(ðs: 4x + 3y + 5 = 0 và y + 3 = 0)
Ví d 6: Cho ñư ng tròn 2 2
( ) :( 1) ( 2) 4C x y− + − = và ñi m M(2; 1). Vi t phương trình ñư ng th ng ∆ qua M c t
(C) t i 2 ñi m A, B sao cho
1) Dây cung AB l n nh t. (ðs: x + y – 3 = 0)
2) Dây cung AB ng n nh t. (ðs: x – y – 1 = 0)
Ví d 7: Cho ñư ng tròn (C) : 2 2
1x y+ = .ðư ng tròn ( C’) tâm I(2;2) c t (C) t i 2 ñi m A,B sao cho 2AB = .
Vi t phương trình ñư ng th ng AB. ( ðs: 1 0x y+ + = ho c 1 0x y+ − = )
29
Bài 2: Vi t phương trình ñư ng th ng ∆ bi t 0 0( ; )n a b∆ =
uur
(ho c ph i tìm nh quan h song song ho c vuông góc) c t
ñư ng tròn (C) t i 2 ñi m phân bi t A, B và th a mãn m t ñi u ki n v ñ nh lư ng.
Cách gi i:
+) Phương trình ∆ có 0 0( ; )n a b∆ =
uur
: 0 0 0a x b y m+ + = 0
0
a x m
y
b
− −
⇒ = (*) (n u 0
0
0
m
b x
a
−
= ⇒ = )
+) Thay (*) vào phương trình ñư ng tròn (C) 2
0ax bx c⇒ + + = (2*) (phương trình ch a tham s m)
+) G i 1 1 2 2 1 2( ; ), ( ; ) ,A x y B x y x x⇒ là nghi m c a phương trình (2*). N u 1 2,x x bi u di n theo m :
Bài t p áp d ng (Các em hãy d a vào ý tư ng Bài 2 ñ gi i các ví d sau)
Ví d 1(D – 2011 – NC): Cho ñi m A(1; 0) và ñư ng tròn (C): 2 2
2 4 5 0x y x y+ − + − = . Vi t phương trình ñư ng
th ng ∆ c t (C) t i hai ñi m M và N sao cho tam giác AMN vuông cân t i A. (ðs: 1y = ho c 3y = − )
Ví d 2(D – 2010 – CB ): Cho tam giác ABC có ñ nh A(3; –7), tr c tâm H(3; –1), tâm ñư ng tròn ngo i ti p là I(–2;
0). Xác ñ nh t a ñ ñ nh C, bi t C có hoành ñ dương. (ðs: ( 2 65;3)C − + )
Ví d 3: Cho ñư ng tròn 2 2
( ) :( 2) ( 3) 10C x y− + − = . Xác ñ nh t a ñ các ñ nh c a hình vuông ngo i ti p ñư ng
tròn, bi t c nh AB ñi qua ñi m ( 3; 2)M − − và ñ nh A có hoành ñ dương. ( ðs: A(6; 1), B(0; –1), C(–2; 5), D(4; 7))
Bài 3: Tìm ñi m M thu c ñư ng th ng ∆ và cách m t ñi m c ñ nh I m t kho ng không ñ i (MI = R)
Cách gi i : Có th hi u bài toán này theo 2 cách (b n ch t là m t)
C2: T a ñ ñi m M là nghi m c a h :
( )C
∆


( ñây (C) là ñư ng tròn tâm I bán kính R)
CHÚ Ý:
+)V i C1 chúng ta không c n quan tâm t i bài toán v s tương giao gi a ñư ng th ng và ñư ng tròn
(ñ c p C2) và gi i theo phương pháp ñ i s thông thư ng.
+) V i C2 ta th y rõ hơn b n ch t c a bài toán.
+) C1 và C2 là hai cách trình bày khác nhau c a cùng m t phương pháp th trong gi i h phương trình.
+) Có th chúng ta chưa nhìn th y luôn ñi m I . Khi ñó ñ bài thư ng cho bi t ñi m M nhìn ño n AB c ñ nh dư i m t
góc vuông (I lúc này là trung ñi m c a AB), và có th ph i thông qua m t vài khâu c t nghĩa v y u t ñ nh lư ng
thì ta m i có ñư c MI = R = const…
+) Ý tư ng c a Bài toán này xu t hi n r t nhi u trong các kì thi ð i H c các năm qua.
30
Bài t p áp d ng (Các em hãy d a vào ý tư ng Bài 3 ñ gi i các ví d sau)
Ví d 1 (A, A1 – 2012 – CB ): Cho hình vuông ABCD. G i M là trung ñi m c a c nh BC, N là ñi m trên c nh CD sao
cho CN = 2ND. Gi s
11 1
;
2 2
M
 
 
 
và ñư ng th ng AN có phương trình 2 3 0x y− − = . Tìm t a ñ ñi m A.
(ðs : (1; 1)A − ho c (4;5)A )
Ví d 2 (A – 2011 – CB ): Cho ñư ng th ng ∆: x + y + 2 = 0 và ñư ng tròn (C): 2 2
4 2 0x y x y+ − − = . G i I là tâm
c a (C), M là ñi m thu c ∆ . Qua M k các ti p tuy n MA và MB ñ n (C) (A và B là các ti p ñi m). Tìm t a ñ ñi m
M, bi t t giác MAIB có di n tích b ng 10. (ðs : (2; 4)M − ho c ( 3;1)M − )
Ví d 3 (A – 2010 – CB): Cho hai ñư ng th ng 1 : 3 0d x y+ = và 2 : 3 0d x y− = . G i (T) là ñư ng tròn ti p xúc
v i 1d t i A, c t 2d t i hai ñi m B và C sao cho tam giác ABC vuông t i B. Vi t phương trình c a (T), bi t tam giác
ABC có di n tích b ng
3
2
và ñi m A có hoành ñ dương. (ðs :
2 2
1 3
1
22 3
x y
   
+ + + =  
  
)
Ví d 4 (D – 2010 – CB): Cho tam giác ABC có ñ nh A(3; –7), tr c tâm là H(3; –1), tâm ñư ng tròn ngo i ti p là
I(–2; 0). Xác ñ nh t a ñ ñ nh C, bi t C có hoành ñ dương. (ðs : ( 2 65;3)C − + )
Ví d 5 (D – 2010 – NC): Cho ñi m A(0; 2) và ∆ là ñư ng th ng ñi qua O. G i H là hình chi u vuông góc c a A trên
∆ . Vi t phương trình ñư ng th ng ∆ , bi t kho ng cách t H ñ n tr c hoành b ng AH.
(ðs : ( 5 1) 2 5 2 0x y− − − = ho c ( 5 1) 2 5 2 0x y− + − = )
Ví d 6 (B – 2009 – CB ): Cho ñư ng tròn (C): (x – 2)2
+ y2
=
4
5
và hai ñư ng th ng ∆1: x – y = 0 và ∆2: x – 7y = 0.
Xác ñ nh to ñ tâm K và bán kính c a ñư ng tròn (C1); bi t ñư ng tròn (C1) ti p xúc v i các ñư ng th ng ∆1, ∆2 và
tâm K thu c ñư ng tròn (C). (ðs :
8 4
;
5 5
K
 
 
 
và bán kính
2 2
5
R = )
Ví d 7 (B – 2009 – NC): Cho tam giác ABC cân t i A có ñ nh A(–1;4) và các ñ nh B,C thu c ñư ng th ng
∆: x – y – 4 = 0. Xác ñ nh to ñ các ñi m B và C, bi t di n tích tam giác ABC b ng 18.
(ðs :
11 3 3 5
; , ;
2 2 2 2
B C
   
−   
   
ho c
3 5 11 3
; , ;
2 2 2 2
B C
   
−   
   
)
Ví d 8 (D – 2007): Cho ñư ng tròn 2 2
( ) : ( 1) ( 2) 9C x y− + + = và ñư ng th ng d: 3x – 4y + m = 0. Tìm m ñ trên d
có duy nh t m t ñi m P mà t ñó có th k ñư c hai ti p tuy n PA, PB t i (C) (A, B là các ti p ñi m) sao cho tam
giác PAB ñ u. (ðs : 19m = ho c 41m = − )
Ví d 9 (D – 2006): Cho ñư ng tròn 2 2
( ) : 2 2 1 0C x y x y+ − − + = và ñư ng th ng d: x – y + 3 = 0. Tìm t a ñ
ñi m M n m trên d sao cho ñư ng tròn tâm M, có bán kính g p ñôi bán kính ñư ng tròn (C), ti p xúc ngoài v i ñư ng
tròn (C). (ðs : (1;4)M ho c ( 2;1)M − )
Ví d 10 (B – 2005): Cho hai ñi m A(2;0) và B(6;4). Vi t phương trình ñư ng tròn (C) ti p xúc v i tr c hoành t i
ñi m A và kho ng cách t tâm c a (C) ñ n ñi m B b ng 5.
(ðs : 2 2
( 2) ( 1) 1x y− + − = ho c 2 2
( 2) ( 7) 49x y− + − = )
Ví d 11 (B – 2003): Cho tam giác ABC có AB = AC , BAC = 900
. Bi t M(1; -1) là trung ñi m c nh BC và G
2
;0
3
 
 
 
là tr ng tâm c a tam giác ABC. Tìm t a ñ các ñ nh A, B, C. (ðs : (0;2), (4;0), ( 2; 2)A B C − − )
Ví d 12 (B – 2002): Cho hình ch nh t ABCD có tâm I
1
;0
2
 
 
 
, phương trình ñư ng th ng AB là x – 2y + 2 = 0 và
AB = 2AD. Tìm t a ñ các ñi m A, B, C, D bi t r ng A có hoành ñ âm. (ðs : ( 2;0), (2;2), (3;0), ( 1; 2)A B C D− − − )
Ví d 13: Cho tam giác ABC có tr c tâm là H(–1; 4), tâm ñư ng tròn ngo i ti p là I(–3; 0) và trung ñi m c a c nh
BC là M(0; 3). Vi t phương trình ñư ng th ng AB, bi t B có hoành ñ dương. (ðs: 3x + 7y – 49 = 0)
31
Ví d 14: Cho ba ñi m I(1; 1), M(–2; 2) và N(2; –2). Tìm t a ñ các ñ nh c a hình vuông ABCD sao cho I là tâm c a
hình vuông, M thu c c nh AB, K thu c c nh CD và A có hoành ñ dương. (ðs: A(1; 5), B(–3; 1), C(1; –3), D(5; 1))
Ví d 15: Cho tam giác ABC có tr ng tâm là G
1 1
;
3 3
 
 
 
, tâm ñư ng tròn ngo i ti p là I
2 4
;
5 5
 
− 
 
và trung ñi m
c a c nh BC là M(–1; 2). Vi t phương trình ñư ng th ng AC, bi t B có hoành ñ âm. (ðs: 3x + y – 6 = 0)
Ví d 16: Cho ñư ng tròn ( C ) : 2 2
8 6 21 0x y x y+ − + + = và ñư ng th ng d : x + y – 1 = 0.Xác ñ nh t a ñ các
ñ nh c a hình vuông ABCD ngo i ti p (C) bi t A thu c d và hoành ñ c a ñi m B l n hơn hoành ñ c a ñi m D)
(ðs : (6;5), (6; 1), (2;1), (2; 5)A B C D− − ho c (2;1), (6; 1), (6;5), (2; 5)A B C D− − )
Bài 4: Qua ñi m M 0 0( ; )x y n m ngoài ñư ng tròn (C) có tâm I bán kính R.
1) Vi t phương trình ti p tuy n 1 2,MT MT ñ n ñư ng tròn.
2) Vi t phương trình ñư ng th ng ∆ ñi qua 1 2,T T .
3) Tính di n tích t giác 1 2MT IT .
Cách gi i:
Cách vi t t ng quát v phương trình ti p tuy n:
TH1: N u bi t ti p ñi m T ⇒ ti p tuy n ∆ c a (C) ñi qua T nh n IT
uur
làm vtpt⇒ phương trình ∆
TH2: N u không bi t ti p ñi m thì dùng ñi u ki n : ∆ là ti p tuy n c a (C) ( , )d I R⇔ ∆ =
1) Như v y v i bài toán này ta s làm theo TH2 :
+) G i ∆ ñi qua ñi m M 0 0( ; )x y có vtpt ( ; )n a b=
r
( 2 2
0a b+ ≠ ) có d ng:
0 0( ) ( ) 0a x x b y y− + − = hay 0 0 0ax by ax by+ − − = ( ∆ )
+) T (*) ch n
?
?
a
b
=
⇒
=
phương trình 1 2,∆ ∆ hay phương trình 1 2,MT MT
( m t trong hai phương trình (*) có th có: a = 0 ho c b = 0)
CHÚ Ý: Có th tìm c th t a ñ 1 2,T T nh gi i h :
( )C
∆


2) +) G i 0 0( ; )T x y là ti p ñi m c a ti p tuy n k t M ñ n (C)
( )
. 0
T C
MT IT
∈
⇒ 
=
uuur uur (*)
3) 1 2 1 1 1 1
1
2 2. . .
2
MT IT MT IS S MT IT MT R= = = v i 2 2
1MT MI R= −
32
Bài t p áp d ng (Các em hãy d a vào ý tư ng Bài 4 ñ gi i các ví d sau)
Ví d 1(B – 2006): Cho ñư ng tròn: 2 2
( ) : 2 6 6 0C x y x y+ − − + = và ñi m M(– 3; 1). G i 1T và 2T là các ti p ñi m
c a các ti p tuy n k t M ñ n (C). Vi t phương trình ñư ng th ng 1T 2T . (ðs: 2 3 0x y+ − = )
Ví d 2: (A – 2011 – CB): Cho ñư ng th ng ∆: x + y + 2 = 0 và ñư ng tròn (C): 2 2
4 2 0x y x y+ − − = . G i I là tâm
c a (C), M là ñi m thu c ∆ . Qua M k các ti p tuy n MA và MB ñ n (C) (A và B là các ti p ñi m). Tìm t a ñ ñi m
M, bi t t giác MAIB có di n tích b ng 10. (ðs : (2; 4)M − ho c ( 3;1)M − )
Bài 5:Cho ñư ng th ng ∆ , ñư ng tròn (C) có tâm I và hai ñi m ,M N n m ngoài ñư ng tròn.
1) Tìm ñi u ki n ñ ∆ c t (C) t i hai ñi m phân bi t A, B sao cho di n tích tam giác IAB l n nh t.
2) Tìm K thu c (C) sao cho di n tích tam giác KMN l n nh t, nh nh t.
3) Tìm P thu c ∆ sao cho qua P k hai ti p tuy n 1 2,PT PT sao cho di n tích tam giác 1 2ITT l n nh t.
TH1 TH2 TH3
Cách gi i :
Bài t p áp d ng (Các em hãy d a vào ý tư ng Bài 5 ñ gi i các ví d sau)
Ví d 1 : Cho ñư ng tròn 2 2
( ) : 2 3 0C x x y− + − = . G i B, C là giao ñi m c a ñư ng th ng : 3 0x y∆ + − = . Hãy
tìm các ñi m A trên ñư ng tròn (C) sao cho tam giác ABC có chu vi l n nh t. (ðs : (1 2; 2)A − − )
Ví d 2 : Cho ñư ng tròn 2 2
( ) : 4 6 12 0C x y x y+ − − + = có tâm I và ñư ng th ng : 4 0x y∆ + − = . Tìm trên
ñư ng th ng ∆ ñi m M sao cho ti p tuy n k t M ti p xúc v i (C) t i A, B mà tam giác IAB có di n tích l n nh t.
(ðs :
3 3 5 3 3 3 5 3
( ; ), ( ; )
2 2 2 2
M M
+ − − +
)
Ví d 3 : Cho ñư ng tròn ngo i ti p tam giác ABC v i A(2 ; - 2), B(4 ; 0), C(3 ; 2 1− ) và ñư ng th ng
33
∆ : 4 4 0x y+ − = . Tìm trên ñư ng th ng ∆ ñi m M sao cho ti p tuy n c a (C) qua M ti p xúc v i C t i N sao cho
di n tích tam giác NAB l n nh t. (ðs :
6 4
(2; 4), ( ; )
5 5
M M− − )
Bài 6: Vi t phương trình ∆ qua 0 0( ; )M x y c t ñư ng tròn (C) có tâm I, bán kính R t i A, B sao cho MA kMB= .
Cách gi i :
C1 : +) ð t IH h=
2 2
2 2
MH IM h
HA HB R h
 = −
→ 
= = −
(*)
CHÚ Ý:
+) Cách gi i trên th y s d ng trư ng h p 1k > ( v i 1k < các em làm tương t )
+) Cách gi i trên th y s d ng 0 0( ; )M x y n m ngoài ñư ng tròn (C) ( 0 0( ; )M x y n m trong (C) các em làm tương t )
C2 :
+) Xét phương trình ∆ qua 0 0( ; )M x y có h s góc k có d ng : 0 0( )y k x x y= − +
+) Xác ñ nh phương trình hoành ñ giao ñi m c a ∆ và (C) : 2
( , , ) 0f x x k = (*)
+) Dùng vi – et cho (*) và k t h p MA kMB= ?k⇒ = ⇒phương trình ∆ .
Bài t p áp d ng (Các em hãy d a vào ý tư ng Bài 6 ñ gi i các ví d sau)
Ví d 1 : Cho ñư ng tròn (C): 2 2
2 2 23 0x y x y+ − + − = , ñi m M(7; 3). Vi t phương trình ñư ng th ng ∆ qua M
c t ñư ng tròn (C) t i A, B sao cho MA = 3MB. ( ðs : 3y = ho c 12 5 69 0x y− − = )
Ví d 2 : Cho ñi m A(-1 ; 14) và ñư ng tròn (C) tâm I(1 ; -5), bán kính b ng 13. Vi t phương trình ñư ng th ng ∆ ñi
qua A c t (C) t i M, N mà kho ng cách t M ñ n AI b ng m t n a kho ng cách t N ñ n AI.
(ðs : x + y – 13 = 0 và 433x – 281y +4367 = 0)
Lo i 2.2: S tương giao gi a hai ñương tròn
34
TH1: 'R r II+ > TH2: 'R r II+ = TH3: 'R r II+ < TH4: 'R r II− =
Ngoài nhau Ti p xúc ngoài C t nhau t i hai ñi m Ti p xúc trong
CHÚ Ý: Còn trư ng h p ñ ng nhau. Nhưng trư ng h p này ít ñư c khai thác nên th y không ñ c p ñây.
Bài t p áp d ng
Ví d 1(D – 2009 – NC): Cho ñư ng tròn 2 2
( ) :( 1) 1C x y− + = . G i I là tâm c a (C). Xác ñ nh t a ñ ñi m M thu c
(C) sao cho 0
30IOM∠ = . (ðs:
3 3
;
2 2
M
 
  
 
ho c
3 3
;
2 2
M
 
−  
 
)
Ví d 2(D – 2003): Cho ñư ng tròn (C) : (x – 1)2
+ (y – 2)2
= 4 và ñư ng th ng d : x – y – 1 = 0.Vi t phương trình
ñư ng tròn (C’) ñ i x ng v i ñư ng tròn (C) qua ñư ng th ng d. Tìm t a ñ các giao ñi m c a (C) và (C’).
(ðs: 2 2
( 3) 4x y− + = (1;0), (3;2)A B )
Ví d 3 (D – 2006): Cho ñư ng tròn 2 2
( ) : 2 2 1 0C x y x y+ − − + = và ñư ng th ng d: x – y + 3 = 0. Tìm t a ñ
ñi m M n m trên d sao cho ñư ng tròn tâm M, có bán kính g p ñôi bán kính ñư ng tròn (C), ti p xúc ngoài v i ñư ng
tròn (C). (ðs: (1;4)M ho c ( 2;1)M − )
Ví d 4: Cho ñư ng tròn 2 2
1( ) : 6 4 7 0C x y x y+ − + − = c t ñư ng tròn 2 2
2( ) :( 6) ( 1) 50C x y+ + − = t i hai ñi m
M, N bi t M có hoành ñ dương. Vi t phương trình ñư ng th ng ∆ ñi qua M l n lư t c t 1 2( ),( )C C t i các ñi m th
hai A, B sao cho M là trung ñi m c a AB. (ðs: 5x – 7y + 9 = 0)
Ví d 5: Cho tam giác ABC n i ti p ñư ng tròn tâm I(6; 6) và ngo i ti p ñư ng tròn tâm K(4; 5), biêt ñ nh A(2; 3).
Vi t phương trình c nh BC. (ðs: 3x + 4y – 42 = 0)
Ví d 6: Cho ñư ng tròn (C) : 2 2
1x y+ = .ðư ng tròn ( C’) tâm I(2;2) c t (C) t i 2 ñi m A,B sao cho 2AB = .
Vi t phương trình ñư ng th ng AB. ( ðs: 1 0x y+ + = ho c 1 0x y+ − = )
D ng 4: Các bài toán v Elip
Lo i 1: Vi t phương trình Elip và xác ñ nh các y u t c a Elip
Cách gi i chung:
+) Gi s phương trình chính t c c a elip có d ng:
2 2
2 2
1
x y
a b
+ = ( )E
Bài t p áp d ng
Ví d 1: L p phương trình chính t c c a elip (E) bi t:
1) Có ñ dài hai tr c là 6, 4.
2) Có m t ñ nh là (5; 0) và tiêu c là 6.
3) Có m t ñ nh là (0; 3) và ñi qua ñi m M(4; 1).
35
4) ði qua hai ñi m
3
1;
2
 
  
 
và
2
2;
2
 
−  
 
.
5) Có tiêu ñi m 2 (2;0)F và qua ñi m
5
2;
3
 
 
 
.
6) Có tiêu ñi m 2 (5;0)F và kho ng cách gi a hai ñ nh là 9.
7) Tiêu c là 4 và kho ng cách t m t ñ nh trên tr c nh ñ n tiêu ñi m b ng 5.
( ðs: 1)
2 2
1
9 4
x y
+ = 2)
2 2
1
25 16
x y
+ = 3)
2 2
1
18 9
x y
+ = 4)
2 2
1
4 1
x y
+ = 5)
2 2
1
9 5
x y
+ =
6)
2 2
1
181 81
4 4
x y
+ = 7)
2 2
1
25 21
x y
+ = ho c
2 2
1
49 45
x y
+ = ho c
2 2
1
9 5
x y
+ = )
Ví d 2(A – 2008): Trong m t ph ng v i h t a ñ Oxy, hãy vi t phương trình chính t c c a elip (E) bi t r ng (E) có
tâm sai b ng
5
3
và hình ch nh t cơ s c a (E) có chu vi b ng 20. (ðs:
2 2
1
9 4
x y
+ = )
Ví d 3(B – 2012 – NC): Cho hình thoi ABCD có AC = 2BD và ñư ng tròn ti p xúc v i các c nh c a hình thoi có
phương trình 2 2
4x y+ = . Vi t phương trình chính t c c a elip (E) ñi qua các ñ nh A, B, C, D c a hình thoi. Bi t A
thu c Ox. (ðs:
2 2
1
20 5
x y
+ = )
Ví d 3: Trong m t ph ng Oxy, cho elip (E) có ñ dài tr c l n b ng 4 2 ,các ñ nh n m trên tr c nh và các tiêu
ñi m c a (E) cùng n m trên m t ñư ng tròn. L p phương trình chính t c c a (E).
Ví d 4: Cho elip (E) có ñ dài tr c l n b ng 6, tâm sai b ng m t ph n hai và kho ng cách t m t ñi m M c a (E) ñ n
tiêu ñi m 1F (có hoành ñ âm) b ng 7.
1) Tìm kho ng cách t M ñ n tiêu ñi m 2F 2) Vi t phương trình chính t c c a elip (E) và tìm t a ñ ñi m M.
Lo i 2: Tìm ñi m thu c Elip
+) T (1) và (2)
0
0
?
?
x
y
=
⇒ 
=
M⇒
CHÚ Ý : N u 0 0( ; ) ( )M x y E∈ ta có th khai thác thêm d ki n:
1 0
2 0
c
MF a x
a
c
MF a x
a

= +

 = −

Bài t p áp d ng
Ví d 1: Cho elip (E):
2 2
1
6 2
x y
+ =
1) Tìm t a ñ giao ñi m c a (E) và ñư ng th ng 3 2y x= − . 2)Tìm trên (E) ñi m M sao cho góc 0
1 2 90F MF∠ =
3) Tìm trên (E) ñi m N sao cho 1 2 6F N F N− = .
1)
3 7
( 3;1), ;
5 5
A B
 
−  
 
2) 1 2 3 4( 3;1), ( 3; 1), ( 3;1), ( 3; 1)M M M M− − − − 3)
3 5
;
2 4
N
 
  
 
ho c
3 5
;
2 4
N
 
−  
 
36
Ví d 2: Cho (E):
2 2
2 2
1
x y
a b
+ = có tiêu ñi m 1 2,F F
1) Cho a = 2, b = 1. Tìm ñi m M sao cho 1 22F M F M= . (ðs:
4 23
;
273 3
M
 
  
 
ho c
4 23
;
273 3
M
 
−  
 
)
2) Ch ng minh r ng v i m i ñi m M ta luôn có: 2 2 2
1 2.F M F M OM a b+ = +
Ví d 3(D – 2005): Cho ñi m C(2;0) và elíp (E):
2 2
1
4 1
x y
+ = . Tìm to ñ các ñi m A, B thu c (E), bi t r ng hai ñi m
A, B ñ i x ng v i nhau qua tr c hoành và tam giác ABC là tam giác ñ u.
(ðs:
2 4 3 2 4 3
; , ;
7 7 7 7
A B
   
−      
   
ho c
2 4 3 2 4 3
; , ;
7 7 7 7
A B
   
−      
   
)
Ví d 4 (A – 2011 – NC) : Cho elip (E) :
2 2
1
4 1
x y
+ = . Tìm ñi m A và B thu c (E), có hoành ñ dương sao cho tam
giác OAB cân t i O và có di n tích l n nh t. (ðs:
2 2
2; , 2;
2 2
A B
   
−      
   
or
2 2
2; , 2;
2 2
A B
   
−      
   
)
Ví d 5: Trong m t ph ng t a ñ Oxy, cho elip (E) : 2 2
9 25 225x y+ = . Tìm t a ñ ñi m M thu c (E) sao cho tam
giác M 1 2F F vuông t i M.
Ví d 6: Cho elip (E) : 2 2
5 9 45x y+ = có tiêu ñi m 1 2,F F . M là m t ñi m b t kì trên (E) và bi u th c
1 2
1 2
1 1
f F M F M
F M F M
= + + +
1) Ch ng minh chu vi tam giác 1 2F MF không ñ i. Tìm M ñ di n tích tam giác 1 2F MF b ng 2.
2) Tìm M sao cho giá tr c a f l n nh t.
Ví d 7: Cho ñi m M di ñ ng elip: 2 2
9 16 144x y+ = và H, K l n lư t là hình chi u c a M lên hai tr c t a ñ . Tìm M
ñ di n tích OHMK l n nh t.
Lo i 3: S tương giao gi a ñư ng th ng và Elip
Cách gi i chung : S tương giao gi a ñư ng th ng : 0Ax By C∆ + + = và (E):
2 2
2 2
1
x y
a b
+ =
+) Gi i h 2 2
2 2
0
1
Ax By C
x y
a b
+ + =


+ =

(I) b ng phương pháp th
( ði u ki n ñ ∆ là ti p tuy n c a (E) : 2 2 2 2 2
A a B b C+ = (ñư c sinh ra t (II) )).
Bài t p áp d ng
Ví d 1: Cho elip (E): 2 2
4 9 36x y+ = và ñi m M(1; 1). L p phương trình ñư ng th ng qua M và c t (E) t i hai ñi m
1 2,M M sao cho 1 2MM MM= . (ðs: 4x + 9y – 13 = 0)
Ví d 2:Cho hai ñi m A ( 3;0)− , B ( 3;0)và ñư ng th ng d: 3 2( 3 1) 3 0x y− − + = . Tìm trên d ñi m M có
hoành ñ âm sao cho chu vi tam giác MAB b ng 4 2 3+ . (ðs:
3
1;
2
M
 
−  
 
)
37
Ví d 3 (D – 2002): Cho elip (E) có phương trình
2 2
1
16 9
x y
+ = . Xét ñi m M chuy n ñ ng trên tia Ox và ñi m N
chuy n ñ ng trên tia Oy sao cho ñư ng th ng MN luôn ti p xúc v i (E). Xác ñ nh t a ñ ñi m M, N ñ ño n MN có
ñ dài nh nh t. Tính giá tr nh nh t ñó. (ðs: (2 7;0), (0; 21)M N và GTNN c a MN b ng 7)
Ví d 4 (B – 2010 – NC): Cho ñi m A(2; 3 ) và (E):
2 2
1
3 2
x y
+ = . G i 1F và 2F là các tiêu ñi m c a (E) ( 1F có
hoành ñ âm); M là giao ñi m có tung ñ dương c a ñương th ng 1AF v i (E); N là ñi m ñ i x ng c a 2F qua M. Vi t
phương trình ñư ng tròn ngo i ti p tam giác 2ANF . (ðs:
2
2 2 3 4
( 1)
3 3
x y
 
− + − =  
 
)
Ví d 5: Cho Elip (E) :
2 2
1
64 9
x y
+ = .Vi t phương trình ti p tuy n d c a (E) bi t d c t 2 tr c t a ñ Ox,Oy l n lư t t i
A,B sao cho AO = 2BO.
CHÚ Ý:
Khi trong bài toán v ñư ng tròn và Elip có y u t min, max chúng ta hay s d ng b t ñ ng th c Cauchy và
Bunhiacopxki (2011A – NC, 2002D…)
C m ơn các em và các b n ñã ñ c t i li u !
M i ý ki n ñóng góp các em và các b n g i qua E- mail: giaidaptoancap3@yahoo.com
ho c ñ a ch : s 9 – Ngõ 880 – B ch ð ng – Hai Bà Trưng – Hà N i
ði n tho i : 043.9871450 ho c Dð: 0947141139
L i gi i các bài t p các em có th tham kh o trên web: http://www.facebook.com/giaidaptoancap3

Mais conteúdo relacionado

Mais procurados

[Nguoithay.org ] tong hop bai giang ve duong thang trong oxy
[Nguoithay.org ] tong hop bai giang ve duong thang trong oxy[Nguoithay.org ] tong hop bai giang ve duong thang trong oxy
[Nguoithay.org ] tong hop bai giang ve duong thang trong oxyDuc Tam
 
GIẢI TAM GIÁC TRONG TỌA ĐỘ PHẲNG
GIẢI TAM GIÁC TRONG TỌA ĐỘ PHẲNGGIẢI TAM GIÁC TRONG TỌA ĐỘ PHẲNG
GIẢI TAM GIÁC TRONG TỌA ĐỘ PHẲNGDANAMATH
 
Cac dang-bai-tap-phuong-trinh-duong-thang-bt-phuong-trinh-duong-thang (2)
Cac dang-bai-tap-phuong-trinh-duong-thang-bt-phuong-trinh-duong-thang (2)Cac dang-bai-tap-phuong-trinh-duong-thang-bt-phuong-trinh-duong-thang (2)
Cac dang-bai-tap-phuong-trinh-duong-thang-bt-phuong-trinh-duong-thang (2)Ty Luong
 
200 Bài toán hình học tọa độ phẳng
200 Bài toán hình học tọa độ phẳng200 Bài toán hình học tọa độ phẳng
200 Bài toán hình học tọa độ phẳngtuituhoc
 
CHUYÊN ĐỀ :TỌA ĐỘ PHẲNG - PHƯƠNG PHÁP VECTƠ
CHUYÊN ĐỀ :TỌA ĐỘ PHẲNG - PHƯƠNG PHÁP VECTƠCHUYÊN ĐỀ :TỌA ĐỘ PHẲNG - PHƯƠNG PHÁP VECTƠ
CHUYÊN ĐỀ :TỌA ĐỘ PHẲNG - PHƯƠNG PHÁP VECTƠDANAMATH
 
Toa do trong mặt phẳng
Toa do trong mặt phẳngToa do trong mặt phẳng
Toa do trong mặt phẳngZenDi ZenDi
 
Tổng hợp các dạng toán về phương trình đường thẳng trong các đề thi (có lời ...
 Tổng hợp các dạng toán về phương trình đường thẳng trong các đề thi (có lời ... Tổng hợp các dạng toán về phương trình đường thẳng trong các đề thi (có lời ...
Tổng hợp các dạng toán về phương trình đường thẳng trong các đề thi (có lời ...Thùy Linh
 
200 bai tap hinh hoc toa do phang tran si tung (2)
200 bai tap hinh hoc toa do phang   tran si tung (2)200 bai tap hinh hoc toa do phang   tran si tung (2)
200 bai tap hinh hoc toa do phang tran si tung (2)Song Tử Mắt Nâu
 
Đường thẳng đường tròn Oxy Mathvn
Đường thẳng đường tròn Oxy MathvnĐường thẳng đường tròn Oxy Mathvn
Đường thẳng đường tròn Oxy MathvnMinh Thắng Trần
 
Toan pt.de026.2011
Toan pt.de026.2011Toan pt.de026.2011
Toan pt.de026.2011BẢO Hí
 
[Vnmath.com] de thi quoc gia lan 1 thpt hau loc 2
[Vnmath.com] de thi quoc gia lan 1 thpt hau loc 2[Vnmath.com] de thi quoc gia lan 1 thpt hau loc 2
[Vnmath.com] de thi quoc gia lan 1 thpt hau loc 2Marco Reus Le
 
692 bai hinh ltdh 17 quang trung
692 bai hinh ltdh  17 quang trung692 bai hinh ltdh  17 quang trung
692 bai hinh ltdh 17 quang trungndphuc910
 
03 bai toan giai tam giac p1_bg
03 bai toan giai tam giac p1_bg03 bai toan giai tam giac p1_bg
03 bai toan giai tam giac p1_bgNgoc Diep Ngocdiep
 
Ôn Thi Đại Học 2015 (Tọa độ mặt phẳng)
Ôn Thi Đại Học 2015 (Tọa độ mặt phẳng)Ôn Thi Đại Học 2015 (Tọa độ mặt phẳng)
Ôn Thi Đại Học 2015 (Tọa độ mặt phẳng)Phạm Lộc
 
25 Đề Thi thử quốc gia năm 2015 môn Toán Hay
25 Đề Thi thử quốc gia năm 2015 môn Toán Hay25 Đề Thi thử quốc gia năm 2015 môn Toán Hay
25 Đề Thi thử quốc gia năm 2015 môn Toán HayZaj Bé Đẹp
 
Giasudhsphn.com.baitap hh10
Giasudhsphn.com.baitap hh10Giasudhsphn.com.baitap hh10
Giasudhsphn.com.baitap hh10Nhập Vân Long
 

Mais procurados (18)

[Nguoithay.org ] tong hop bai giang ve duong thang trong oxy
[Nguoithay.org ] tong hop bai giang ve duong thang trong oxy[Nguoithay.org ] tong hop bai giang ve duong thang trong oxy
[Nguoithay.org ] tong hop bai giang ve duong thang trong oxy
 
GIẢI TAM GIÁC TRONG TỌA ĐỘ PHẲNG
GIẢI TAM GIÁC TRONG TỌA ĐỘ PHẲNGGIẢI TAM GIÁC TRONG TỌA ĐỘ PHẲNG
GIẢI TAM GIÁC TRONG TỌA ĐỘ PHẲNG
 
Cac dang-bai-tap-phuong-trinh-duong-thang-bt-phuong-trinh-duong-thang (2)
Cac dang-bai-tap-phuong-trinh-duong-thang-bt-phuong-trinh-duong-thang (2)Cac dang-bai-tap-phuong-trinh-duong-thang-bt-phuong-trinh-duong-thang (2)
Cac dang-bai-tap-phuong-trinh-duong-thang-bt-phuong-trinh-duong-thang (2)
 
200 Bài toán hình học tọa độ phẳng
200 Bài toán hình học tọa độ phẳng200 Bài toán hình học tọa độ phẳng
200 Bài toán hình học tọa độ phẳng
 
CHUYÊN ĐỀ :TỌA ĐỘ PHẲNG - PHƯƠNG PHÁP VECTƠ
CHUYÊN ĐỀ :TỌA ĐỘ PHẲNG - PHƯƠNG PHÁP VECTƠCHUYÊN ĐỀ :TỌA ĐỘ PHẲNG - PHƯƠNG PHÁP VECTƠ
CHUYÊN ĐỀ :TỌA ĐỘ PHẲNG - PHƯƠNG PHÁP VECTƠ
 
Toa do trong mặt phẳng
Toa do trong mặt phẳngToa do trong mặt phẳng
Toa do trong mặt phẳng
 
Hinh chuong3
Hinh chuong3Hinh chuong3
Hinh chuong3
 
Tổng hợp các dạng toán về phương trình đường thẳng trong các đề thi (có lời ...
 Tổng hợp các dạng toán về phương trình đường thẳng trong các đề thi (có lời ... Tổng hợp các dạng toán về phương trình đường thẳng trong các đề thi (có lời ...
Tổng hợp các dạng toán về phương trình đường thẳng trong các đề thi (có lời ...
 
200 bai tap hinh hoc toa do phang tran si tung (2)
200 bai tap hinh hoc toa do phang   tran si tung (2)200 bai tap hinh hoc toa do phang   tran si tung (2)
200 bai tap hinh hoc toa do phang tran si tung (2)
 
Tuyển tập hình học giải tích trong mặt phẳng
Tuyển tập hình học giải tích trong mặt phẳngTuyển tập hình học giải tích trong mặt phẳng
Tuyển tập hình học giải tích trong mặt phẳng
 
Đường thẳng đường tròn Oxy Mathvn
Đường thẳng đường tròn Oxy MathvnĐường thẳng đường tròn Oxy Mathvn
Đường thẳng đường tròn Oxy Mathvn
 
Toan pt.de026.2011
Toan pt.de026.2011Toan pt.de026.2011
Toan pt.de026.2011
 
[Vnmath.com] de thi quoc gia lan 1 thpt hau loc 2
[Vnmath.com] de thi quoc gia lan 1 thpt hau loc 2[Vnmath.com] de thi quoc gia lan 1 thpt hau loc 2
[Vnmath.com] de thi quoc gia lan 1 thpt hau loc 2
 
692 bai hinh ltdh 17 quang trung
692 bai hinh ltdh  17 quang trung692 bai hinh ltdh  17 quang trung
692 bai hinh ltdh 17 quang trung
 
03 bai toan giai tam giac p1_bg
03 bai toan giai tam giac p1_bg03 bai toan giai tam giac p1_bg
03 bai toan giai tam giac p1_bg
 
Ôn Thi Đại Học 2015 (Tọa độ mặt phẳng)
Ôn Thi Đại Học 2015 (Tọa độ mặt phẳng)Ôn Thi Đại Học 2015 (Tọa độ mặt phẳng)
Ôn Thi Đại Học 2015 (Tọa độ mặt phẳng)
 
25 Đề Thi thử quốc gia năm 2015 môn Toán Hay
25 Đề Thi thử quốc gia năm 2015 môn Toán Hay25 Đề Thi thử quốc gia năm 2015 môn Toán Hay
25 Đề Thi thử quốc gia năm 2015 môn Toán Hay
 
Giasudhsphn.com.baitap hh10
Giasudhsphn.com.baitap hh10Giasudhsphn.com.baitap hh10
Giasudhsphn.com.baitap hh10
 

Destaque

Голубятников Антон
Голубятников АнтонГолубятников Антон
Голубятников Антонweb2win
 
Acme Stadium View
Acme Stadium ViewAcme Stadium View
Acme Stadium ViewWallsnroof1
 
Pp 47 tahun 2015 tentang desa
Pp 47 tahun 2015 tentang desaPp 47 tahun 2015 tentang desa
Pp 47 tahun 2015 tentang desaAinur Rofiq
 
заседание метод
заседание методзаседание метод
заседание методschool35
 
Het gebruik van gestandaardiseerde vragenlijsten in de huisartsenpraktijk voo...
Het gebruik van gestandaardiseerde vragenlijsten in de huisartsenpraktijk voo...Het gebruik van gestandaardiseerde vragenlijsten in de huisartsenpraktijk voo...
Het gebruik van gestandaardiseerde vragenlijsten in de huisartsenpraktijk voo...Stefanie_VM
 
Updated course outline-2.pdf
Updated course outline-2.pdfUpdated course outline-2.pdf
Updated course outline-2.pdfrachaelcheong
 
BULK IEEE PROJECTS IN POWER ELECTRONICS
BULK IEEE PROJECTS IN POWER ELECTRONICSBULK IEEE PROJECTS IN POWER ELECTRONICS
BULK IEEE PROJECTS IN POWER ELECTRONICSNexgen Technology
 

Destaque (14)

Голубятников Антон
Голубятников АнтонГолубятников Антон
Голубятников Антон
 
Acme Stadium View
Acme Stadium ViewAcme Stadium View
Acme Stadium View
 
Help
HelpHelp
Help
 
Fendi
FendiFendi
Fendi
 
Mayfair Akshay
Mayfair Akshay Mayfair Akshay
Mayfair Akshay
 
UNIQUE EMPORIA
UNIQUE EMPORIAUNIQUE EMPORIA
UNIQUE EMPORIA
 
Pp 47 tahun 2015 tentang desa
Pp 47 tahun 2015 tentang desaPp 47 tahun 2015 tentang desa
Pp 47 tahun 2015 tentang desa
 
robo16
robo16robo16
robo16
 
заседание метод
заседание методзаседание метод
заседание метод
 
Het gebruik van gestandaardiseerde vragenlijsten in de huisartsenpraktijk voo...
Het gebruik van gestandaardiseerde vragenlijsten in de huisartsenpraktijk voo...Het gebruik van gestandaardiseerde vragenlijsten in de huisartsenpraktijk voo...
Het gebruik van gestandaardiseerde vragenlijsten in de huisartsenpraktijk voo...
 
Updated course outline-2.pdf
Updated course outline-2.pdfUpdated course outline-2.pdf
Updated course outline-2.pdf
 
1
11
1
 
Ergonomía
ErgonomíaErgonomía
Ergonomía
 
BULK IEEE PROJECTS IN POWER ELECTRONICS
BULK IEEE PROJECTS IN POWER ELECTRONICSBULK IEEE PROJECTS IN POWER ELECTRONICS
BULK IEEE PROJECTS IN POWER ELECTRONICS
 

Semelhante a Các hướng tư duy và phương pháp giải trong hình oxy

Toan pt.de039.2012
Toan pt.de039.2012Toan pt.de039.2012
Toan pt.de039.2012BẢO Hí
 
Cac huongtuduy phuongphapgiai_hinhhocoxyz
Cac huongtuduy phuongphapgiai_hinhhocoxyzCac huongtuduy phuongphapgiai_hinhhocoxyz
Cac huongtuduy phuongphapgiai_hinhhocoxyzHuynh ICT
 
Toan pt.de078.2010
Toan pt.de078.2010Toan pt.de078.2010
Toan pt.de078.2010BẢO Hí
 
Toan pt.de017.2011
Toan pt.de017.2011Toan pt.de017.2011
Toan pt.de017.2011BẢO Hí
 
Toan pt.de001.2012
Toan pt.de001.2012Toan pt.de001.2012
Toan pt.de001.2012BẢO Hí
 
chuong 1 hinh hoc 11 - phep doi hinh dong dang bien soan cong phu - hay nhat ...
chuong 1 hinh hoc 11 - phep doi hinh dong dang bien soan cong phu - hay nhat ...chuong 1 hinh hoc 11 - phep doi hinh dong dang bien soan cong phu - hay nhat ...
chuong 1 hinh hoc 11 - phep doi hinh dong dang bien soan cong phu - hay nhat ...Hoàng Thái Việt
 
Dap an de thi dai hocmon toan khoi a a1 v nam 2013
Dap an de thi dai hocmon toan khoi a a1 v nam 2013Dap an de thi dai hocmon toan khoi a a1 v nam 2013
Dap an de thi dai hocmon toan khoi a a1 v nam 2013Hương Lan Hoàng
 
Bai toan tim diem tren do thi ham so
Bai toan tim diem tren do thi ham soBai toan tim diem tren do thi ham so
Bai toan tim diem tren do thi ham soVui Lên Bạn Nhé
 
Thi thử toán THPT Chu Văn An TN lần 2 2014
Thi thử toán THPT Chu Văn An TN lần 2 2014Thi thử toán THPT Chu Văn An TN lần 2 2014
Thi thử toán THPT Chu Văn An TN lần 2 2014dlinh123
 
Toan pt.de024.2010
Toan pt.de024.2010Toan pt.de024.2010
Toan pt.de024.2010BẢO Hí
 
De thi thu dh thpt nam sach hai duong
De thi thu dh thpt nam sach hai duongDe thi thu dh thpt nam sach hai duong
De thi thu dh thpt nam sach hai duongVui Lên Bạn Nhé
 
Toan pt.de094.2011
Toan pt.de094.2011Toan pt.de094.2011
Toan pt.de094.2011BẢO Hí
 
Toan pt.de015.2011
Toan pt.de015.2011Toan pt.de015.2011
Toan pt.de015.2011BẢO Hí
 
Toan pt.de101.2011
Toan pt.de101.2011Toan pt.de101.2011
Toan pt.de101.2011BẢO Hí
 
Toan pt.de038.2012
Toan pt.de038.2012Toan pt.de038.2012
Toan pt.de038.2012BẢO Hí
 
Tai lieu luyen thi dai hoc de thi dh toan khoi b - nam 2013
Tai lieu luyen thi dai hoc   de thi dh toan khoi b - nam 2013Tai lieu luyen thi dai hoc   de thi dh toan khoi b - nam 2013
Tai lieu luyen thi dai hoc de thi dh toan khoi b - nam 2013Trungtâmluyệnthi Qsc
 
3 đề thi thử toán 2015 + đáp án (Bình Thuận)
3 đề thi thử toán 2015 + đáp án (Bình Thuận)3 đề thi thử toán 2015 + đáp án (Bình Thuận)
3 đề thi thử toán 2015 + đáp án (Bình Thuận)Vui Lên Bạn Nhé
 

Semelhante a Các hướng tư duy và phương pháp giải trong hình oxy (20)

Toan pt.de039.2012
Toan pt.de039.2012Toan pt.de039.2012
Toan pt.de039.2012
 
Cac huongtuduy phuongphapgiai_hinhhocoxyz
Cac huongtuduy phuongphapgiai_hinhhocoxyzCac huongtuduy phuongphapgiai_hinhhocoxyz
Cac huongtuduy phuongphapgiai_hinhhocoxyz
 
Toan pt.de078.2010
Toan pt.de078.2010Toan pt.de078.2010
Toan pt.de078.2010
 
Toan pt.de017.2011
Toan pt.de017.2011Toan pt.de017.2011
Toan pt.de017.2011
 
Toan pt.de001.2012
Toan pt.de001.2012Toan pt.de001.2012
Toan pt.de001.2012
 
chuong 1 hinh hoc 11 - phep doi hinh dong dang bien soan cong phu - hay nhat ...
chuong 1 hinh hoc 11 - phep doi hinh dong dang bien soan cong phu - hay nhat ...chuong 1 hinh hoc 11 - phep doi hinh dong dang bien soan cong phu - hay nhat ...
chuong 1 hinh hoc 11 - phep doi hinh dong dang bien soan cong phu - hay nhat ...
 
Dap an de thi dai hocmon toan khoi a a1 v nam 2013
Dap an de thi dai hocmon toan khoi a a1 v nam 2013Dap an de thi dai hocmon toan khoi a a1 v nam 2013
Dap an de thi dai hocmon toan khoi a a1 v nam 2013
 
Bai toan tim diem tren do thi ham so
Bai toan tim diem tren do thi ham soBai toan tim diem tren do thi ham so
Bai toan tim diem tren do thi ham so
 
Thi thử toán THPT Chu Văn An TN lần 2 2014
Thi thử toán THPT Chu Văn An TN lần 2 2014Thi thử toán THPT Chu Văn An TN lần 2 2014
Thi thử toán THPT Chu Văn An TN lần 2 2014
 
Toan pt.de024.2010
Toan pt.de024.2010Toan pt.de024.2010
Toan pt.de024.2010
 
De thi thu dh thpt nam sach hai duong
De thi thu dh thpt nam sach hai duongDe thi thu dh thpt nam sach hai duong
De thi thu dh thpt nam sach hai duong
 
Toan pt.de094.2011
Toan pt.de094.2011Toan pt.de094.2011
Toan pt.de094.2011
 
Toan pt.de015.2011
Toan pt.de015.2011Toan pt.de015.2011
Toan pt.de015.2011
 
Giao an day them toan 9
Giao an day them toan 9Giao an day them toan 9
Giao an day them toan 9
 
Hình oxy
Hình oxyHình oxy
Hình oxy
 
Toan pt.de101.2011
Toan pt.de101.2011Toan pt.de101.2011
Toan pt.de101.2011
 
Toan pt.de038.2012
Toan pt.de038.2012Toan pt.de038.2012
Toan pt.de038.2012
 
Tai lieu luyen thi dai hoc de thi dh toan khoi b - nam 2013
Tai lieu luyen thi dai hoc   de thi dh toan khoi b - nam 2013Tai lieu luyen thi dai hoc   de thi dh toan khoi b - nam 2013
Tai lieu luyen thi dai hoc de thi dh toan khoi b - nam 2013
 
3 đề thi thử toán 2015 + đáp án (Bình Thuận)
3 đề thi thử toán 2015 + đáp án (Bình Thuận)3 đề thi thử toán 2015 + đáp án (Bình Thuận)
3 đề thi thử toán 2015 + đáp án (Bình Thuận)
 
De thi thu dai hoc so 88
De thi thu dai hoc so 88De thi thu dai hoc so 88
De thi thu dai hoc so 88
 

Último

TỔNG HỢP ĐỀ THI CHÍNH THỨC KỲ THI TUYỂN SINH VÀO LỚP 10 THPT MÔN NGỮ VĂN NĂM ...
TỔNG HỢP ĐỀ THI CHÍNH THỨC KỲ THI TUYỂN SINH VÀO LỚP 10 THPT MÔN NGỮ VĂN NĂM ...TỔNG HỢP ĐỀ THI CHÍNH THỨC KỲ THI TUYỂN SINH VÀO LỚP 10 THPT MÔN NGỮ VĂN NĂM ...
TỔNG HỢP ĐỀ THI CHÍNH THỨC KỲ THI TUYỂN SINH VÀO LỚP 10 THPT MÔN NGỮ VĂN NĂM ...Nguyen Thanh Tu Collection
 
Đề cương môn giải phẫu......................
Đề cương môn giải phẫu......................Đề cương môn giải phẫu......................
Đề cương môn giải phẫu......................TrnHoa46
 
BỘ LUYỆN NGHE VÀO 10 TIẾNG ANH DẠNG TRẮC NGHIỆM 4 CÂU TRẢ LỜI - CÓ FILE NGHE.pdf
BỘ LUYỆN NGHE VÀO 10 TIẾNG ANH DẠNG TRẮC NGHIỆM 4 CÂU TRẢ LỜI - CÓ FILE NGHE.pdfBỘ LUYỆN NGHE VÀO 10 TIẾNG ANH DẠNG TRẮC NGHIỆM 4 CÂU TRẢ LỜI - CÓ FILE NGHE.pdf
BỘ LUYỆN NGHE VÀO 10 TIẾNG ANH DẠNG TRẮC NGHIỆM 4 CÂU TRẢ LỜI - CÓ FILE NGHE.pdfNguyen Thanh Tu Collection
 
CD21 Exercise 2.1 KEY.docx tieng anh cho
CD21 Exercise 2.1 KEY.docx tieng anh choCD21 Exercise 2.1 KEY.docx tieng anh cho
CD21 Exercise 2.1 KEY.docx tieng anh chonamc250
 
ĐỀ CHÍNH THỨC KỲ THI TUYỂN SINH VÀO LỚP 10 THPT CÁC TỈNH THÀNH NĂM HỌC 2020 –...
ĐỀ CHÍNH THỨC KỲ THI TUYỂN SINH VÀO LỚP 10 THPT CÁC TỈNH THÀNH NĂM HỌC 2020 –...ĐỀ CHÍNH THỨC KỲ THI TUYỂN SINH VÀO LỚP 10 THPT CÁC TỈNH THÀNH NĂM HỌC 2020 –...
ĐỀ CHÍNH THỨC KỲ THI TUYỂN SINH VÀO LỚP 10 THPT CÁC TỈNH THÀNH NĂM HỌC 2020 –...Nguyen Thanh Tu Collection
 
TÀI LIỆU BỒI DƯỠNG HỌC SINH GIỎI LÝ LUẬN VĂN HỌC NĂM HỌC 2023-2024 - MÔN NGỮ ...
TÀI LIỆU BỒI DƯỠNG HỌC SINH GIỎI LÝ LUẬN VĂN HỌC NĂM HỌC 2023-2024 - MÔN NGỮ ...TÀI LIỆU BỒI DƯỠNG HỌC SINH GIỎI LÝ LUẬN VĂN HỌC NĂM HỌC 2023-2024 - MÔN NGỮ ...
TÀI LIỆU BỒI DƯỠNG HỌC SINH GIỎI LÝ LUẬN VĂN HỌC NĂM HỌC 2023-2024 - MÔN NGỮ ...Nguyen Thanh Tu Collection
 
Chuong trinh dao tao Su pham Khoa hoc tu nhien, ma nganh - 7140247.pdf
Chuong trinh dao tao Su pham Khoa hoc tu nhien, ma nganh - 7140247.pdfChuong trinh dao tao Su pham Khoa hoc tu nhien, ma nganh - 7140247.pdf
Chuong trinh dao tao Su pham Khoa hoc tu nhien, ma nganh - 7140247.pdfhoangtuansinh1
 
TÀI LIỆU BỒI DƯỠNG HỌC SINH GIỎI KỸ NĂNG VIẾT ĐOẠN VĂN NGHỊ LUẬN XÃ HỘI 200 C...
TÀI LIỆU BỒI DƯỠNG HỌC SINH GIỎI KỸ NĂNG VIẾT ĐOẠN VĂN NGHỊ LUẬN XÃ HỘI 200 C...TÀI LIỆU BỒI DƯỠNG HỌC SINH GIỎI KỸ NĂNG VIẾT ĐOẠN VĂN NGHỊ LUẬN XÃ HỘI 200 C...
TÀI LIỆU BỒI DƯỠNG HỌC SINH GIỎI KỸ NĂNG VIẾT ĐOẠN VĂN NGHỊ LUẬN XÃ HỘI 200 C...Nguyen Thanh Tu Collection
 
Campbell _2011_ - Sinh học - Tế bào - Ref.pdf
Campbell _2011_ - Sinh học - Tế bào - Ref.pdfCampbell _2011_ - Sinh học - Tế bào - Ref.pdf
Campbell _2011_ - Sinh học - Tế bào - Ref.pdfTrnHoa46
 
Kiểm tra cuối học kì 1 sinh học 12 đề tham khảo
Kiểm tra cuối học kì 1 sinh học 12 đề tham khảoKiểm tra cuối học kì 1 sinh học 12 đề tham khảo
Kiểm tra cuối học kì 1 sinh học 12 đề tham khảohoanhv296
 
GIÁO ÁN DẠY THÊM (KẾ HOẠCH BÀI DẠY BUỔI 2) - TIẾNG ANH 7 GLOBAL SUCCESS (2 CỘ...
GIÁO ÁN DẠY THÊM (KẾ HOẠCH BÀI DẠY BUỔI 2) - TIẾNG ANH 7 GLOBAL SUCCESS (2 CỘ...GIÁO ÁN DẠY THÊM (KẾ HOẠCH BÀI DẠY BUỔI 2) - TIẾNG ANH 7 GLOBAL SUCCESS (2 CỘ...
GIÁO ÁN DẠY THÊM (KẾ HOẠCH BÀI DẠY BUỔI 2) - TIẾNG ANH 7 GLOBAL SUCCESS (2 CỘ...Nguyen Thanh Tu Collection
 
30 ĐỀ PHÁT TRIỂN THEO CẤU TRÚC ĐỀ MINH HỌA BGD NGÀY 22-3-2024 KỲ THI TỐT NGHI...
30 ĐỀ PHÁT TRIỂN THEO CẤU TRÚC ĐỀ MINH HỌA BGD NGÀY 22-3-2024 KỲ THI TỐT NGHI...30 ĐỀ PHÁT TRIỂN THEO CẤU TRÚC ĐỀ MINH HỌA BGD NGÀY 22-3-2024 KỲ THI TỐT NGHI...
30 ĐỀ PHÁT TRIỂN THEO CẤU TRÚC ĐỀ MINH HỌA BGD NGÀY 22-3-2024 KỲ THI TỐT NGHI...Nguyen Thanh Tu Collection
 
SÁNG KIẾN ÁP DỤNG CLT (COMMUNICATIVE LANGUAGE TEACHING) VÀO QUÁ TRÌNH DẠY - H...
SÁNG KIẾN ÁP DỤNG CLT (COMMUNICATIVE LANGUAGE TEACHING) VÀO QUÁ TRÌNH DẠY - H...SÁNG KIẾN ÁP DỤNG CLT (COMMUNICATIVE LANGUAGE TEACHING) VÀO QUÁ TRÌNH DẠY - H...
SÁNG KIẾN ÁP DỤNG CLT (COMMUNICATIVE LANGUAGE TEACHING) VÀO QUÁ TRÌNH DẠY - H...Nguyen Thanh Tu Collection
 
sách sinh học đại cương - Textbook.pdf
sách sinh học đại cương   -   Textbook.pdfsách sinh học đại cương   -   Textbook.pdf
sách sinh học đại cương - Textbook.pdfTrnHoa46
 
30 ĐỀ PHÁT TRIỂN THEO CẤU TRÚC ĐỀ MINH HỌA BGD NGÀY 22-3-2024 KỲ THI TỐT NGHI...
30 ĐỀ PHÁT TRIỂN THEO CẤU TRÚC ĐỀ MINH HỌA BGD NGÀY 22-3-2024 KỲ THI TỐT NGHI...30 ĐỀ PHÁT TRIỂN THEO CẤU TRÚC ĐỀ MINH HỌA BGD NGÀY 22-3-2024 KỲ THI TỐT NGHI...
30 ĐỀ PHÁT TRIỂN THEO CẤU TRÚC ĐỀ MINH HỌA BGD NGÀY 22-3-2024 KỲ THI TỐT NGHI...Nguyen Thanh Tu Collection
 
30 ĐỀ PHÁT TRIỂN THEO CẤU TRÚC ĐỀ MINH HỌA BGD NGÀY 22-3-2024 KỲ THI TỐT NGHI...
30 ĐỀ PHÁT TRIỂN THEO CẤU TRÚC ĐỀ MINH HỌA BGD NGÀY 22-3-2024 KỲ THI TỐT NGHI...30 ĐỀ PHÁT TRIỂN THEO CẤU TRÚC ĐỀ MINH HỌA BGD NGÀY 22-3-2024 KỲ THI TỐT NGHI...
30 ĐỀ PHÁT TRIỂN THEO CẤU TRÚC ĐỀ MINH HỌA BGD NGÀY 22-3-2024 KỲ THI TỐT NGHI...Nguyen Thanh Tu Collection
 
1.DOANNGOCPHUONGTHAO-APDUNGSTEMTHIETKEBTHHHGIUPHSHOCHIEUQUA (1).docx
1.DOANNGOCPHUONGTHAO-APDUNGSTEMTHIETKEBTHHHGIUPHSHOCHIEUQUA (1).docx1.DOANNGOCPHUONGTHAO-APDUNGSTEMTHIETKEBTHHHGIUPHSHOCHIEUQUA (1).docx
1.DOANNGOCPHUONGTHAO-APDUNGSTEMTHIETKEBTHHHGIUPHSHOCHIEUQUA (1).docxTHAO316680
 
30 ĐỀ PHÁT TRIỂN THEO CẤU TRÚC ĐỀ MINH HỌA BGD NGÀY 22-3-2024 KỲ THI TỐT NGHI...
30 ĐỀ PHÁT TRIỂN THEO CẤU TRÚC ĐỀ MINH HỌA BGD NGÀY 22-3-2024 KỲ THI TỐT NGHI...30 ĐỀ PHÁT TRIỂN THEO CẤU TRÚC ĐỀ MINH HỌA BGD NGÀY 22-3-2024 KỲ THI TỐT NGHI...
30 ĐỀ PHÁT TRIỂN THEO CẤU TRÚC ĐỀ MINH HỌA BGD NGÀY 22-3-2024 KỲ THI TỐT NGHI...Nguyen Thanh Tu Collection
 
GIÁO TRÌNH KHỐI NGUỒN CÁC LOẠI - ĐIỆN LẠNH BÁCH KHOA HÀ NỘI
GIÁO TRÌNH  KHỐI NGUỒN CÁC LOẠI - ĐIỆN LẠNH BÁCH KHOA HÀ NỘIGIÁO TRÌNH  KHỐI NGUỒN CÁC LOẠI - ĐIỆN LẠNH BÁCH KHOA HÀ NỘI
GIÁO TRÌNH KHỐI NGUỒN CÁC LOẠI - ĐIỆN LẠNH BÁCH KHOA HÀ NỘIĐiện Lạnh Bách Khoa Hà Nội
 

Último (20)

TỔNG HỢP ĐỀ THI CHÍNH THỨC KỲ THI TUYỂN SINH VÀO LỚP 10 THPT MÔN NGỮ VĂN NĂM ...
TỔNG HỢP ĐỀ THI CHÍNH THỨC KỲ THI TUYỂN SINH VÀO LỚP 10 THPT MÔN NGỮ VĂN NĂM ...TỔNG HỢP ĐỀ THI CHÍNH THỨC KỲ THI TUYỂN SINH VÀO LỚP 10 THPT MÔN NGỮ VĂN NĂM ...
TỔNG HỢP ĐỀ THI CHÍNH THỨC KỲ THI TUYỂN SINH VÀO LỚP 10 THPT MÔN NGỮ VĂN NĂM ...
 
Đề cương môn giải phẫu......................
Đề cương môn giải phẫu......................Đề cương môn giải phẫu......................
Đề cương môn giải phẫu......................
 
BỘ LUYỆN NGHE VÀO 10 TIẾNG ANH DẠNG TRẮC NGHIỆM 4 CÂU TRẢ LỜI - CÓ FILE NGHE.pdf
BỘ LUYỆN NGHE VÀO 10 TIẾNG ANH DẠNG TRẮC NGHIỆM 4 CÂU TRẢ LỜI - CÓ FILE NGHE.pdfBỘ LUYỆN NGHE VÀO 10 TIẾNG ANH DẠNG TRẮC NGHIỆM 4 CÂU TRẢ LỜI - CÓ FILE NGHE.pdf
BỘ LUYỆN NGHE VÀO 10 TIẾNG ANH DẠNG TRẮC NGHIỆM 4 CÂU TRẢ LỜI - CÓ FILE NGHE.pdf
 
CD21 Exercise 2.1 KEY.docx tieng anh cho
CD21 Exercise 2.1 KEY.docx tieng anh choCD21 Exercise 2.1 KEY.docx tieng anh cho
CD21 Exercise 2.1 KEY.docx tieng anh cho
 
ĐỀ CHÍNH THỨC KỲ THI TUYỂN SINH VÀO LỚP 10 THPT CÁC TỈNH THÀNH NĂM HỌC 2020 –...
ĐỀ CHÍNH THỨC KỲ THI TUYỂN SINH VÀO LỚP 10 THPT CÁC TỈNH THÀNH NĂM HỌC 2020 –...ĐỀ CHÍNH THỨC KỲ THI TUYỂN SINH VÀO LỚP 10 THPT CÁC TỈNH THÀNH NĂM HỌC 2020 –...
ĐỀ CHÍNH THỨC KỲ THI TUYỂN SINH VÀO LỚP 10 THPT CÁC TỈNH THÀNH NĂM HỌC 2020 –...
 
1 - MÃ LỖI SỬA CHỮA BOARD MẠCH BẾP TỪ.pdf
1 - MÃ LỖI SỬA CHỮA BOARD MẠCH BẾP TỪ.pdf1 - MÃ LỖI SỬA CHỮA BOARD MẠCH BẾP TỪ.pdf
1 - MÃ LỖI SỬA CHỮA BOARD MẠCH BẾP TỪ.pdf
 
TÀI LIỆU BỒI DƯỠNG HỌC SINH GIỎI LÝ LUẬN VĂN HỌC NĂM HỌC 2023-2024 - MÔN NGỮ ...
TÀI LIỆU BỒI DƯỠNG HỌC SINH GIỎI LÝ LUẬN VĂN HỌC NĂM HỌC 2023-2024 - MÔN NGỮ ...TÀI LIỆU BỒI DƯỠNG HỌC SINH GIỎI LÝ LUẬN VĂN HỌC NĂM HỌC 2023-2024 - MÔN NGỮ ...
TÀI LIỆU BỒI DƯỠNG HỌC SINH GIỎI LÝ LUẬN VĂN HỌC NĂM HỌC 2023-2024 - MÔN NGỮ ...
 
Chuong trinh dao tao Su pham Khoa hoc tu nhien, ma nganh - 7140247.pdf
Chuong trinh dao tao Su pham Khoa hoc tu nhien, ma nganh - 7140247.pdfChuong trinh dao tao Su pham Khoa hoc tu nhien, ma nganh - 7140247.pdf
Chuong trinh dao tao Su pham Khoa hoc tu nhien, ma nganh - 7140247.pdf
 
TÀI LIỆU BỒI DƯỠNG HỌC SINH GIỎI KỸ NĂNG VIẾT ĐOẠN VĂN NGHỊ LUẬN XÃ HỘI 200 C...
TÀI LIỆU BỒI DƯỠNG HỌC SINH GIỎI KỸ NĂNG VIẾT ĐOẠN VĂN NGHỊ LUẬN XÃ HỘI 200 C...TÀI LIỆU BỒI DƯỠNG HỌC SINH GIỎI KỸ NĂNG VIẾT ĐOẠN VĂN NGHỊ LUẬN XÃ HỘI 200 C...
TÀI LIỆU BỒI DƯỠNG HỌC SINH GIỎI KỸ NĂNG VIẾT ĐOẠN VĂN NGHỊ LUẬN XÃ HỘI 200 C...
 
Campbell _2011_ - Sinh học - Tế bào - Ref.pdf
Campbell _2011_ - Sinh học - Tế bào - Ref.pdfCampbell _2011_ - Sinh học - Tế bào - Ref.pdf
Campbell _2011_ - Sinh học - Tế bào - Ref.pdf
 
Kiểm tra cuối học kì 1 sinh học 12 đề tham khảo
Kiểm tra cuối học kì 1 sinh học 12 đề tham khảoKiểm tra cuối học kì 1 sinh học 12 đề tham khảo
Kiểm tra cuối học kì 1 sinh học 12 đề tham khảo
 
GIÁO ÁN DẠY THÊM (KẾ HOẠCH BÀI DẠY BUỔI 2) - TIẾNG ANH 7 GLOBAL SUCCESS (2 CỘ...
GIÁO ÁN DẠY THÊM (KẾ HOẠCH BÀI DẠY BUỔI 2) - TIẾNG ANH 7 GLOBAL SUCCESS (2 CỘ...GIÁO ÁN DẠY THÊM (KẾ HOẠCH BÀI DẠY BUỔI 2) - TIẾNG ANH 7 GLOBAL SUCCESS (2 CỘ...
GIÁO ÁN DẠY THÊM (KẾ HOẠCH BÀI DẠY BUỔI 2) - TIẾNG ANH 7 GLOBAL SUCCESS (2 CỘ...
 
30 ĐỀ PHÁT TRIỂN THEO CẤU TRÚC ĐỀ MINH HỌA BGD NGÀY 22-3-2024 KỲ THI TỐT NGHI...
30 ĐỀ PHÁT TRIỂN THEO CẤU TRÚC ĐỀ MINH HỌA BGD NGÀY 22-3-2024 KỲ THI TỐT NGHI...30 ĐỀ PHÁT TRIỂN THEO CẤU TRÚC ĐỀ MINH HỌA BGD NGÀY 22-3-2024 KỲ THI TỐT NGHI...
30 ĐỀ PHÁT TRIỂN THEO CẤU TRÚC ĐỀ MINH HỌA BGD NGÀY 22-3-2024 KỲ THI TỐT NGHI...
 
SÁNG KIẾN ÁP DỤNG CLT (COMMUNICATIVE LANGUAGE TEACHING) VÀO QUÁ TRÌNH DẠY - H...
SÁNG KIẾN ÁP DỤNG CLT (COMMUNICATIVE LANGUAGE TEACHING) VÀO QUÁ TRÌNH DẠY - H...SÁNG KIẾN ÁP DỤNG CLT (COMMUNICATIVE LANGUAGE TEACHING) VÀO QUÁ TRÌNH DẠY - H...
SÁNG KIẾN ÁP DỤNG CLT (COMMUNICATIVE LANGUAGE TEACHING) VÀO QUÁ TRÌNH DẠY - H...
 
sách sinh học đại cương - Textbook.pdf
sách sinh học đại cương   -   Textbook.pdfsách sinh học đại cương   -   Textbook.pdf
sách sinh học đại cương - Textbook.pdf
 
30 ĐỀ PHÁT TRIỂN THEO CẤU TRÚC ĐỀ MINH HỌA BGD NGÀY 22-3-2024 KỲ THI TỐT NGHI...
30 ĐỀ PHÁT TRIỂN THEO CẤU TRÚC ĐỀ MINH HỌA BGD NGÀY 22-3-2024 KỲ THI TỐT NGHI...30 ĐỀ PHÁT TRIỂN THEO CẤU TRÚC ĐỀ MINH HỌA BGD NGÀY 22-3-2024 KỲ THI TỐT NGHI...
30 ĐỀ PHÁT TRIỂN THEO CẤU TRÚC ĐỀ MINH HỌA BGD NGÀY 22-3-2024 KỲ THI TỐT NGHI...
 
30 ĐỀ PHÁT TRIỂN THEO CẤU TRÚC ĐỀ MINH HỌA BGD NGÀY 22-3-2024 KỲ THI TỐT NGHI...
30 ĐỀ PHÁT TRIỂN THEO CẤU TRÚC ĐỀ MINH HỌA BGD NGÀY 22-3-2024 KỲ THI TỐT NGHI...30 ĐỀ PHÁT TRIỂN THEO CẤU TRÚC ĐỀ MINH HỌA BGD NGÀY 22-3-2024 KỲ THI TỐT NGHI...
30 ĐỀ PHÁT TRIỂN THEO CẤU TRÚC ĐỀ MINH HỌA BGD NGÀY 22-3-2024 KỲ THI TỐT NGHI...
 
1.DOANNGOCPHUONGTHAO-APDUNGSTEMTHIETKEBTHHHGIUPHSHOCHIEUQUA (1).docx
1.DOANNGOCPHUONGTHAO-APDUNGSTEMTHIETKEBTHHHGIUPHSHOCHIEUQUA (1).docx1.DOANNGOCPHUONGTHAO-APDUNGSTEMTHIETKEBTHHHGIUPHSHOCHIEUQUA (1).docx
1.DOANNGOCPHUONGTHAO-APDUNGSTEMTHIETKEBTHHHGIUPHSHOCHIEUQUA (1).docx
 
30 ĐỀ PHÁT TRIỂN THEO CẤU TRÚC ĐỀ MINH HỌA BGD NGÀY 22-3-2024 KỲ THI TỐT NGHI...
30 ĐỀ PHÁT TRIỂN THEO CẤU TRÚC ĐỀ MINH HỌA BGD NGÀY 22-3-2024 KỲ THI TỐT NGHI...30 ĐỀ PHÁT TRIỂN THEO CẤU TRÚC ĐỀ MINH HỌA BGD NGÀY 22-3-2024 KỲ THI TỐT NGHI...
30 ĐỀ PHÁT TRIỂN THEO CẤU TRÚC ĐỀ MINH HỌA BGD NGÀY 22-3-2024 KỲ THI TỐT NGHI...
 
GIÁO TRÌNH KHỐI NGUỒN CÁC LOẠI - ĐIỆN LẠNH BÁCH KHOA HÀ NỘI
GIÁO TRÌNH  KHỐI NGUỒN CÁC LOẠI - ĐIỆN LẠNH BÁCH KHOA HÀ NỘIGIÁO TRÌNH  KHỐI NGUỒN CÁC LOẠI - ĐIỆN LẠNH BÁCH KHOA HÀ NỘI
GIÁO TRÌNH KHỐI NGUỒN CÁC LOẠI - ĐIỆN LẠNH BÁCH KHOA HÀ NỘI
 

Các hướng tư duy và phương pháp giải trong hình oxy

  • 1. GI I ðÁP TOÁN C P 3 – THI ð I H C CÁC BÀI TOÁN TRONG TAM GIÁC, T GIÁC CÁC BÀI TOÁN V ðƯ NG TH NG CÁC BÀI TOÁN V ðƯ NG TRÒN CÁC BÀI TOÁN V ELIP CÁC HƯ NG TƯ DUY VÀ PHƯƠNG PHÁP GI I TRONG HÌNH H C OXY Biên so n: Thanh Tùng *) Tóm t t lý thuy t ñ y ñ theo m t trình t logic và có h th ng. *) ðưa ra các hư ng tư duy và phương pháp gi i khái quát cho t ng l p bài toán. *) Có bài toán m u minh h a ñi kèm. *) Ph n bài t p áp d ng có g i ý. *) L i gi i chi ti t cho t ng bài toán c th (tham kh o thêm trên http://www.facebook.com/giaidaptoancap3 ). BÀI TOÁN TÌM ðI M H À N I 0 3 / 2 0 1 3
  • 2. 2 CÁC HƯ NG TƯ DUY VÀ PHƯƠNG PHÁP GI I HÌNH H C OXY A. KI N TH C CƠ B N
  • 3. 3
  • 4. 4 B. CÁC BÀI TOÁN BÀI TOÁN 1: BÀI TOÁN TÌM ðI M ð hi u rõ hơn cho 4 hư ng tư duy tương ng v i 4 TH c a Bài toán 1: “Bài Toán Tìm ði m” th y s dùng 6 bài thi ð i H c năm 2012 v a qua ñ minh h a. 1) (A, A1 – 2012:CB). Cho hình vuông ABCD. G i M là trung ñi m c a c nh BC, N là ñi m trên c nh CD sao cho CN = 2ND. Gi s 11 1 ; 2 2 M       và ñư ng th ng AN có phương trình 2 3 0x y− − = . Tìm t a ñ ñi m A. 2) (A, A1 – 2012 :NC). Cho ñư ng tròn 2 2 ( ) : 8C x y+ = . Vi t phương trình chính t c c a elip (E), bi t r ng (E) có ñ dài tr c l n b ng 8 và (E) c t ( )C t i b n ñi m phân bi t t o thành b n ñ nh c a m t hình vuông. 3) (B – 2012:CB). Cho ñư ng tròn 2 2 1( ) : 4C x y+ = , 2 2 2( ) : 12 18 0C x y x+ − + = và ñư ng th ng : 4 0d x y− − = . Vi t phương trình ñư ng tròn có tâm thu c 2( )C , ti p xúc v i d và c t 1( )C t i hai ñi m phân bi t A và B sao cho AB vuông góc v i d. 4) (B – 2012 :NC). Cho hình thoi ABCD có AC = 2BD và ñư ng tròn ti p xúc v i các c nh c a hình thoi có phương trình 2 2 4x y+ = . Vi t phương trình chính t c c a elip (E) ñi qua các ñ nh A, B, C, D c a hình thoi. Bi t A thu c Ox. 5) (D – 2012:CB). Cho hình ch nh t ABCD. Các ñư ng th ng AC và AD l n lư t có phương trình là 3 0x y+ = và 4 0x y− + = ; ñư ng th ng BD ñi qua ñi m 1 ( ;1) 3 M − . Tìm t a ñ các ñ nh c a hình ch nh t ABCD. 6) (D – 2012 :NC). Cho ñư ng th ng : 2 3 0d x y− + = . Vi t phương trình ñư ng tròn có tâm thu c d , c t tr c Ox t i A và B, c t tr c Oy t i C và D sao cho AB = CD = 2.
  • 5. 5 1) (A, A1 – 2012:CB). Cho hình vuông ABCD. G i M là trung ñi m c a c nh BC, N là ñi m trên c nh CD sao cho CN = 2ND. Gi s 11 1 ; 2 2 M       và ñư ng th ng AN có phương trình 2 3 0x y− − = . Tìm t a ñ ñi m A. Cách 1 Phân tích: : +) Ta có { }A AN AM= ∩ nên Theo hư ng tư duy 1 (TH1) ta ph i ñi l p thêm phương trình AM +) Bi t M nhưng chưa bi t A (chính là ñáp s ta c n tìm) nên ta ph i ñi tìm thêm vtpt ho c vtcp +) Bài toán không có y u t song song, vuông góc ñ tìm vtpt ho c vtcp nên ta ph i khai thác yt ñ nh lư ng +) Y u t ñ nh lư ng: cos MAN∠ = ( )cos ,AM ANn n uuuur uuur AMn⇒ uuuur ⇒ phương trình AM → t a ñ ñi m A Gi i: ð t AB a= 2 ; ; 3 3 2 a a a ND NC MB MC⇒ = = = = ( vì ABCD là hình vuông và 2CN ND= ) Và áp d ng Pitago ta ñư c: 5 5 ; 2 6 a a AM MN= = và 10 3 a AN = Trong AMN∆ ta có: cos MAN∠ 2 2 2 2 2 . 2 AM AN MN AM AN + − = = G i ( ; )AMn a b= uuuur là vtpt c a AM và ta có (2; 1)ANn = − uuur cos⇒ MAN∠ = ( )cos ,AM ANn n uuuur uuur 2 2 2 2 2 2 2 2 2 322 2(2 ) 5( ) 3 8 3 0 (3 )( 3 ) 0 32 . 2 1 a ba b a b a b a ab b a b a b a ba b = −−  ⇔ = ⇔ − = + ⇔ − − = ⇔ + − = ⇔  =+ +  +) V i 3a b= − ch n 1; 3a b= = − (1; 3)AMn⇒ = − uuuur ⇒ phương trình 11 1 : 3 0 2 2 AM x y     − − − =        hay : 3 4 0AM x y− − = . Vì { }A AN AM= ∩ nên ta gi i h : 2 3 0 1 (1; 1) 3 4 0 1 x y x A x y y − − = =  ⇔ ⇒ −  − − = = −  +) V i 3a b= ch n 3; 1a b= = (3;1)AMn⇒ = uuuur ⇒ phương trình 11 1 :3 0 2 2 AM x y     − + − =        hay :3 17 0AM x y+ − = . Vì { }A AN AM= ∩ nên ta gi i h : 2 3 0 4 (4;5) 3 17 0 5 x y x A x y y − − = =  ⇔ ⇒  + − = =  V y (1; 1)A − ho c (4;5)A
  • 6. 6 Cách 2: Phân tích: A AN∈ nên Theo hư ng tư duy 2 (TH2) ta g i ( )A t AN∈ ta c n thi t l p 1 phương trình ( ) 0f t = (còn d ki n 11 1 ; 2 2 M       là trung ñi m c a BC ta chưa s d ng – s giúp ta làm ñi u này) ?t A→ = → Gi i: +) G i H là hình chi u c a M lên AN 2 2 11 1 2. 3 3 52 2 ( , ) 22 1 MH d M AN − − ⇒ = = = + ð t AB a= 2 ; ; 3 3 2 a a a ND NC MB MC⇒ = = = = ( vì ABCD là hình vuông và 2CN ND= ) Và áp d ng Pitago ta ñư c: 5 5 ; 2 6 a a AM MN= = và 10 3 a AN = Trong AMN∆ ta có: cos MAN∠ 2 2 2 2 2 . 2 AM AN MN AM AN + − = = ⇒ MAN∠ = 0 45 MAH⇒ ∆ c n t i H 3 5 3 10 2 2. 2 2 AM MH⇒ = = = (*) +) G i ( ;2 3)A t t AN− ∈ và 2 45 2 AM = (theo (*)) ⇔ 2 2 2 1 (1; 1)11 7 45 2 5 4 0 4 (4;5)2 2 2 t A t t t t t A = −     − + − = ⇔ − + = ⇔ ⇒     =      V y (1; 1)A − ho c (4;5)A Cách 3: Phân tích: A AN∈ và 11 1 ; 2 2 M       c ñ nh . N u AM h const= = ( ta s tìm cách ñi tính AM ). Nên Theo hư ng tư duy 3 (TH3) : { } ( )A AN C= ∩ v i ( )C là ñư ng tròn tâm M bán kính R h=
  • 7. 7 Gi i: +) G i H là hình chi u c a M lên AN 2 2 11 1 2. 3 3 52 2 ( , ) 22 1 MH d M AN − − ⇒ = = = + ð t AB a= 2 ; ; 3 3 2 a a a ND NC MB MC⇒ = = = = ( vì ABCD là hình vuông và 2CN ND= ) Và áp d ng Pitago ta ñư c: 5 5 ; 2 6 a a AM MN= = và 10 3 a AN = Trong AMN∆ ta có: cos MAN∠ 2 2 2 2 2 . 2 AM AN MN AM AN + − = = ⇒ MAN∠ = 0 45 MAH⇒ ∆ c n t i H 3 5 3 10 2 2. 2 2 AM MH⇒ = = = V y 3 10 2 AM = ⇒ A n m trên ñư ng tròn có phương trình: 2 2 11 1 45 2 2 2 x y     − + − =        Mà : 2 3 0A AN x y∈ − − = Nên ta xét h : 2 2 11 1 45 1 2 2 2 1 2 3 0 xx y y x y     =− + − =    ⇔     = − − − = ho c 4 5 x y =  = V y (1; 1)A − ho c (4;5)A Cách 4: (Các em có th tham kh o thêm cách gi i c a B Giáo D c nhưng vì cách gi i này theo th y không ñư c “t nhiên” nên th y không trình bày ñây) 2) (A, A1 – 2012 :NC). Cho ñư ng tròn 2 2 ( ) : 8C x y+ = . Vi t phương trình chính t c c a elip (E), bi t r ng (E) có ñ dài tr c l n b ng 8 và (E) c t ( )C t i b n ñi m phân bi t t o thành b n ñ nh c a m t hình vuông. Phân tích: +) Phương trình ( )E : 2 2 2 2 1 x y a b + = như v y ta c n tìm ;a b +) (E) có ñ dài tr c l n b ng 8 2 8 4a a⇒ = ⇒ = +) Theo Hư ng tư duy 4 (TH4) ta g i ( ; )A x y ( 0x > ) là m t giao ñi m c a (E) và ( )C : 2 2 ( ) 8A C x y∈ ⇒ + = và d ki n (E) c t ( )C t i b n ñi m phân bi t t o thành b n ñ nh c a m t hình vuông giúp ta thi t l p thêm phương trình: y x= (4 ñ nh n m trên hai ñư ng phân giác thu c góc ph n tư th nh t và th hai – nhưng vì ta ch n ñi m ( ; )A x y ( 0x > ) thu c góc ph n tư th nh t)⇒ t a ñ ñi m A +) Mà ( )A E b∈ ⇒ → phương trình (E). Gi i: G i phương trình chính t c c a elip ( )E có d ng: 2 2 2 2 1 x y a b + = +) (E) có ñ dài tr c l n b ng 8 2 8 4a a⇒ = ⇒ = +) G i ( ; )A x y ( 0x > ) là m t giao ñi m c a (E) và ( )C .Ta có: 2 2 ( ) 8A C x y∈ ⇒ + = (1) M t khác: (E) c t ( )C t i b n ñi m phân bi t t o thành b n ñ nh c a m t hình vuông ⇒ y x= (2) T (1) và (2) 2 2 8 2x x⇒ = ⇒ = (vì 0x > ) 2 (2;2)y A⇒ = ⇒ +) Mà ( )A E∈ 2 2 2 2 2 2 2 16 1 4 3 b b ⇒ + = ⇒ = . V y phương trình chính t c c a elip (E) là: 2 2 1 1616 3 x y + =
  • 8. 8 3) (B – 2012:CB). Cho ñư ng tròn 2 2 1( ) : 4C x y+ = , 2 2 2( ) : 12 18 0C x y x+ − + = và ñư ng th ng : 4 0d x y− − = . Vi t phương trình ñư ng tròn có tâm thu c 2( )C , ti p xúc v i d và c t 1( )C t i hai ñi m phân bi t A và B sao cho AB vuông góc v i d. Phân tích: Mu n vi t phương trình ñư ng tròn ta c n: +) Xác ñ nh tâm I (dùng Thu t Toán Tìm ði m) . Khi ñó theo Hư ng tư duy 2 (TH2) ta g i 1( )I t II∈ (Trư c ñó ta ñi l p phương trình 1II ñi qua 1I vuông góc v i AB (tính ch t ñư ng n i tâm) hay song song v i d ) Và d ki n 2( )I C∈ giúp ta thi t l p ñư c phương trình : ( ) 0 ?f t t= → = → t a ñ ñi m I ( Ta có th làm theo Hư ng tư duy 3 (TH3) v i { } 1 2( )I II C= ∩ → t a ñ I - cách trình bày khác c a TH2) +) Xác ñ nh bán kính: R nh ( , )R d I d= Gi i: G i I là tâm ñư ng tròn ( )C c n vi t phương trình. Ta có 2 2 1( ) : 4C x y+ = ⇒ tâm c a 1( )C là 1(0;0)I Vì 1 1 II AB II AB d ⊥ ⇒ ⊥ // d ⇒ phương trình 1II : 0x y− = . G i 1( ; )I t t II∈ mà 2( )I C∈ 2 2 12 18 0t t t⇒ + − + = 2 6 9 0 3t t t⇔ − + = ⇔ = (3;3)I⇒ Mà ( )C ti p xúc v i d ⇒ 2 2 3 3 4 ( , ) 2 2 1 1 R d I d − + = = = + . V y phương trình( )C là: 2 2 ( 3) ( 3) 8x y− + − = 4) (B – 2012 :NC). Cho hình thoi ABCD có AC = 2BD và ñư ng tròn ti p xúc v i các c nh c a hình thoi có phương trình 2 2 4x y+ = . Vi t phương trình chính t c c a elip (E) ñi qua các ñ nh A, B, C, D c a hình thoi. Bi t A thu c Ox. Phân tích: +) Phương trình ( )E : 2 2 2 2 1 x y a b + = ( 0)a b> > như v y ta c n tìm ;a b +) Theo Hư ng tư duy 2 (TH2) vì (E) ñi qua các ñ nh A, B, C, D và A Ox∈ nên g i ( ;0)A a Ox∈ và (0; )B b Oy∈ +) Khai thác d ki n: AC = 2BD 1( , ) 0f a b→ = (1) +) Khai thác d ki n: ñư ng tròn 2 2 4x y+ = ti p xúc v i các c nh c a hình thoi 2 ( , ) 0f a b→ = (2) T (1) và (2) 2 ?a→ = và 2 ?b = → phương trình (E).
  • 9. 9 Gi i: G i phương trình chính t c c a elip ( )E : 2 2 2 2 1 x y a b + = ( v i 0a b> > ) Vì (E) ñi qua các ñ nh A, B, C, D và A Ox∈ nên không m t tính t ng quát gi s : ( ;0)A a và (0; )B b . Mà hình thoi ABCD có AC = 2BD 2 4 2OA OB OA OB⇔ = ⇔ = 2a b⇔ = (vì 0a b> > ) hay (2 ;0)A b , (0; )B b G i H là hình chi u c a O lên AB 2OH R⇒ = = ( vì ñư ng tròn 2 2 4x y+ = ti p xúc v i các c nh c a hình thoi) Xét tam giác OAB ta có: 2 2 2 1 1 1 OH OA OB = + hay 2 2 2 1 1 1 5 4 4 b b b = + ⇔ = 2 2 4 20a b⇒ = = V y phương trình chính t c c a elip ( )E là: 2 2 1 20 5 x y + = 5) (D – 2012:CB). Cho hình ch nh t ABCD. Các ñư ng th ng AC và AD l n lư t có phương trình là 3 0x y+ = và 4 0x y− + = ; ñư ng th ng BD ñi qua ñi m 1 ( ;1) 3 M − . Tìm t a ñ các ñ nh c a hình ch nh t ABCD. Cách 1: Phân tích: +) Theo Hư ng tư duy 1 (TH1) : { }A AC AD= ∩ → t a ñ ñi m A +) Theo Hư ng tư duy 2 (TH2) : D AD∈ , B AB∈ nên ta g i 1 2( ), ( )D t B t (trư c ñó ta ñi l p pt AB ) +) G i { }I AC BD= ∩ ( I là trung ñi m c a AC và BD ) 1 2( , )I t t⇒ mà 1 1 2( , ) 0I AC f t t∈ ⇒ = (1) Vì ,MB MD uuur uuuur cùng phương 2 1 2( , ) 0f t t⇒ = (2) +) T (1) và (2) 1 2 ? ? t t = ⇒ ⇒ = t a ñ c a , ,B D I và C Gi i: Vì { }A AC AD= ∩ nên xét h : 3 0 4 0 x y x y + =  − + = 3 1 x y = − ⇔ ⇒ = ( 3;1)A − AB ñi qua A và vuông góc v i AD nên AB có phương trình: 3 1 2 0 1 1 x y x y + − = ⇔ + + = − G i 1 1( ; 2)B t t AB− − ∈ và 2 2( ; 4)D t t AD+ ∈ ( 1 2; 3t t ≠ − ) 2 1 2 1 2 ; 2 2 t t t t I + − +  ⇒     : là trung ñi m c a BD Mà 2 1 2 1 2 1 1 2 2 3. 0 2 3 0 2 3 2 2 t t t t I AC t t t t + − + ∈ ⇒ + = ⇔ − + = ⇔ = + (*) Có: 1 1 2 2 1 10 ; 3 2 ; 2 6 3 3 MB t t t t     = + − − = + − −        uuur (theo (*)) và 2 2 1 ; 3 3 MD t t   = + +    uuuur M t khác , ,B D M th ng hàng⇒ ,MB MD uuur uuuur cùng phương 2 2 2 2 2 6 10 2 6 2 1 3 1 3 t t t t t + − − ⇒ = = − ⇔ = − + + 1 1t⇒ = ⇒ (1; 3), ( 1;3)B D− − và (0;0)I ⇒ (3; 1)C − ( vì I là trung ñi m c a AC )
  • 10. 10 5) (D – 2012:CB). Cho hình ch nh t ABCD. Các ñư ng th ng AC và AD l n lư t có phương trình là 3 0x y+ = và 4 0x y− + = ; ñư ng th ng BD ñi qua ñi m 1 ( ;1) 3 M − . Tìm t a ñ các ñ nh c a hình ch nh t ABCD. Cách 2: Phân tích: +) Theo Hư ng tư duy 1 (TH1) : { }A AC AD= ∩ → t a ñ ñi m A +) Do trong bài toán có nhi u tính ch t ñ i x ng nên ta nghĩ t i vi c tìm các ñi m ph liên quan. C th : +) Ta tìm ñi m N ñ i x ng v i M qua ñư ng trung tr c d c a AD b ng cách vi t pt 'd ñi qua M song song v i AD và { } 'N d AC= ∩ ⇒ pt trung tr c d c a AD ⇒ t a ñ trung ñi m ,I J c a AC và AD ⇒ t a ñ , ,C D B Gi i: Vì { }A AC AD= ∩ nên xét h : 3 0 4 0 x y x y + =  − + = 3 1 x y = − ⇔ ⇒ = ( 3;1)A − Phương trình c a 'd ñi qua M song song AD có d ng: 1 ( 1) 0 3 3 4 0 3 x y x y+ − − = ⇔ − + = G i { } 'N d AC= ∩ nên ta xét h : 13 0 1 1;13 3 4 0 33 xx y N yx y = −+ =    ⇔ ⇒ −   =− + =    G i d là ñư ng trung tr c c a AD c t , ,MN AC AD l n lư t t i , ,H I J ⇒ , ,H I J l n lư t là trung ñi m , ,MN AC AD 5 5 ; 4 4 H   ⇒ −    ⇒ pt c a d : 5 5 0 0 4 4 x y x y     + + − = ⇔ + =        Ta có: }{I d AC= ∩ nên ta xét h : ( ) 0 0 0;0 3 0 0 x y x I x y y + = =  ⇔ ⇒  + = =  ⇒ (3; 1)C − ( I là trung ñi m c a AC ) và }{J d AD= ∩ nên ta xét h : ( ) 0 2 2;2 4 0 2 x y x J x y y + = = −  ⇔ ⇒ −  − + = =  ⇒ ( 1;3)D − ( J là trung ñi m c a AD ) ⇒ (1; 3)B − ( I là trung ñi m c a BD ) 6) (D – 2012 :NC). Cho ñư ng th ng : 2 3 0d x y− + = . Vi t phương trình ñư ng tròn có tâm thu c d , c t tr c Ox t i A và B, c t tr c Oy t i C và D sao cho AB = CD = 2.
  • 11. 11 Phân tích: Mu n vi t phương trình ñư ng tròn ta c n: +) Xác ñ nh tâm I (dùng Thu t Toán Tìm ði m) . Khi ñó theo Hư ng tư duy 2 (TH2) ta g i ( )I t d∈ Và d ki n AB CD= giúp ta thi t l p ñư c phương trình : ( ) 0 ?f t t= → = → t a ñ ñi m I +) Xác ñ nh bán kính: R nh 2 2 2 2 R IA IH HA= = + v i ( , )IH d I Ox= và 1 2 AB HA = = Gi i: +) G i I là tâm ñư ng tròn c n l p và g i ( ;2 3)I t t d+ ∈ +) Ta có AB CD= 2 3 3 ( 3; 3) ( , ) ( , ) 2 3 2 3 1 ( 1;1) t t t I d I Ox d I Oy t t t t t I + = = − − −   ⇔ = ⇔ + = ⇔ ⇔ ⇒  + = − = − −   +) V i ( 3; 3)I − − ( , ) 3 3IH d I Ox⇒ = = − = và ta có: 2 1 2 2 AB AH = = = 2 2 2 2 10R IA IH HA⇒ = = + = V y phương trình ñư ng tròn: 2 2 ( 3) ( 3) 10x y+ + + = . +) V i ( 1;1)I − ( , ) 1 1IH d I Ox⇒ = = = và ta có: 2 1 2 2 AB AH = = = 2 2 2 2 2R IA IH HA⇒ = = + = V y phương trình ñư ng tròn: 2 2 ( 1) ( 1) 2x y+ + − = . CHÚ Ý: Trư c khi vào ph n BÀI TOÁN 2 chúng ta có m t s quy ư c sau: +) ( )M t ∈∆ : ta ràng bu c ñi m M theo m t n là t. +) 1 2( , )M t t : ñi m M có t a ñ ph thu c vào hai n 1t và 2t . +) 1 2( ; )M t t : ñi m M có t a ñ : 1 2 M M x t y t =  = BÀI TOÁN 2: CÁC BÀI TOÁN LIÊN QUAN ð N BÀI TOÁN 1 D ng 1: Các bài toán trong tam giác, t giác Lo i 1: Các bài toán v ð nh Tính Lo i 1.1: Các bài toán v ñư ng trung tuy n, ñư ng cao, trung tr c Bài 1: Bi t ñ nh A c a tam giác ABC và 2 trung tuy n BM, CN. Vi t phương trình các c nh c a ABC∆ . Cách gi i: Bài t p áp d ng (Các em hãy d a vào ý tư ng Bài 1 ñ gi i các ví d sau) Ví d 1: Cho tam giác ABC có A(4; – 1) và phương trình hai ñư ng trung tuy n BM: 8x – y – 3 = 0, CN: 14x – 13y – 9 = 0. Tìm t a ñ các ñ nh B, C. (ðs: B(1; 5), C(–4; – 5)) Ví d 2: Cho tam giác ABC có A(1; – 2) và phương trình hai ñư ng trung tuy n BM và CN l n lư t là x – 6y + 3 = 0 và 5x – 6y – 1 = 0. Tính di n tích tam giác ABC. (ðs: 16ABCS∆ = (ñvdt))
  • 12. 12 Ví d 3: Cho tam giác ABC có tr ng tâm G(-2;0) bi t phương trình các c nh AB, AC theo th t là 4x + y + 14 = 0 và 2x + 5y – 2 = 0. Vi t phương trình BC. (ðs: x – 2y – 1 = 0 ; v i B( 3; 2)− − , C(1;0) ) Ví d 4: Cho hai ñư ng th ng 1d : x – y + 1 = 0, 2d : 2x + y – 1 = 0 và ñi m P(2; 1). Vi t phương trình ñư ng th ng 3d qua P và c t 1d , 2d l n lư t t i A và B sao cho P là trung ñi m c a AB. (ðs: 4x – y – 7 = 0) Ví d 5: Cho tam giác ABC có trung ñi m c a AB là I(1; 3), trung ñi m AC là J(-3; 0). ði m A thu c Oy và ñư ng BC qua g c t a ñ O. Tìm t a ñ các ñ nh c a tam giác ABC. (ðs: A 9 0; 2       , B 3 2; 2       , 9 6; 2   − −    ) ( Các em tham kh o ph n gi i m u qua các Ví d 2, Ví d 3, Ví d 5 ) Ví d 2: Cho tam giác ABC có A(1; – 2) và phương trình hai ñư ng trung tuy n BM và CN l n lư t là x – 6y + 3 = 0 và 5x – 6y – 1 = 0. Tính di n tích tam giác ABC. Gi i: +) G i B 3 ; 6 t t +  ∈    BM . Do N là trung ñi m c a AB 1 2 2 9 2 12 A B N A B N x x t x y y t x + + = = ⇒  + − = =  ⇒ N 1 9 ; 2 12 t t+ −      Mà N∈CN 1 9 5. 6. 1 0 3 2 12 t t t + − ⇒ − − = ⇔ = − ⇒ B ( 3;0)− +) T a ñ tr ng tâm G c a ∆ ABC là nghi m c a h : 1 6 3 0 2 5 6 1 0 3 x x y x y y = − + =  ⇔  − − = =  ⇒ G 2 1; 3       3 3 1 3 5 3 2 2 0 4 C G A B C G A B x x x x y y y y = − − = − + = ⇒  = − − = + − = ⇒ C (5;4) .V y phương trình BC: 3 2 3 0 8 4 x y x y + = ⇔ − + = G i H là chân ñư ng cao k t A xu ng BC ⇒ AH = 2 2 1 2.( 2) 3 8 ( ; ) 51 2 d A BC − − + = = + Ta có: BC = 2 2 8 4 4 5+ = 1 1 8 . . .4 5 16 2 2 5 ABCS AH BC∆⇒ = = = (ñvdt) Ví d 3: Cho tam giác ABC có tr ng tâm G(-2;0) bi t phương trình các c nh AB, AC theo th t là 4x + y + 14 = 0 và 2x + 5y – 2 = 0. Vi t phương trình BC Gi i:
  • 13. 13 +) T a ñ ñi m A là nghi m c a h : 4 14 0 2 5 2 0 x y x y + + =  + − = 4 2 x y = − ⇔  = ⇒ A( 4;2)− +) G i B 1 1( ; 4 14)t t− − ∈AB và C 2 2(5 1; 2 )t t+ − ∈AC +) Vì G là tr ng ∆ ABC nên ta có: 3 3 A B C G A B C G x x x x y y y y + + =  + + = 1 2 1 2 1 1 2 1 2 2 4 5 1 6 5 3 3 2 4 14 2 0 2 6 0 t t t t t t t t t t − + + + = − + = − = −   ⇒ ⇔ ⇔   − − − = + = − =   ⇒ B(-3;-2) C(1;0)    V y phương trình BC: 3 2 4 2 x y+ + = ⇔ 2 1 0x y− − = Ví d 5: Cho tam giác ABC có trung ñi m c a AB là I(1; 3), trung ñi m AC là J(-3; 0). ði m A thu c Oy và ñư ng BC qua g c t a ñ O. Tìm t a ñ các ñ nh c a tam giác ABC. Gi i: G i A(0; ) Oya ∈ B(2;6 a) C( 6; a) − ⇒  − − ( Vì I(1; 3), J(-3; 0) l n lư t là trung ñi m c a AB và AC) Ta có: OB (2;6 a) OC ( 6; a)  = −  = − − uuur uuur Mà BC ñi qua g c t a ñ O hay O,B,C th ng hàng 2 6 9 6 2 a a a − ⇒ = ⇔ = − − ⇒ A 9 0; 2       , B 3 2; 2       , C 9 6; 2   − −    Bài 2: Bi t ñ nh A c a tam giác ABC và 2 ñư ng cao BH và CK. Vi t phương trình các c nh. Cách gi i: +) Vi t phương trình AB, AC v i AB CK AC BH n u n u  =  = uuur uuur uuur uuur +) Tìm B, C v i { } { } B AB BH C AC CK =  = I I Bài t p áp d ng (Các em hãy d a vào ý tư ng Bài 2 ñ gi i các ví d sau) Ví d 1: Cho tam giác ABC bi t ñ nh A(1; –3); phương trình hai ñư ng cao xu t phát t B và C l n lư t là x+ 2y – 8 =0 và 3x + 5y – 1 = 0. Vi t phương trình c nh BC. (ðs: 3x – 2y – 8 = 0) Ví d 2 (A – 2004): Cho hai ñi m A (0; 2) và B( 3− ; 1− ). Tìm t a ñ tr c tâm và t a ñ tâm ñư ng tròn ngo i ti p c a tam giác OAB. (ðs: H( 3; 1)− , ( 3;1)I − )
  • 14. 14 Bài 3: Cho ñ nh A và hai ñư ng trung tr c 1 2,d d c a c nh AB và AC (ho c BC).Vi t phương trình các c nh. TH1 TH2 Cách gi i: TH1: B, C l n lư t ñ i x ng v i A qua 1d và 2d B C  ⇒   TH2: +) B ñ i x ng v i A qua 1d ⇒B +) C ñ i x ng v i B qua 2d ⇒ C Bài t p áp d ng (Các em hãy d a vào ý tư ng Bài 3 ñ gi i các ví d sau) Ví d 1: Cho tam giác ABC có ñ nh A(1; 2) và hai ñư ng trung tr c c a c nh AB và AC l n lư t là x – 2y – 2 = 0 và x – y + 5 = 0. Vi t phương trình trung tuy n AM c a tam giác ABC. (ðs: y = 2) Ví d 2 : Cho tam giác ABC có ñi m M(0; 3) thu c ño n AC; hai ñư ng trung tr c c a c nh AB và AC l n lư t có phương trình là x – 2y – 2 = 0 và x – y + 5 = 0. Vi t phương trình c nh BC c a tam giác ABC bi t AC = 4AM. (ðs: 4x + 3y – 6 = 0) Bài 4: Bi t ñ nh A c a tam giác ABC và ñư ng cao BH, trung tuy n CM. L p phương trình các c nh. Cách gi i: Bài t p áp d ng (Các em hãy d a vào ý tư ng Bài 4 ñ gi i các ví d sau) Ví d 1: Cho tam giác ABC có A(–4; – 5) và phương trình ñư ng cao BH: x + 2y – 2 = 0, ñư ng trung tuy n CM: 8x – y – 3 = 0. Tìm t a ñ ñ nh B, C. (ðs: B(4; –1), C(1; 5)) Ví d 2: Cho tam giác ABC có phương trình c a trung tuy n AM và ñư ng cao BH l n lư t là: 2x + y – 3 = 0; 2x – y – 4 = 0. ði m 1 5 ; 2 2 N   −    thu c ño n BC và ñ nh C thu c ñư ng th ng d: x + y – 3 = 0. Vi t phương trình ñư ng tròn ngo i ti p tam giác ABC bi t BC không song song v i hai tr c t a ñ . (ðs: 2 2 3 1 5 2 2 2 x y     − + + =        ) Ví d 3: Cho tam giác ABC có ñ nh A(2;1) , ñư ng cao qua ñ nh B có phương trình là : x – 3y – 7 = 0 và ñư ng trung tuy n qua ñ nh C có phương trình : x + y + 1 = 0.Xác ñ nh t a ñ c a B và C. (ðs: ( 2; 3), (4; 5)B C− − − ) Ví d 4: Cho tam giác ABC cân t i ñ nh A có tr ng tâm 4 1 ; 3 3 G       , phương trình ñư ng th ng BC là x – 2y – 4 = 0 và ñư ng th ng BG là 7x – 4y – 8 = 0.Tìm t a ñ các ñ nh A,B,C. (ðs: 16 19 52 8 ; , (0; 2), ; 9 9 9 9 A B C     − −        ) Ví d 5:Cho tam giác ABC có ñ nh A thu c ñư ng th ng d : x – 4y – 2 = 0 , c nh BC song song v i ñư ng th ng d. Phương trình ñư ng cao BH : x + y + 3 = 0 và trung ñi m c a c nh AC là M(1;1). Tìm t a ñ các ñ nh A,B,C. (ðs: 2 2 18 3 8 8 ; , ; , ; 3 3 5 5 3 3 A B C       − − −            )
  • 15. 15 Bài 5: Bi t ñ nh A và trung tuy n CC’, ñư ng trung tr c c a c nh BC. Tìm t a ñ B, C. Cách gi i: Bài t p áp d ng (Các em hãy d a vào ý tư ng Bài 5 ñ gi i các ví d sau) Ví d 1: Cho tam giác ABC bi t A(5; 13). Phương trình ñư ng trung tr c c nh BC, ñư ng trung tuy n CC’ (C’ thu c AB) l n lư t là x + y – 6 = 0 và 2x – y + 1 = 0. Vi t phương trình c nh BC. (ðs: x – y + 2 = 0) Ví d 2 : Cho tam giác ABC có ñư ng trung tr c c a c nh BC c t ñương th ng ñi qua AB t i ñi m (1;2)M và song song v i ñư ng th ng 2 2013 0x y− + = bi t 2AB MA= uuur uuur và ñư ng trung tuy n xu t phát t ñ nh C có phương trình 11 7 11 0x y+ + = . Tìm t a ñ 3 ñ nh c a tam giác ABC. (ðs: 2 11 (0;1), ( 2; 1), ; 5 5 A B C   − − −    ) Bài 6: Bi t trung ñi m M c a AB và trung tuy n AN, ñư ng cao BH. Vi t phương trình các c nh c a ∆ ABC. Cách gi i: Bài t p áp d ng (Các em hãy d a vào ý tư ng Bài 6 ñ gi i các ví d sau) Ví d 1: Tam giác ABC có ñư ng trung tuy n AN : x – y + 1 = 0, ñư ng cao BH : x + 2y – 1= 0, ño n AB có trung ñi m M(1; 1). Vi t phương trình các c nh c a tam giác ABC. (ðs: AB: x = 1; AC: 2x – y = 0; BC: 3x – y – 3 = 0) Ví d 2: Cho tam giác ABC có ñi m M(0; 3) là trung ñi m c a AB. Phương trình trung tuy n AN: 2x – y – 2 = 0, ñư ng cao BH: x – 3y + 14 = 0.Vi t phương trình ñư ng tròn ngo i ti p tam giác ABC (ðs: 2 2 ( 3) ( 3) 50x y+ + + = )
  • 16. 16 Bài 7: Bi t ñ nh A (ho c ñư ng cao xu t phát t A ñi qua ñi m N và tr ng tâm G thu c m t ñư ng th ng…) c a tam giác ABC và trung tuy n BM, ñư ng cao BH. Vi t phương trình các c nh. TH1 TH2 Cách gi i: Bài t p áp d ng (Các em hãy d a vào ý tư ng Bài 7 ñ gi i các ví d sau) Ví d 1:Cho tam giác ABC bi t ñ nh A(1; – 1), ñư ng cao và trung tuy n cùng xu t phát t B l n lư t có phương trình: x + 2y – 3 = 0 và x + 3y – 5 = 0 . Vi t phương trình BC. (ðs: x – 4y + 9 = 0) Ví d 2: Cho tam giác ABC bi t ñư ng cao BH và trung tuy n BM l n lư t có phương trình: 4x + 3y + 2 = 0; x – 1 = 0. Tính di n tích tam giác ABC bi t r ng tr ng tâm G c a tam giác thu c ñư ng th ng d: 2x + 3y – 1 = 0 và ñư ng cao xu t phát t ñ nh A có hoành ñ âm ñi qua ñi m N(3; –3). (ðs: 5ABCS∆ = (ñvdt)) Ví d 3 (D – 2009): Cho tam giác ABC có M(2; 0) là trung ñi m c a c nh AB. ðư ng trung tuy n và ñư ng cao qua ñ nh A l n lư t có phương trình là 7x – 2y – 3 = 0 và 6x – y – 4 = 0. Vi t phương trình ñư ng th ng AC. (ðs: 3x – 4y + 5 = 0) Ví d 4 (B – 2003): Cho tam giác ABC có AB = AC , BAC∠ = 900 . Bi t M(1; -1) là trung ñi m c nh BC và G 2 ;0 3       là tr ng tâm c a tam giác ABC. Tìm t a ñ các ñ nh A, B, C. (ðs: A(0; 2), B(4; 0), C(–2; – 2)) Ví d 5 (A – 2009): Cho hình ch nh t ABCD có ñi m I(6; 2) là giao ñi m c a hai ñư ng chéo AC và BD. ði m M(1; 5) thu c ñư ng th ng AB và trung ñi m E c a c nh CD thu c ñư ng th ng ∆ : x + y – 5 = 0. Vi t phương trình ñư ng th ng AB. (ðs: x – 4y + 19 = 0 và y – 5 = 0) Bài 8: S d ng ñi u ki n vuông góc (trư ng h p riêng c a Bài 19) ñ gi i bài toán. Cách gi i: *) G i t a ñ các ñi m (n u chưa bi t) liên quan t i y u t vuông góc theo m t n nh vào: +) ñi m thu c ñư ng th ng. +) ñi m có m i liên h v i ñi m khác: trung ñi m, tr ng tâm, th a mãn h th c véctơ…
  • 17. 17 *) “C t nghĩa” ñi u ki n vuông góc: 0 . 0 . 0 . 0 90 . 0 a b a b AB MN AB MN n n a b u u AMB MA MB  ⊥ ⇔ =   =  ⊥ ⇒  =   ∠ = ⇔ = uuur uuuur uur uur uur uur uuur uuur ( ) 0 ?f t t⇒ = ⇒ = ⇒ t a ñ các ñi m Bài t p áp d ng (Các em hãy d a vào ý tư ng Bài 8 ñ gi i các ví d sau) Ví d 1 (D – 2004): Cho tam giác ABC có các ñ nh A(-1; 0); B (4; 0); C(0;m) v i m ≠ 0. Tìm t a ñ tr ng tâm G c a tam giác ABC theo m. Xác ñ nh m ñ tam giác GAB vuông t i G. (ðs: (1; ), 3 6 3 m G m = ± ) Ví d 2 (D – 2008): Cho (P): 2 16y x= và ñi m A(1; 4). Hai ñi m phân bi t B, C (B và C khác A) di ñ ng trên (P) sao cho BAC∠ = 0 90 . Ch ng minh r ng ñư ng th ng BC luôn ñi qua m t ñi m c ñ nh. (ðs: ñi m c ñ nh I(17; –4)) Ví d 3 (A – 2009): Cho hình ch nh t ABCD có ñi m I(6; 2) là giao ñi m c a hai ñư ng chéo AC và BD. ði m M(1; 5) thu c ñư ng th ng AB và trung ñi m E c a c nh CD thu c ñư ng th ng ∆ : x + y – 5 = 0. Vi t phương trình ñư ng th ng AB. (ðs: x – 4y + 19 = 0 và y – 5 = 0) Ví d 4: Cho ñi m M(3; 3), vi t phương trình ñư ng th ng ñi qua I(2; 1) c t Ox, Oy l n lư t t i A và B sao cho tam giác AMB vuông t i M. (ðs: x + 2y – 4 = 0 và x + y – 3 = 0). CHÚ Ý: Qua các bài toán trên liên quan t i y u t trung tuy n và ñư ng cao, ñư ng trung tr c các em có th rút ra ñư c m t vài ñi u như sau (tuy ñơn gi n nhưng hư ng tư duy này s giúp chúng ta gi i quy t t t nh ng bài toán d ng trên): +) N u M là trung ñi m c a AB 2 2 A B M A B M x x x y y y + = ⇒  + = : nghĩa là khi có d ki n này s giúp chúng ta thi t l p ñư c 2 phương trình +) AH là ñư ng cao c a BC: giúp chúng ta bi t ñư c phương c a ñư ng này n u bi t ñư ng kia. +) d là trung tr c c a BC: nghĩa là B ñ i x ng v i C qua d. Lo i 1.2: Các bài toán v ñư ng phân giác trong Bài 9: Bi t ñ nh A và hai ñư ng phân giác trong BB’ và CC’. L p phương trình BC. Cách gi i: +) Tìm 1A ñ i x ng v i A qua BB’ 1A BC⇒ ∈ (1) +) Tìm 2A ñ i x ng v i A qua CC’ 2A BC⇒ ∈ (2) +) T (1) và (2) ⇒ phương trình BC (chính là phương trình 1 2A A ) Bài t p áp d ng (Các em hãy d a vào ý tư ng Bài 9 ñ gi i các ví d sau) Ví d 1: Trong m t ph ng t a ñ Oxy, cho tam giác ABC có ñ nh A(1; 1), phương trình ñư ng phân giác trong góc B, góc C l n lư t là BD: 2x + y + 4 = 0; CE: x + 3y + 1 = 0. L p phương trình c nh BC. (ðs:x + 23y + 46 = 0) Ví d 2: Trong m t ph ng t a ñ Oxy, cho tam giác ABC có phương trình ñư ng phân giác trong góc B, góc C l n lư t là BD: 6x + 8y – 17 = 0; CE: x – 2y + 3 = 0, ñi m M 17 1; 7   −    và N 1 1; 3       l n lư t thu c c nh AB, AC. Tìm t a ñ các ñ nh c a tam giác ABC. (ðs:A(0; – 1), B 7 ;3 6   −    , C(3; 3))
  • 18. 18 Bài 10: Bi t ñ nh A và trung tuy n BM, phân giác trong BD. Vi t phương trình các c nh. Cách gi i: Bài t p áp d ng (Các em hãy d a vào ý tư ng Bài 10 ñ gi i các ví d sau) Ví d 1: Cho tam giác ABC có ñ nh A(3; 4), ñư ng phân giác trong và trung tuy n xu t phát t ñ nh B l n lư t có phương trình x – y + 1 = 0 và 2x + 3y – 4 = 0. Tìm t a ñ ñ nh C. (ðs:C(–1; 6)) Ví d 2: Cho tam giác ABC có ñ nh A(1; 2), ñư ng phân giác trong và trung tuy n xu t phát t ñ nh B l n lư t có phương trình 2x + y – 1 = 0 và 2x + 3y – 3 = 0. Tìm t a ñi m D là chân ñư ng phân giác trong c a góc B xu ng AC. ( ðs: D( 5;11)− ) Bài 11: Bi t ñ nh A và trung tuy n BM, phân giác trong CD. Vi t phương trình các canh. Cách gi i: Bài t p áp d ng (Các em hãy d a vào ý tư ng Bài 11 ñ gi i các ví d sau) Ví d 1: Trong m t ph ng Oxy cho tam giác ABC có ñ nh A(1; 2), ñư ng trung tuy n BM: 2x + y + 1 = 0 và phân giác trong CD: x + y – 1 = 0. Vi t phương trình ñư ng th ng BC (ðs: 4x + 3y + 4 = 0). Ví d 2: Cho tam giác ABC có chân ñư ng trung tuy n k t B xu ng AC là M(1; – 1), ñư ng phân giác trong c a góc C là x + y – 2 = 0. Vi t phương trình c nh AC bi t ñi m N(–7; 7) thu c c nh BC. (ðs: 5x + 3y – 2 = 0) Ví d 3 (D – 2011 ): Cho tam giác ABC có ñ nh B(– 4; 1), tr ng tâm G(1; 1) và ñư ng th ng ch a phân giác trong c a góc A có phương trình x – y – 1 = 0. Tìm t a ñ các ñ nh A và C. (ðs: (4;3), (3; 1)A C − )
  • 19. 19 Bài 12: Bi t ñ nh A và ñư ng cao BH, phân giác trong BD. Vi t phương trình các c nh tam giác. Cách gi i: Bài t p áp d ng (Các em hãy d a vào ý tư ng Bài 12 ñ gi i các ví d sau) Ví d : Tam giác ABC có A(-3;1), ñư ng cao BH, phân giác trong BD l n lư t có phương trình: x + 7y + 32 = 0 và x + 3y + 12 = 0. Tìm t a ñ ñi m C. (ðs: C(–4; – 6) ) Bài 13: Bi t ñ nh A và ñư ng cao BH, phân giác trong CD. Vi t phương trình các c nh tam giác. Cách gi i: Bài t p áp d ng (Các em hãy d a vào ý tư ng Bài 13 ñ gi i các ví d sau) Ví d 1: Cho tam giác ABC có C(–2; 3). ðư ng cao c a tam giác k t ñ nh A và ñư ng phân giác trong c a góc B l n lư t là: 3x – 2y – 25 = 0; x – y = 0. Vi t phương trình c nh AC c a tam giác. (ðs: 8x + 7y – 5 = 0) Ví d 2 (B – 2008): Trong m t ph ng v i h t a ñ Oxy, hãy xác ñ nh t a ñ ñ nh C c a tam giác ABC bi t r ng hình chi u vuông góc c a C trên ñư ng th ng AB là ñi m H(– 1; – 1), ñư ng phân giác trong c a góc A có phương trình x – y + 2 = 0 và ñư ng cao k t B có phương trình 4x + 3y – 1 = 0. (ðs: 10 3 ; 3 4 C   −    ) CHÚ Ý: Như v y qua các bài toán liên quan ñ n ñư ng phân giác trong c a tam giác các em s nh n th y ta luôn tìm thêm ñi m ñ i x ng v i ñi m ñã bi t t a ñ trên c nh k c a góc ch a phân giác qua phân giác ñó , và ñi m ñó s thu c c nh k còn l i (ñây là ñ c ñi m luôn ñư c khai thác khi có bài toán ch a phân giác)
  • 20. 20 Lo i 2: Các bài toán v ð nh Lư ng Bài 14: Bi t ñ nh A ho c tr ng tâm G c a tam giác ABC thu c m t ñư ng th ng d cho trư c. Bi t t a ñ ñ nh B, C và di n tích tam giác ABC( ho c di n tích c a 1 trong 3 tam giác ABG, BCG, CAG) . Tìm t a ñ ñ nh A.(N u bi t thêm trung tuy n AM thì thay d ki n bi t t a ñ B, C b i bi t ñư ng th ng BC và câu h i là tìm t a ñ ñ nh B, C) TH1 TH2 Cách gi i: (trư ng h p riêng c a Bài 16) Bài t p áp d ng (Các em hãy d a vào ý tư ng Bài 14 ñ gi i các ví d sau) Ví d 1 (B – 2004): Cho hai ñi m A(1; 1), B(4; -3). Tìm ñi m C thu c ñư ng th ng x – 2y – 1 = 0 sao cho kho ng cách t C ñ n ñư ng th ng AB b ng 6. ( ðs: (7;3)C ho c 43 27 ; 11 11 C   − −    ) Ví d 2: Cho tam giác ABC v i A(2; –1), B(1; –2), tr ng tâm G c a tam giác n m trên ñư ng th ng x + y – 2 = 0. Tìm t a ñ ñ nh C bi t tam giác ABC có di n tích là 13,5. (ðs: C(15; –9) ho c C(–12;18)) Ví d 3: Cho tam giác ABC v i A(1; 1), B(–2; 5), tr ng tâm G thu c ñư ng th ng 1∆ : 2x + 3y – 1 = 0, ñ nh C thu c ñư ng th ng 2∆ : x + y – 1 = 0. Tính di n tích tam giác ABC. (ðs: 6ABCS∆ = (ñvdt)) CHÚ Ý: Tam giác ABC có G là tr ng tâm thì: 1 3 ABG BCG CAG ABCS S S S∆ ∆ ∆ ∆= = =
  • 21. 21 Bài 15: Bi t ñ nh A và phương trình ñư ng th ng BC và hình chi u H c a A xu ng BC chia theo BH kHC= uuur uuur và bi t di n tích tam giác ABC (ho c bi t ñ dài ño n BC). Tìm t a ñ B, C. Cách gi i: Bài t p áp d ng (Các em hãy d a vào ý tư ng Bài 15 ñ gi i các ví d sau) Ví d 1:Cho tam giác ABC có ñ nh A(2; – 2) và phương trình ñư ng th ng BC là 3x – 4y + 1 = 0 và hình chi u H c a A xu ng BC th a mãn 6HC BH= − uuur uuur . Tìm t a ñ các ñi m B và C, bi t di n tích tam giác ABC b ng 7,5 . (ðs: B(1; 1), C(5; 4)) Ví d 2 (B – 2009 – NC): Cho tam giác ABC cân t i A có ñ nh A(–1;4) và các ñ nh B,C thu c ñư ng th ng ∆: x – y – 4 = 0. Xác ñ nh to ñ các ñi m B và C, bi t di n tích tam giác ABC b ng 18. ( ðs: 11 3 3 5 ; , ; 2 2 2 2 B C     −        ho c 3 5 11 3 ; , ; 2 2 2 2 B C     −        ) Ví d 3 (B – 2002): Cho hình ch nh t ABCD có tâm I 1 ;0 2       , phương trình ñư ng th ng AB là x – 2y + 2 = 0 và AB = 2AD. Tìm t a ñ các ñi m A, B, C, D bi t r ng A có hoành ñ âm. (ðs: ( 2;0), (2;2), (3;0), ( 1; 2)A B C D− − − ) Bài 16: TH1: Bi t ñ nh A và phương trình ñư ng th ng BC, ñư ng th ng d ñi qua ñi m H thu c BC th a mãn BH kHC= uuur uuur và bi t di n tích tam giác ABC (ho c bi t ñ dài ño n BC). Tìm t a ñ B, C. TH2: Bi t phương trình AC và bi t phương trình ñi qua A căt BC t i H (bi t A), bi t B ( ho c C) và th a mãn BH.BC = k. Tìm ñ nh còn l i. BH.BC = k TH1 TH2
  • 22. 22 Cách gi i: Bài t p áp d ng (Các em hãy d a vào ý tư ng Bài 16 ñ gi i các ví d sau) Ví d 1:Cho tam giác ABC có ñ nh A(1; 1), c nh BC có phương trình 3x – 4y + 6 = 0. ðư ng th ng d c t ño n BC t i ñi m H sao cho HC = 3BH. Xác ñ nh t a ñ ñi m B, C bi t ñư ng th ng d có phương trình x – 4y + 8 = 0 và tam giác ABC có di n tích b ng 1,5. (ðs: B(2; 3), C(-2; 0) ) Ví d 2 (B – 2011): Cho hai ñư ng th ng ∆ : x – y – 4 = 0 và d: 2x – y – 2 = 0. Tìm t a ñ ñi m N thu c ñư ng th ng d sao cho ñư ng th ng ON c t ñư ng th ng ∆ t i ñi m M th a mãn OM.ON = 8. (ðs: (0; 2)N − ho c 6 2 ; 5 5 N       Bài 17: Bi t t a ñ ñi m A, ñư ng phân giác trong c a góc B và cho bi t ñ l n góc B và di n tích tam giác ABC. Tìm t a ñ B,C Cách gi i: Bài t p áp d ng (Các em hãy d a vào ý tư ng Bài 17 ñ gi i ví d sau)
  • 23. 23 Ví d (B – 2010): Cho tam giác ABC vuông t i A có ñ nh C(– 4; 1), phân giác trong góc A có phương trình x + y – 5 = 0. Vi t phương trình ñư ng th ng BC, bi t di n tích tam giác ABC b ng 24 và ñ nh A có hoành ñ dương. Bài 18: Tam giác ABC cân t i A, bi t AB và BC n m l n lư t trên 2 ñư ng th ng 1 2,d d . Bi t M 0 0( ; )x y ∈ AC. Tìm t a ñ các ñ nh. Cách gi i: C2: +) Tìm 1 2{ }B d d= I +) Vi t phương trình 3d qua M song song v i 2d +) Tìm 1 3{ }N d d= I ⇒ phương trình trung tr c 4d c a MN 4 1{ }A d d⇒ = I +) Vi t phương trình AM 2{C} AM d⇒ = I NH N XÉT: C2 hay hơn C1 Bài t p áp d ng (Các em hãy d a vào ý tư ng Bài 18 ñ gi i các ví d sau) Ví d 1: Cho tam giác ABC cân t i A có phương trình hai c nh BC, AB l n lư t là: x – 3y – 1 = 0 và x – y – 5 = 0. ðư ng th ng AC ñi qua M(–4; 1). Tìm t a ñ ñ nh C. (ðs: 8 1 ; 5 5 C       ). Ví d 2: Trong m t ph ng Oxy, hãy xác ñ nh t a ñ các ñ nh c a tam giác ABC vuông cân t i A. Bi t r ng c nh huy n n m trên ñư ng th ng d: x + 7y – 31 = 0, ñi m N(7; 7) thu c ñư ng th ng AC, ñi m M(2; –3) thu c AB và n m ngoài ño n AB. (ðs: ( 1;1), ( 4;5), (3;4)A B C− − ) Ví d 3: Cho tam giác ABC cân t i A có phương trình AB, BC l n lư t là y + 1 = 0 và x + y – 2 = 0. Tính di n tích
  • 24. 24 tam giác ABC bi t AC ñi qua ñi m M(–1; 2) (ðs: 8ABCS∆ = ). Ví d 4 (A – 2010 – NC): Cho tam giác ABC cân t i A có ñ nh A(6; 6); ñư ng th ng ñi qua trung ñi m c a các c nh AB và AC có phương trình x + y – 4 = 0. Tìm t a ñ các ñ nh B và C, bi t ñi m E(1; - 3) n m trên ñư ng cao ñi qua ñ nh C c a tam giác ñã cho. (ðs: (0; 4), ( 4;0)B C− − ho c ( 6;2), (2; 6)B C− − ) Ví d 5 (B – 2007): Cho ñi m A(2; 2) và các ñư ng th ng 1d : x + y – 2 = 0, 2d : x + y – 8 = 0. Tìm t a ñ các ñi m B và C l n lư t thu c 1d và 2d sao cho tam giác ABC vuông cân t i A. (ðs: ( 1;3), (3;5)B C− ho c (3; 1), (5;3)B C− ) Ví d 6 (B – 2011 – NC): Cho tam giác ABC có ñ nh B 1 ;1 2       . ðư ng tròn n i ti p tam giác ABC ti p xúc v i các c nh BC, CA, AB tương ng t i các ñi m D, E, F. Cho D(3; 1) và ñư ng th ng EF có phương trình y – 3 = 0. Tìm t a ñ ñ nh A, bi t A có tung ñ dương. ( ðs: 13 3; 3 A       ) Bài 19: Các ñi m liên h v i nhau b i m t n và m t ñi u ki n v ñ nh lư ng Cách gi i: +) Khai thác d ki n bài toán ñ chuy n các ñi m v 1 n t (nh thu t toán tìm ñi m) +) Thi t l p phương trình: ( ) 0 ?f t t= ⇒ = ⇒ các ñi m c n tìm. CHÚ Ý: Bài 8 là trư ng h p ñ c bi t c a Bài 19 khi ñi u ki n ñ nh lư ng là ñi u ki n góc 0 90 (vuông góc). Bài t p áp d ng (Các em hãy d a vào ý tư ng Bài 19 ñ gi i các ví d sau) Ví d 1: Cho tam giác ABC vuông t i A. Hai ñi m A, B thu c tr c hoành. Phương trình c nh BC là 4x + 3y – 16 = 0. Xác ñ nh t a ñ tr ng tâm G c a tam giác ABC, bi t bán kính ñư ng tròn n i ti p tam giác ABC b ng 1. (ðs: G 4 2; 3       ho c G 4 6; 3   −    ) Ví d 2 (A – 2002): Cho tam giác ABC vuông t i A, phương trình ñư ng th ng BC là 3x y 3 0− − = , các ñ nh A và B thu c tr c hoành và bán kính ñư ng tròn n i ti p b ng 2. Tìm t a ñ tr ng tâm G c a tam giác ABC. (ðs: 7 4 3 6 2 3 ; 3 3 G  + +      ho c 4 3 1 6 2 3 ; 3 3 G  − − − −      ) Ví d 3 (D – 2008): Cho (P): 2 16y x= và ñi m A(1; 4). Hai ñi m phân bi t B, C (B và C khác A) di ñ ng trên (P) sao cho góc 0 90BAC∠ = . Ch ng minh r ng ñư ng th ng BC luôn ñi qua m t ñi m c ñ nh. (ðs: ñi m c ñ nh I(17; –4)) Ví d 4 (A – 2006): Cho các ñư ng th ng: 1d : x + y + 3 = 0, 2d : x – y – 4 = 0, 3d : x – 2y = 0. Tìm t a ñ ñi m M n m trên ñư ng th ng 3d sao cho kho ng cách t M ñ n ñư ng th ng 1d b ng hai l n kho ng cách t M ñ n ñư ng th ng 2d . (ðs: ( 22; 11)M − − ho c (2;1)M ) Ví d 5 (B – 2005): Cho hai ñi m A(2;0) và B(6;4). Vi t phương trình ñư ng tròn (C) ti p xúc v i tr c hoành t i ñi m A và kho ng cách t tâm c a (C) ñ n ñi m B b ng 5. (ðs: 2 2 ( ):( 2) ( 1) 1C x y− + − = ho c 2 2 ( ):( 2) ( 7) 49C x y− + − = ) Ví d 6 (A – 2005): Cho hai ñư ng th ng d1 : x – y = 0 và d2 : 2x + y – 1 = 0 tìm t a ñ các ñ nh c a hình vuông ABCD bi t r ng ñ nh A thu c d1 , ñ nh C thu c d2 và các ñ nh B, D thu c tr c hoành. (ðs: (1;1), (0;0), (1; 1), (2;0)A B C D− ho c (1;1), (2;0), (1; 1), (0;0)A B C D− ) Ví d 7 (D – 2006): Cho ñư ng tròn 2 2 ( ) : 2 2 1 0C x y x y+ − − + = và ñư ng th ng d: x – y + 3 = 0. Tìm t a ñ ñi m M n m trên d sao cho ñư ng tròn tâm M, có bán kính g p ñôi bán kính ñư ng tròn (C), ti p xúc ngoài v i ñư ng tròn (C). ( ðs: (1;4)M ho c ( 2;1)M − ) Ví d 8 (D – 2004): Cho tam giác ABC có các ñ nh A(-1; 0); B (4; 0); C(0;m) v i m ≠ 0. Tìm t a ñ tr ng tâm G c a
  • 25. 25 tam giác ABC theo m. Xác ñ nh m ñ tam giác GAB vuông t i G. (ðs: (1; ), 3 6 3 m G m = ± ) Ví d 9 (B – 2002): Cho hình ch nh t ABCD có tâm I 1 ;0 2       , phương trình ñư ng th ng AB là x – 2y + 2 = 0 và AB = 2AD. Tìm t a ñ các ñi m A, B, C, D bi t r ng A có hoành ñ âm. (ðs: ( 2;0), (2;2), (3;0), ( 1; 2)A B C D− − − ) D ng 2: Các bài toán v ñư ng th ng Lo i 1: ði qua m t ñi m và th a mãn m t y u t ñ nh lư ng Cách gi i chung: C1: +) G i phương trình ñi qua ñi m M 0 0( ; )x y có h s góc k có d ng: 0 0( )y k x x y= − + hay 0 0 0kx y y kx− + − = ( ∆ ) +) Sau ñó “c t nghĩa” d ki n v ñ nh lư ng ñ thi t l p phương trình: ( ) 0 ?f k k= ⇒ = ⇒ phương trình ∆ . C2: +) G i phương trình ñi qua ñi m M 0 0( ; )x y có vtpt ( ; )n a b= r ( 2 2 0a b+ ≠ ) có d ng: 0 0( ) ( ) 0a x x b y y− + − = hay 0 0 0ax by ax by+ − − = ( ∆ ) +) Sau ñó “c t nghĩa” d ki n v ñ nh lư ng ñ thi t l p phương trình: ( , ) 0f a b a kb= → = (*) +) T (*) ch n ? ? a b = ⇒ = phương trình ∆ CHÚ Ý: Chúng ta ñã s d ng cách này trong Bài 18 Bài t p áp d ng Ví d 1: Trong m t ph ng t a ñ Oxy, cho hai ñi m M(1; 4) và N(6; 2). L p phương trình ñư ng th ng ∆ qua M sao cho kho ng cách t N t i ∆ b ng 5. (ðs: 21 20 59 0x y− + = và x = 1). Ví d 2: Trong m t ph ng t a ñ Oxy, cho hai ñi m A(1; 2) và B(5; –1). Vi t phương trình ñư ng th ng qua M(3; 5) và cách ñ u A và B. (ðs: 3x + 4y – 29 = 0 và x = 3) Ví d 3: Trong m t ph ng Oxy, cho ñi m M(1; 2). Vi t phương trình ñư ng th ng qua M c t Ox, Oy l n lư t t i hai ñiêm A, B sao cho OAB là tam giác vuông cân. (ðs: x + y – 3 = 0 và x – y + 1 = 0) Ví d 4: Trong m t ph ng Oxy, cho ñi m M(4; 3). Vi t phương trình ñư ng th ng qua M sao cho nó t o v i hai tr c t a ñ m t tam giác có di n tích b ng 3. (ðs: 3 2 6 0x y− − = và 3x – 8y + 12 = 0). Ví d 5: Trong m t ph ng Oxy, cho tam giác ABC có A(4; 1), B(1; 7) và C(-1; 0). Vi t phương trình ñư ng th ng ñi qua C và chia tam giác thành hai ph n b ng nhau, ph n ch a ñi m A có di n tích g p ñôi ph n ch a ñi m B. (ðs: 6x – 5y + 6 = 0). Ví d 6: Trong m t ph ng Oxy, cho ba ñi m A( - 1; 2), B(5; 4) và M(2; 5). Vi t phương trình ñư ng th ng ñi qua M và cách ñ u hai ñi m A và B (ðs: 5x – 3y + 13 = 0 và x = 2) Ví d 7: Trong m t ph ng Oxy, cho ñi m M(9; 4). Vi t phương trình ñư ng th ng qua M, c t hai tia Ox và tia Oy t i A và B sao cho: 1) tam giác OAB có di n tích nh nh t. (ðs: 4x + 9y – 72 = 0) 2) OB + OC nh nh t. (ðs: 4x + 9y – 72 = 0) Ví d 8 : Trong m t ph ng Oxy, cho hình thoi ABCD có c nh AB n m trên ñư ng th ng x – 2y + 5 = 0 và ba ñi m M(–1; 4), N(1; 1), P(–3; 3) l n lư t thu c các c nh BC, CD và AD. Vi t phương trình c nh AD. (ðs: 2 3 0x y+ − = ho c 11 2 39 0x y− + = )
  • 26. 26 CHÚ Ý: +) N u bài toán ñ c p t i các ñi m A(a; 0) và B(0; b) là các giao ñi m v i hai tr c t a ñ các em có th vi t phương trình ñư ng th ng theo ño n ch n ñi qua AB: 1 x y a b + = +) N u A(a; 0) , B(0; b) thì OA = a và OB = b Lo i 2: C t ñư ng tròn, Elip (xem D ng 3, D ng 4) D ng 3: Các bài toán v ñư ng tròn Lo i 1: Vi t phương trình ñư ng tròn và xác ñ nh các y u t c a ñư ng tròn Bài 1: Thi t l p phương trình ñư ng tròn Cách gi i chung: C2: +) G i phương trình ñư ng tròn có d ng 2 2 0x y ax by c+ + + + = +) Tìm a, b, c nh “c t nghĩa” d ki n bài toán Bài t p áp d ng Ví d 1: Vi t phương trình ñư ng tròn: 1) ñư ng kính AB v i A(3; 1) và (B(2; – 2). 2) Có tâm I(1; – 2) và ti p xúc v i ñư ng th ng d: x + y – 2 = 0. 3) Có bán kính b ng 5, tâm thu c tr c hoành và ñi qua A(2; 4). 4) Có tâm I(2; – 1) và ti p xúc ngoài v i ñư ng tròn: 2 2 ( 5) ( 3) 9x y− + − = 5) có tâm n m trên ñư ng th ng ∆ và ti p xúc v i hai tr c t a ñ Ox, Oy. 6) qua A(–2; –1), B(–1; 4) và C(4; 3) (ñư ng tròn ngo i ti p tam giác ABC). 7) qua A(0; 2), B(–1; 1) và có tâm n m trên ñư ng th ng 2x + 3y = 0. 8) qua A(5; 3) và ti p xúc v i ñư ng th ng d: x + 3y + 2 = 0 t i ñi m T(1; –1). 9) N i ti p tam giác OAB bi t A(3; 0) và B(0; 4). Ví d 2(A – 2007): Trong m t ph ng v i h t a ñ Oxy, cho tam giác ABC có A(0; 2), B(– 2; – 2) và C(4; – 2). G i H là chân ñư ng cao k t B; M và N l n lư t là trung ñi m c a các c nh AB và BC. Vi t phương trình ñư ng tròn ñi qua các ñi m H, M, N. ( ðs: 2 2 2 2 0x y z x y+ + − + − = ) Ví d 3(B – 2010 – NC): Trong m t ph ng t a ñ Oxy, cho ñi m A(2; 3 ) và (E): 2 2 1 3 2 x y + = . G i 1F và 2F là các tiêu ñi m c a (E) ( 1F có hoành ñ âm); M là giao ñi m có tung ñ dương c a ñương th ng 1AF v i (E); N là ñi m ñ i x ng c a 2F qua M. Vi t phương trình ñư ng tròn ngo i ti p tam giác 2ANF . (ðs: 2 2 2 3 4 ( 1) 3 3 x y   − + − =     )
  • 27. 27 Bài 2: Xác ñ nh tâm và bán kính. L p phương trình ti p tuy n c a ñư ng tròn Cách gi i chung: *) Xác ñ nh tâm và bán kính trong ñó 2 2 0 2 2 a b c     + − >        : ði u ki n t n t i ñư ng tròn. C2: S d ng h ng ñ ng th c (tách ghép) ñưa ñư ng tròn v d ng: trong ñó 0h > : ði u ki n t n t i ñư ng tròn. *) L p phương trình ti p tuy n c a ñư ng tròn C1: N u bi t ti p ñi m M ⇒ phương trình ti p tuy n d c a (C) t i M nh n IM uuur làm véc tơ pháp tuy n. C2: N u không bi t ti p ñi m thì dùng ñi u ki n : ∆ là ti p tuy n c a (C) ( , )d I R⇔ ∆ = Bài t p áp d ng Ví d 1: Cho ñư ng tròn (C): 2 2 2 4 4 0x y x y+ − + − = 1) Tìm tâm và bán kính c a (C). 2) Cho A(3; – 1). Ch ng minh A là ñi m n m trong ñư ng tròn. Vi t phương trình ñư ng th ng qua A và c t (C) theo m t dây cung có ñ dài nh nh t. 3) Cho d: 3x – 4y = 0. Ch ng minh d c t (C) t i hai ñi m phân bi t M, N và sau ñó tính MN. Ví d 2(Các bài toán cơ b n: Vi t phương trình ti p tuy n t i m t ñi m cho trư c, có phương cho trư c và qua 1 ñi m cho trư c) Vi t phương trình ti p tuy n c a ñư ng tròn: 1) 2 2 ( 3) ( 1) 25x y− + + = t i ñi m có hoành ñ b ng – 1 2) 2 2 4 2 5 0x y x y+ + − − = t i ñi m ñư ng tròn c t tr c Ox. 3) 2 2 2x y+ = có h s góc b ng 1. 4) 2 2 2 24 0x y y+ − − = bi t ti p tuy n vuông góc v i ñư ng th ng 3x – 4y + 2012 = 0. 5) có tâm I(2; 1), bán kính R = 3 và ñi qua ñi m A(–1; 2). Lo i 2: S tương giao Lo i 2.1: S tương giao gi a ñư ng th ng và ñư ng tròn
  • 28. 28 Bài 1: Vi t phương trình ñư ng th ng ∆ ñi qua M 0 0( ; )x y c t ñư ng tròn (C) t i A, B sao cho AB = l. Cách gi i +) G i ( ; )n a b∆ = ⇒ uur phương trình ∆ : 0 0 0 0( ) ( ) 0 : ( ) 0a x x b y y ax by ax by− + − = ⇔ ∆ + − + = +) T (*) ta ch n : ? ? a b = ⇒ = phương trình ∆ ( N u mu n tìm c th A, B ta gi i h : ( )C ∆   ) Bài t p áp d ng (Các em hãy d a vào ý tư ng Bài 1 ñ gi i các ví d sau) Ví d 1: Cho ñư ng tròn 2 2 ( ) : 6 4 12 0C x y x y+ − + − = . Vi t phương trình ñư ng th ng ∆ ñi qua M(1; 3) c t (C) theo dây cung AB có ñ dài b ng 2 . (ðs: x – y + 2 = 0 và x + 41y – 124 = 0) Ví d 2 (A – 2009 – NC): Cho ñư ng tròn 2 2 ( ) : 4 4 6 0C x y x y+ + + + = và ñư ng th ng ∆ : 2 3 0x my m+ − + = , v i m là tham s th c. G i I là tâm c a ñư ng tròn (C). Tìm m ñ ∆ c t (C) t i hai ñi m phân bi t A và B sao cho di n tích tam giác IAB l n nh t. (ðs: 0m = ho c 8 15 m = ) Ví d 3 (B – 2009 – NC): Cho tam giác ABC cân t i A có ñ nh A(–1;4) và các ñ nh B,C thu c ñư ng th ng ∆: x – y – 4 = 0. Xác ñ nh to ñ các ñi m B và C, bi t di n tích tam giác ABC b ng 18. ( ðs: 11 3 3 5 ; , ; 2 2 2 2 B C     −        ho c 3 5 11 3 ; , ; 2 2 2 2 B C     −        ) Ví d 4(D – 2009 – NC): Cho ñư ng tròn 2 2 ( ) :( 1) 1C x y− + = . G i I là tâm c a (C). Xác ñ nh t a ñ ñi m M thu c (C) sao cho IOM∠ = 0 90 . ( ðs: 3 3 ; 2 2 M        ho c 3 3 ; 2 2 M   −     ) Ví d 5: Cho ñư ng tròn 2 2 ( ) : 4 2 15 0C x y x y+ − + − = . G i I là tâm ñư ng tròn (C). Vi t phương trình ñư ng thăng ∆ qua M(1; –3) c t (C) t i A, B sao cho tam giác IAB có di n tích b ng 8 và c nh AB là c nh l n nh t. (ðs: 4x + 3y + 5 = 0 và y + 3 = 0) Ví d 6: Cho ñư ng tròn 2 2 ( ) :( 1) ( 2) 4C x y− + − = và ñi m M(2; 1). Vi t phương trình ñư ng th ng ∆ qua M c t (C) t i 2 ñi m A, B sao cho 1) Dây cung AB l n nh t. (ðs: x + y – 3 = 0) 2) Dây cung AB ng n nh t. (ðs: x – y – 1 = 0) Ví d 7: Cho ñư ng tròn (C) : 2 2 1x y+ = .ðư ng tròn ( C’) tâm I(2;2) c t (C) t i 2 ñi m A,B sao cho 2AB = . Vi t phương trình ñư ng th ng AB. ( ðs: 1 0x y+ + = ho c 1 0x y+ − = )
  • 29. 29 Bài 2: Vi t phương trình ñư ng th ng ∆ bi t 0 0( ; )n a b∆ = uur (ho c ph i tìm nh quan h song song ho c vuông góc) c t ñư ng tròn (C) t i 2 ñi m phân bi t A, B và th a mãn m t ñi u ki n v ñ nh lư ng. Cách gi i: +) Phương trình ∆ có 0 0( ; )n a b∆ = uur : 0 0 0a x b y m+ + = 0 0 a x m y b − − ⇒ = (*) (n u 0 0 0 m b x a − = ⇒ = ) +) Thay (*) vào phương trình ñư ng tròn (C) 2 0ax bx c⇒ + + = (2*) (phương trình ch a tham s m) +) G i 1 1 2 2 1 2( ; ), ( ; ) ,A x y B x y x x⇒ là nghi m c a phương trình (2*). N u 1 2,x x bi u di n theo m : Bài t p áp d ng (Các em hãy d a vào ý tư ng Bài 2 ñ gi i các ví d sau) Ví d 1(D – 2011 – NC): Cho ñi m A(1; 0) và ñư ng tròn (C): 2 2 2 4 5 0x y x y+ − + − = . Vi t phương trình ñư ng th ng ∆ c t (C) t i hai ñi m M và N sao cho tam giác AMN vuông cân t i A. (ðs: 1y = ho c 3y = − ) Ví d 2(D – 2010 – CB ): Cho tam giác ABC có ñ nh A(3; –7), tr c tâm H(3; –1), tâm ñư ng tròn ngo i ti p là I(–2; 0). Xác ñ nh t a ñ ñ nh C, bi t C có hoành ñ dương. (ðs: ( 2 65;3)C − + ) Ví d 3: Cho ñư ng tròn 2 2 ( ) :( 2) ( 3) 10C x y− + − = . Xác ñ nh t a ñ các ñ nh c a hình vuông ngo i ti p ñư ng tròn, bi t c nh AB ñi qua ñi m ( 3; 2)M − − và ñ nh A có hoành ñ dương. ( ðs: A(6; 1), B(0; –1), C(–2; 5), D(4; 7)) Bài 3: Tìm ñi m M thu c ñư ng th ng ∆ và cách m t ñi m c ñ nh I m t kho ng không ñ i (MI = R) Cách gi i : Có th hi u bài toán này theo 2 cách (b n ch t là m t) C2: T a ñ ñi m M là nghi m c a h : ( )C ∆   ( ñây (C) là ñư ng tròn tâm I bán kính R) CHÚ Ý: +)V i C1 chúng ta không c n quan tâm t i bài toán v s tương giao gi a ñư ng th ng và ñư ng tròn (ñ c p C2) và gi i theo phương pháp ñ i s thông thư ng. +) V i C2 ta th y rõ hơn b n ch t c a bài toán. +) C1 và C2 là hai cách trình bày khác nhau c a cùng m t phương pháp th trong gi i h phương trình. +) Có th chúng ta chưa nhìn th y luôn ñi m I . Khi ñó ñ bài thư ng cho bi t ñi m M nhìn ño n AB c ñ nh dư i m t góc vuông (I lúc này là trung ñi m c a AB), và có th ph i thông qua m t vài khâu c t nghĩa v y u t ñ nh lư ng thì ta m i có ñư c MI = R = const… +) Ý tư ng c a Bài toán này xu t hi n r t nhi u trong các kì thi ð i H c các năm qua.
  • 30. 30 Bài t p áp d ng (Các em hãy d a vào ý tư ng Bài 3 ñ gi i các ví d sau) Ví d 1 (A, A1 – 2012 – CB ): Cho hình vuông ABCD. G i M là trung ñi m c a c nh BC, N là ñi m trên c nh CD sao cho CN = 2ND. Gi s 11 1 ; 2 2 M       và ñư ng th ng AN có phương trình 2 3 0x y− − = . Tìm t a ñ ñi m A. (ðs : (1; 1)A − ho c (4;5)A ) Ví d 2 (A – 2011 – CB ): Cho ñư ng th ng ∆: x + y + 2 = 0 và ñư ng tròn (C): 2 2 4 2 0x y x y+ − − = . G i I là tâm c a (C), M là ñi m thu c ∆ . Qua M k các ti p tuy n MA và MB ñ n (C) (A và B là các ti p ñi m). Tìm t a ñ ñi m M, bi t t giác MAIB có di n tích b ng 10. (ðs : (2; 4)M − ho c ( 3;1)M − ) Ví d 3 (A – 2010 – CB): Cho hai ñư ng th ng 1 : 3 0d x y+ = và 2 : 3 0d x y− = . G i (T) là ñư ng tròn ti p xúc v i 1d t i A, c t 2d t i hai ñi m B và C sao cho tam giác ABC vuông t i B. Vi t phương trình c a (T), bi t tam giác ABC có di n tích b ng 3 2 và ñi m A có hoành ñ dương. (ðs : 2 2 1 3 1 22 3 x y     + + + =      ) Ví d 4 (D – 2010 – CB): Cho tam giác ABC có ñ nh A(3; –7), tr c tâm là H(3; –1), tâm ñư ng tròn ngo i ti p là I(–2; 0). Xác ñ nh t a ñ ñ nh C, bi t C có hoành ñ dương. (ðs : ( 2 65;3)C − + ) Ví d 5 (D – 2010 – NC): Cho ñi m A(0; 2) và ∆ là ñư ng th ng ñi qua O. G i H là hình chi u vuông góc c a A trên ∆ . Vi t phương trình ñư ng th ng ∆ , bi t kho ng cách t H ñ n tr c hoành b ng AH. (ðs : ( 5 1) 2 5 2 0x y− − − = ho c ( 5 1) 2 5 2 0x y− + − = ) Ví d 6 (B – 2009 – CB ): Cho ñư ng tròn (C): (x – 2)2 + y2 = 4 5 và hai ñư ng th ng ∆1: x – y = 0 và ∆2: x – 7y = 0. Xác ñ nh to ñ tâm K và bán kính c a ñư ng tròn (C1); bi t ñư ng tròn (C1) ti p xúc v i các ñư ng th ng ∆1, ∆2 và tâm K thu c ñư ng tròn (C). (ðs : 8 4 ; 5 5 K       và bán kính 2 2 5 R = ) Ví d 7 (B – 2009 – NC): Cho tam giác ABC cân t i A có ñ nh A(–1;4) và các ñ nh B,C thu c ñư ng th ng ∆: x – y – 4 = 0. Xác ñ nh to ñ các ñi m B và C, bi t di n tích tam giác ABC b ng 18. (ðs : 11 3 3 5 ; , ; 2 2 2 2 B C     −        ho c 3 5 11 3 ; , ; 2 2 2 2 B C     −        ) Ví d 8 (D – 2007): Cho ñư ng tròn 2 2 ( ) : ( 1) ( 2) 9C x y− + + = và ñư ng th ng d: 3x – 4y + m = 0. Tìm m ñ trên d có duy nh t m t ñi m P mà t ñó có th k ñư c hai ti p tuy n PA, PB t i (C) (A, B là các ti p ñi m) sao cho tam giác PAB ñ u. (ðs : 19m = ho c 41m = − ) Ví d 9 (D – 2006): Cho ñư ng tròn 2 2 ( ) : 2 2 1 0C x y x y+ − − + = và ñư ng th ng d: x – y + 3 = 0. Tìm t a ñ ñi m M n m trên d sao cho ñư ng tròn tâm M, có bán kính g p ñôi bán kính ñư ng tròn (C), ti p xúc ngoài v i ñư ng tròn (C). (ðs : (1;4)M ho c ( 2;1)M − ) Ví d 10 (B – 2005): Cho hai ñi m A(2;0) và B(6;4). Vi t phương trình ñư ng tròn (C) ti p xúc v i tr c hoành t i ñi m A và kho ng cách t tâm c a (C) ñ n ñi m B b ng 5. (ðs : 2 2 ( 2) ( 1) 1x y− + − = ho c 2 2 ( 2) ( 7) 49x y− + − = ) Ví d 11 (B – 2003): Cho tam giác ABC có AB = AC , BAC = 900 . Bi t M(1; -1) là trung ñi m c nh BC và G 2 ;0 3       là tr ng tâm c a tam giác ABC. Tìm t a ñ các ñ nh A, B, C. (ðs : (0;2), (4;0), ( 2; 2)A B C − − ) Ví d 12 (B – 2002): Cho hình ch nh t ABCD có tâm I 1 ;0 2       , phương trình ñư ng th ng AB là x – 2y + 2 = 0 và AB = 2AD. Tìm t a ñ các ñi m A, B, C, D bi t r ng A có hoành ñ âm. (ðs : ( 2;0), (2;2), (3;0), ( 1; 2)A B C D− − − ) Ví d 13: Cho tam giác ABC có tr c tâm là H(–1; 4), tâm ñư ng tròn ngo i ti p là I(–3; 0) và trung ñi m c a c nh BC là M(0; 3). Vi t phương trình ñư ng th ng AB, bi t B có hoành ñ dương. (ðs: 3x + 7y – 49 = 0)
  • 31. 31 Ví d 14: Cho ba ñi m I(1; 1), M(–2; 2) và N(2; –2). Tìm t a ñ các ñ nh c a hình vuông ABCD sao cho I là tâm c a hình vuông, M thu c c nh AB, K thu c c nh CD và A có hoành ñ dương. (ðs: A(1; 5), B(–3; 1), C(1; –3), D(5; 1)) Ví d 15: Cho tam giác ABC có tr ng tâm là G 1 1 ; 3 3       , tâm ñư ng tròn ngo i ti p là I 2 4 ; 5 5   −    và trung ñi m c a c nh BC là M(–1; 2). Vi t phương trình ñư ng th ng AC, bi t B có hoành ñ âm. (ðs: 3x + y – 6 = 0) Ví d 16: Cho ñư ng tròn ( C ) : 2 2 8 6 21 0x y x y+ − + + = và ñư ng th ng d : x + y – 1 = 0.Xác ñ nh t a ñ các ñ nh c a hình vuông ABCD ngo i ti p (C) bi t A thu c d và hoành ñ c a ñi m B l n hơn hoành ñ c a ñi m D) (ðs : (6;5), (6; 1), (2;1), (2; 5)A B C D− − ho c (2;1), (6; 1), (6;5), (2; 5)A B C D− − ) Bài 4: Qua ñi m M 0 0( ; )x y n m ngoài ñư ng tròn (C) có tâm I bán kính R. 1) Vi t phương trình ti p tuy n 1 2,MT MT ñ n ñư ng tròn. 2) Vi t phương trình ñư ng th ng ∆ ñi qua 1 2,T T . 3) Tính di n tích t giác 1 2MT IT . Cách gi i: Cách vi t t ng quát v phương trình ti p tuy n: TH1: N u bi t ti p ñi m T ⇒ ti p tuy n ∆ c a (C) ñi qua T nh n IT uur làm vtpt⇒ phương trình ∆ TH2: N u không bi t ti p ñi m thì dùng ñi u ki n : ∆ là ti p tuy n c a (C) ( , )d I R⇔ ∆ = 1) Như v y v i bài toán này ta s làm theo TH2 : +) G i ∆ ñi qua ñi m M 0 0( ; )x y có vtpt ( ; )n a b= r ( 2 2 0a b+ ≠ ) có d ng: 0 0( ) ( ) 0a x x b y y− + − = hay 0 0 0ax by ax by+ − − = ( ∆ ) +) T (*) ch n ? ? a b = ⇒ = phương trình 1 2,∆ ∆ hay phương trình 1 2,MT MT ( m t trong hai phương trình (*) có th có: a = 0 ho c b = 0) CHÚ Ý: Có th tìm c th t a ñ 1 2,T T nh gi i h : ( )C ∆   2) +) G i 0 0( ; )T x y là ti p ñi m c a ti p tuy n k t M ñ n (C) ( ) . 0 T C MT IT ∈ ⇒  = uuur uur (*) 3) 1 2 1 1 1 1 1 2 2. . . 2 MT IT MT IS S MT IT MT R= = = v i 2 2 1MT MI R= −
  • 32. 32 Bài t p áp d ng (Các em hãy d a vào ý tư ng Bài 4 ñ gi i các ví d sau) Ví d 1(B – 2006): Cho ñư ng tròn: 2 2 ( ) : 2 6 6 0C x y x y+ − − + = và ñi m M(– 3; 1). G i 1T và 2T là các ti p ñi m c a các ti p tuy n k t M ñ n (C). Vi t phương trình ñư ng th ng 1T 2T . (ðs: 2 3 0x y+ − = ) Ví d 2: (A – 2011 – CB): Cho ñư ng th ng ∆: x + y + 2 = 0 và ñư ng tròn (C): 2 2 4 2 0x y x y+ − − = . G i I là tâm c a (C), M là ñi m thu c ∆ . Qua M k các ti p tuy n MA và MB ñ n (C) (A và B là các ti p ñi m). Tìm t a ñ ñi m M, bi t t giác MAIB có di n tích b ng 10. (ðs : (2; 4)M − ho c ( 3;1)M − ) Bài 5:Cho ñư ng th ng ∆ , ñư ng tròn (C) có tâm I và hai ñi m ,M N n m ngoài ñư ng tròn. 1) Tìm ñi u ki n ñ ∆ c t (C) t i hai ñi m phân bi t A, B sao cho di n tích tam giác IAB l n nh t. 2) Tìm K thu c (C) sao cho di n tích tam giác KMN l n nh t, nh nh t. 3) Tìm P thu c ∆ sao cho qua P k hai ti p tuy n 1 2,PT PT sao cho di n tích tam giác 1 2ITT l n nh t. TH1 TH2 TH3 Cách gi i : Bài t p áp d ng (Các em hãy d a vào ý tư ng Bài 5 ñ gi i các ví d sau) Ví d 1 : Cho ñư ng tròn 2 2 ( ) : 2 3 0C x x y− + − = . G i B, C là giao ñi m c a ñư ng th ng : 3 0x y∆ + − = . Hãy tìm các ñi m A trên ñư ng tròn (C) sao cho tam giác ABC có chu vi l n nh t. (ðs : (1 2; 2)A − − ) Ví d 2 : Cho ñư ng tròn 2 2 ( ) : 4 6 12 0C x y x y+ − − + = có tâm I và ñư ng th ng : 4 0x y∆ + − = . Tìm trên ñư ng th ng ∆ ñi m M sao cho ti p tuy n k t M ti p xúc v i (C) t i A, B mà tam giác IAB có di n tích l n nh t. (ðs : 3 3 5 3 3 3 5 3 ( ; ), ( ; ) 2 2 2 2 M M + − − + ) Ví d 3 : Cho ñư ng tròn ngo i ti p tam giác ABC v i A(2 ; - 2), B(4 ; 0), C(3 ; 2 1− ) và ñư ng th ng
  • 33. 33 ∆ : 4 4 0x y+ − = . Tìm trên ñư ng th ng ∆ ñi m M sao cho ti p tuy n c a (C) qua M ti p xúc v i C t i N sao cho di n tích tam giác NAB l n nh t. (ðs : 6 4 (2; 4), ( ; ) 5 5 M M− − ) Bài 6: Vi t phương trình ∆ qua 0 0( ; )M x y c t ñư ng tròn (C) có tâm I, bán kính R t i A, B sao cho MA kMB= . Cách gi i : C1 : +) ð t IH h= 2 2 2 2 MH IM h HA HB R h  = − →  = = − (*) CHÚ Ý: +) Cách gi i trên th y s d ng trư ng h p 1k > ( v i 1k < các em làm tương t ) +) Cách gi i trên th y s d ng 0 0( ; )M x y n m ngoài ñư ng tròn (C) ( 0 0( ; )M x y n m trong (C) các em làm tương t ) C2 : +) Xét phương trình ∆ qua 0 0( ; )M x y có h s góc k có d ng : 0 0( )y k x x y= − + +) Xác ñ nh phương trình hoành ñ giao ñi m c a ∆ và (C) : 2 ( , , ) 0f x x k = (*) +) Dùng vi – et cho (*) và k t h p MA kMB= ?k⇒ = ⇒phương trình ∆ . Bài t p áp d ng (Các em hãy d a vào ý tư ng Bài 6 ñ gi i các ví d sau) Ví d 1 : Cho ñư ng tròn (C): 2 2 2 2 23 0x y x y+ − + − = , ñi m M(7; 3). Vi t phương trình ñư ng th ng ∆ qua M c t ñư ng tròn (C) t i A, B sao cho MA = 3MB. ( ðs : 3y = ho c 12 5 69 0x y− − = ) Ví d 2 : Cho ñi m A(-1 ; 14) và ñư ng tròn (C) tâm I(1 ; -5), bán kính b ng 13. Vi t phương trình ñư ng th ng ∆ ñi qua A c t (C) t i M, N mà kho ng cách t M ñ n AI b ng m t n a kho ng cách t N ñ n AI. (ðs : x + y – 13 = 0 và 433x – 281y +4367 = 0) Lo i 2.2: S tương giao gi a hai ñương tròn
  • 34. 34 TH1: 'R r II+ > TH2: 'R r II+ = TH3: 'R r II+ < TH4: 'R r II− = Ngoài nhau Ti p xúc ngoài C t nhau t i hai ñi m Ti p xúc trong CHÚ Ý: Còn trư ng h p ñ ng nhau. Nhưng trư ng h p này ít ñư c khai thác nên th y không ñ c p ñây. Bài t p áp d ng Ví d 1(D – 2009 – NC): Cho ñư ng tròn 2 2 ( ) :( 1) 1C x y− + = . G i I là tâm c a (C). Xác ñ nh t a ñ ñi m M thu c (C) sao cho 0 30IOM∠ = . (ðs: 3 3 ; 2 2 M        ho c 3 3 ; 2 2 M   −     ) Ví d 2(D – 2003): Cho ñư ng tròn (C) : (x – 1)2 + (y – 2)2 = 4 và ñư ng th ng d : x – y – 1 = 0.Vi t phương trình ñư ng tròn (C’) ñ i x ng v i ñư ng tròn (C) qua ñư ng th ng d. Tìm t a ñ các giao ñi m c a (C) và (C’). (ðs: 2 2 ( 3) 4x y− + = (1;0), (3;2)A B ) Ví d 3 (D – 2006): Cho ñư ng tròn 2 2 ( ) : 2 2 1 0C x y x y+ − − + = và ñư ng th ng d: x – y + 3 = 0. Tìm t a ñ ñi m M n m trên d sao cho ñư ng tròn tâm M, có bán kính g p ñôi bán kính ñư ng tròn (C), ti p xúc ngoài v i ñư ng tròn (C). (ðs: (1;4)M ho c ( 2;1)M − ) Ví d 4: Cho ñư ng tròn 2 2 1( ) : 6 4 7 0C x y x y+ − + − = c t ñư ng tròn 2 2 2( ) :( 6) ( 1) 50C x y+ + − = t i hai ñi m M, N bi t M có hoành ñ dương. Vi t phương trình ñư ng th ng ∆ ñi qua M l n lư t c t 1 2( ),( )C C t i các ñi m th hai A, B sao cho M là trung ñi m c a AB. (ðs: 5x – 7y + 9 = 0) Ví d 5: Cho tam giác ABC n i ti p ñư ng tròn tâm I(6; 6) và ngo i ti p ñư ng tròn tâm K(4; 5), biêt ñ nh A(2; 3). Vi t phương trình c nh BC. (ðs: 3x + 4y – 42 = 0) Ví d 6: Cho ñư ng tròn (C) : 2 2 1x y+ = .ðư ng tròn ( C’) tâm I(2;2) c t (C) t i 2 ñi m A,B sao cho 2AB = . Vi t phương trình ñư ng th ng AB. ( ðs: 1 0x y+ + = ho c 1 0x y+ − = ) D ng 4: Các bài toán v Elip Lo i 1: Vi t phương trình Elip và xác ñ nh các y u t c a Elip Cách gi i chung: +) Gi s phương trình chính t c c a elip có d ng: 2 2 2 2 1 x y a b + = ( )E Bài t p áp d ng Ví d 1: L p phương trình chính t c c a elip (E) bi t: 1) Có ñ dài hai tr c là 6, 4. 2) Có m t ñ nh là (5; 0) và tiêu c là 6. 3) Có m t ñ nh là (0; 3) và ñi qua ñi m M(4; 1).
  • 35. 35 4) ði qua hai ñi m 3 1; 2        và 2 2; 2   −     . 5) Có tiêu ñi m 2 (2;0)F và qua ñi m 5 2; 3       . 6) Có tiêu ñi m 2 (5;0)F và kho ng cách gi a hai ñ nh là 9. 7) Tiêu c là 4 và kho ng cách t m t ñ nh trên tr c nh ñ n tiêu ñi m b ng 5. ( ðs: 1) 2 2 1 9 4 x y + = 2) 2 2 1 25 16 x y + = 3) 2 2 1 18 9 x y + = 4) 2 2 1 4 1 x y + = 5) 2 2 1 9 5 x y + = 6) 2 2 1 181 81 4 4 x y + = 7) 2 2 1 25 21 x y + = ho c 2 2 1 49 45 x y + = ho c 2 2 1 9 5 x y + = ) Ví d 2(A – 2008): Trong m t ph ng v i h t a ñ Oxy, hãy vi t phương trình chính t c c a elip (E) bi t r ng (E) có tâm sai b ng 5 3 và hình ch nh t cơ s c a (E) có chu vi b ng 20. (ðs: 2 2 1 9 4 x y + = ) Ví d 3(B – 2012 – NC): Cho hình thoi ABCD có AC = 2BD và ñư ng tròn ti p xúc v i các c nh c a hình thoi có phương trình 2 2 4x y+ = . Vi t phương trình chính t c c a elip (E) ñi qua các ñ nh A, B, C, D c a hình thoi. Bi t A thu c Ox. (ðs: 2 2 1 20 5 x y + = ) Ví d 3: Trong m t ph ng Oxy, cho elip (E) có ñ dài tr c l n b ng 4 2 ,các ñ nh n m trên tr c nh và các tiêu ñi m c a (E) cùng n m trên m t ñư ng tròn. L p phương trình chính t c c a (E). Ví d 4: Cho elip (E) có ñ dài tr c l n b ng 6, tâm sai b ng m t ph n hai và kho ng cách t m t ñi m M c a (E) ñ n tiêu ñi m 1F (có hoành ñ âm) b ng 7. 1) Tìm kho ng cách t M ñ n tiêu ñi m 2F 2) Vi t phương trình chính t c c a elip (E) và tìm t a ñ ñi m M. Lo i 2: Tìm ñi m thu c Elip +) T (1) và (2) 0 0 ? ? x y = ⇒  = M⇒ CHÚ Ý : N u 0 0( ; ) ( )M x y E∈ ta có th khai thác thêm d ki n: 1 0 2 0 c MF a x a c MF a x a  = +   = −  Bài t p áp d ng Ví d 1: Cho elip (E): 2 2 1 6 2 x y + = 1) Tìm t a ñ giao ñi m c a (E) và ñư ng th ng 3 2y x= − . 2)Tìm trên (E) ñi m M sao cho góc 0 1 2 90F MF∠ = 3) Tìm trên (E) ñi m N sao cho 1 2 6F N F N− = . 1) 3 7 ( 3;1), ; 5 5 A B   −     2) 1 2 3 4( 3;1), ( 3; 1), ( 3;1), ( 3; 1)M M M M− − − − 3) 3 5 ; 2 4 N        ho c 3 5 ; 2 4 N   −    
  • 36. 36 Ví d 2: Cho (E): 2 2 2 2 1 x y a b + = có tiêu ñi m 1 2,F F 1) Cho a = 2, b = 1. Tìm ñi m M sao cho 1 22F M F M= . (ðs: 4 23 ; 273 3 M        ho c 4 23 ; 273 3 M   −     ) 2) Ch ng minh r ng v i m i ñi m M ta luôn có: 2 2 2 1 2.F M F M OM a b+ = + Ví d 3(D – 2005): Cho ñi m C(2;0) và elíp (E): 2 2 1 4 1 x y + = . Tìm to ñ các ñi m A, B thu c (E), bi t r ng hai ñi m A, B ñ i x ng v i nhau qua tr c hoành và tam giác ABC là tam giác ñ u. (ðs: 2 4 3 2 4 3 ; , ; 7 7 7 7 A B     −           ho c 2 4 3 2 4 3 ; , ; 7 7 7 7 A B     −           ) Ví d 4 (A – 2011 – NC) : Cho elip (E) : 2 2 1 4 1 x y + = . Tìm ñi m A và B thu c (E), có hoành ñ dương sao cho tam giác OAB cân t i O và có di n tích l n nh t. (ðs: 2 2 2; , 2; 2 2 A B     −           or 2 2 2; , 2; 2 2 A B     −           ) Ví d 5: Trong m t ph ng t a ñ Oxy, cho elip (E) : 2 2 9 25 225x y+ = . Tìm t a ñ ñi m M thu c (E) sao cho tam giác M 1 2F F vuông t i M. Ví d 6: Cho elip (E) : 2 2 5 9 45x y+ = có tiêu ñi m 1 2,F F . M là m t ñi m b t kì trên (E) và bi u th c 1 2 1 2 1 1 f F M F M F M F M = + + + 1) Ch ng minh chu vi tam giác 1 2F MF không ñ i. Tìm M ñ di n tích tam giác 1 2F MF b ng 2. 2) Tìm M sao cho giá tr c a f l n nh t. Ví d 7: Cho ñi m M di ñ ng elip: 2 2 9 16 144x y+ = và H, K l n lư t là hình chi u c a M lên hai tr c t a ñ . Tìm M ñ di n tích OHMK l n nh t. Lo i 3: S tương giao gi a ñư ng th ng và Elip Cách gi i chung : S tương giao gi a ñư ng th ng : 0Ax By C∆ + + = và (E): 2 2 2 2 1 x y a b + = +) Gi i h 2 2 2 2 0 1 Ax By C x y a b + + =   + =  (I) b ng phương pháp th ( ði u ki n ñ ∆ là ti p tuy n c a (E) : 2 2 2 2 2 A a B b C+ = (ñư c sinh ra t (II) )). Bài t p áp d ng Ví d 1: Cho elip (E): 2 2 4 9 36x y+ = và ñi m M(1; 1). L p phương trình ñư ng th ng qua M và c t (E) t i hai ñi m 1 2,M M sao cho 1 2MM MM= . (ðs: 4x + 9y – 13 = 0) Ví d 2:Cho hai ñi m A ( 3;0)− , B ( 3;0)và ñư ng th ng d: 3 2( 3 1) 3 0x y− − + = . Tìm trên d ñi m M có hoành ñ âm sao cho chu vi tam giác MAB b ng 4 2 3+ . (ðs: 3 1; 2 M   −     )
  • 37. 37 Ví d 3 (D – 2002): Cho elip (E) có phương trình 2 2 1 16 9 x y + = . Xét ñi m M chuy n ñ ng trên tia Ox và ñi m N chuy n ñ ng trên tia Oy sao cho ñư ng th ng MN luôn ti p xúc v i (E). Xác ñ nh t a ñ ñi m M, N ñ ño n MN có ñ dài nh nh t. Tính giá tr nh nh t ñó. (ðs: (2 7;0), (0; 21)M N và GTNN c a MN b ng 7) Ví d 4 (B – 2010 – NC): Cho ñi m A(2; 3 ) và (E): 2 2 1 3 2 x y + = . G i 1F và 2F là các tiêu ñi m c a (E) ( 1F có hoành ñ âm); M là giao ñi m có tung ñ dương c a ñương th ng 1AF v i (E); N là ñi m ñ i x ng c a 2F qua M. Vi t phương trình ñư ng tròn ngo i ti p tam giác 2ANF . (ðs: 2 2 2 3 4 ( 1) 3 3 x y   − + − =     ) Ví d 5: Cho Elip (E) : 2 2 1 64 9 x y + = .Vi t phương trình ti p tuy n d c a (E) bi t d c t 2 tr c t a ñ Ox,Oy l n lư t t i A,B sao cho AO = 2BO. CHÚ Ý: Khi trong bài toán v ñư ng tròn và Elip có y u t min, max chúng ta hay s d ng b t ñ ng th c Cauchy và Bunhiacopxki (2011A – NC, 2002D…) C m ơn các em và các b n ñã ñ c t i li u ! M i ý ki n ñóng góp các em và các b n g i qua E- mail: giaidaptoancap3@yahoo.com ho c ñ a ch : s 9 – Ngõ 880 – B ch ð ng – Hai Bà Trưng – Hà N i ði n tho i : 043.9871450 ho c Dð: 0947141139 L i gi i các bài t p các em có th tham kh o trên web: http://www.facebook.com/giaidaptoancap3